X



トップページ物理
1002コメント392KB
■ちょっとした物理の質問はここに書いてね250■
レス数が1000を超えています。これ以上書き込みはできません。
0001ご冗談でしょう?名無しさん垢版2020/05/06(水) 22:51:09.97ID:ZTOSReZi
★荒らし厳禁、煽りは黙殺
★書き込む前に   >>2   の注意事項を読んでね
★数式の書き方(参考)はこちら   >>3-5   (予備リンク:   >>2-10   )

===質問者へ===

重要 【 丸 投 げ 禁 止 】

・質問する前に
1. 教科書や参考書をよく読む
2. http://www.google.com/ などの検索サイトを利用し、各自で調べる
3. 学生は自分の学年、物理科目の履修具合を書く
4. 宿題を聞くときは、どこまでやってみてどこが分からないのかを書く
・質問に対する回答には返答してね、感謝だけでなく「分からん」とかダメ出しでもOK
・質問するときはage&ID表示推奨
・高度すぎる質問には住人は回答できないかもしれないけれど、了承の上での質問なら大歓迎

===回答者へ===

・丸投げは専用スレに誘導
・不快な質問は無視、構った方が負け
・質問者の理解度に応じた適切な回答をよろしく
・単発質問スレを発見したらこのスレッドへの誘導をよろしくね
・逆に議論が深まりそうなら新スレ立てて移動するのもあり
・板違いの質問は適切な板に誘導を
・不適切な回答は適宜訂正、名回答は素直に賞賛

前スレ:■ちょっとした物理の質問はここに書いてね249■
http://rio2016.5ch.net/test/read.cgi/sci/1583506686/
0002ご冗談でしょう?名無しさん垢版2020/05/06(水) 22:54:59.40ID:???
数式の書き方 ※適切にスペースを入れると読みやすくなります

●括弧: (), [], {}を適切に入れ子にして分かりやすく書く
●スカラー: a,b,...,z, A,...,Z, α,β,...,ω, Α,Β,...,Ω,...(「ぎりしゃ」「あるふぁ〜おめが」で変換)
●ベクトル: V=(v1,v2,...), |V>,V↑, (混同しないならスカラーの記号でいい。通常は縦ベクトル)
●テンソル: T^[i,j,k...]_[p,q,r,...], T[i,j,k,...; p,q,r,...]  (上下付き1成分表示)
●行列: M[i,j], I[i,j]=δ_[i,j] M = [[M[1,1],M[2,1],...], [M[1,2],M[2,2],...],...], I = [[1,0,0,...],[0,1,0,...],...]
 (右は全成分表示。行または列ごとに表示する。例:M=[[1,-1],[3,2]])
●対角行列: diag(a,b) = [[a,0],[0,b]]
●転置行列・随伴行列:M^T, M†("†"は「だがー」で変換可) ●行列式・トレース:|A|=det(A), tr(A)
●複号: a±b("±"は「きごう」で変換可)
●内積・外積: a・b, a×b
●関数・汎関数・数列: f(x), F[x(t)] {a_n}
●平方根: √(a+b) = (a+b)^(1/2) = sqrt(a+b) ("√"は「るーと」で変換可)
●指数関数・対数関数: exp(x+y)=e^(x+y) ln(x)=log_e(x) (底を省略して単にlogと書いたとき多くは自然対数)
 括弧を省略しても意味が容易に分かるときは省略可: sin(x) = sin x
●三角関数、逆三角関数、双曲線関数: sin(a), cos(x+y), tan(x/2), asin(x)=sin^[-1](x), cosh(x)=[e^x+e^(-x)]/2
●絶対値:|x| ●ノルム:||x|| ●共役複素数:z^* = conj(z)
●階乗:n!=n*(n-1)*(n-2)*...*2*1, n!!=n*(n-2)*(n-4)*...
0003ご冗談でしょう?名無しさん垢版2020/05/06(水) 22:55:53.11ID:???
●微分・偏微分: dy/dx=y', ∂y/∂x=y_x ("∂"は「きごう」で変換可)
●ベクトル微分: ∇f=grad f, ∇・A=div A,∇xA=rot A, (∇^2)f=Δf ("∇"は「きごう」,"Δ"は「でるた」で変換可.)
●積分: ∫[0,1] f(x)dx = F(x)|_[x=0,1], ∫[y=0,x] f(x,y)dy, ∬[D] f(x,y)dxdy, ∬[C] f(r)dl 
("∫"は「いんてぐらる」,"∬"は「きごう」で変換可)
●数列和・数列積: Σ[k=1,n] a(k), Π[k=1,n] a(k) ("Σ"は「しぐま」,"Π"は「ぱい」で変換可)
文脈によっては単に同じ添字が2回出てきただけで a_i b_i = Σ[i] a_i b_i と積の総和をとることも(Einsteinの縮約)
●極限: lim[x→∞] f(x) ("∞"は「むげんだい」で変換可)
●論理・集合: "⇔⇒∀∃∧∨¬∈∋⊆⊇⊂⊃∪∩"は「きごう」で変換
●等号・不等号: "≠≒<>≦≧≪≫"は「きごう」で変換

読みやすい書き方の例:∫[-∞,∞] exp{ -Σ[i,j=1,n] A_[i,j](x_i)(x_j) } dx = √(π^n/det A)

読みにくい書き方の例:∫[-∞,∞]exp(-Σ[i,j=1,n]A_[i,j]x_ix_j)dx=√(π^n/det A)
0004ご冗談でしょう?名無しさん垢版2020/05/06(水) 22:56:36.79ID:???
a:加速度、昇降演算子 A:振幅、ベクトルポテンシャル B:磁束密度 c:光速 C:定数、熱・電気容量
d:次元、深さ D:領域、電束密度 e:自然対数の底、素電荷 E:エネルギー、電場
f:周波数 f,F:力 F:Helmholtzエネルギー g:重力加速度、伝導度
G:万有引力定数、Gibbsエネルギー、重心 h:高さ、プランク定数 H:エンタルピー、Hamiltonian、磁場
i:虚数単位 i,j,k,l,m:整数のインデックス I:電流、慣性モーメント j:電流密度・流束密度
J:グランドポテンシャル、一般の角運動量 k:バネ定数、波数、Boltzmann定数 K:運動エネルギー
l,L:長さ L:Lagrangian、角運動量、インダクタンス m,M:質量 n:物質量 N:個数、トルク
M:磁化 O:原点 p:双極子モーメント p,P:運動量、圧力 P:分極 q:波数
q,Q:一般化座標、電荷 Q:熱 r:距離 R:抵抗、気体定数 S:エントロピー、面積 t:時間 T:温度
U:ポテンシャル、内部エネルギー v:速度 V:体積、ポテンシャル、電位
W:仕事、状態数 x,y,z:変数、位置 z:複素変数 Z:分配関数

β:逆温度 γ:抵抗係数 Γ:ガンマ関数 δ:微小変化 Δ:変化 ε:微小量、誘電率 θ:角度
λ:波長 μ:換算質量、化学ポテンシャル、透磁率 ν:周波数 π:円周率 ρ:(電荷)密度、抵抗率
σ:スピン φ:角度、ポテンシャル、波動関数 ψ:波動関数 ω:角振動数 Ω:状態密度
0016ご冗談でしょう?名無しさん垢版2020/05/07(木) 19:32:56.06ID:???
>>13
貼った画像なんて危ないものを見る気はないな
ここに直接書く画像が簡単に書けて綺麗なら良いんだが
TeXコマンドは問題ないが、見れない人が多そう
0018ご冗談でしょう?名無しさん垢版2020/05/07(木) 19:37:09.15ID:???
テニスで軽いラケットを使うと球威(球の持つエネルギー)に負けて打ち返しにくいというのは何となくわかるのですが
同じ理屈で卓球でも重いラケットは球威に負けないと言う俗説があります
個人的にはピン球の球威なんて知れているので軽いラケットを早く振った方が球威が出ると思うのですが
物理的な考察としてはどうでしょうか?

ピン球2.7g
重いラケット190g、軽いラケット150g
一般男子のラリーは初速60km/h、手元で40km/hくらいです

ちなみにテニスはボール57g
フェラデーのクソ重ラケットで340g、一般男子300g
サーブは150km/h出てれば速いと思います
0020ご冗談でしょう?名無しさん垢版2020/05/08(金) 04:36:30.76ID:xWruT7W5
ニホンザルネトウヨゴキブリ日本軍テロリスト礼賛南京大虐殺テロリスト漫画家平野耕太豚焼き殺せ
0024ご冗談でしょう?名無しさん垢版2020/05/08(金) 12:12:51.40ID:???
>>18
物理は物理現象を分析し、物理量の基本要素を抜き出し、(力学)理論で解析する。
>球威(球の持つエネルギー)に負けて打ち返しにくい
体育会系の表現だから球威や(スポーツ選手の)エネルギーの定義が体感的なのが判る。
体感・経験からスポーツが上手くなるのは、熟練技術と同様で物理と方法が違うだけ

物理で球威を例えば衝突で押し返される現象だとすれば、物理のエネルギーではなく
運動量の変化、力積、重心の関係だと解るだろう。
後は高校教育レベルの力学を使えば自分でもモデル解析できるだろう。

>ラリーは初速60km/h、手元で40km/h
これも慣習的な単位で、おそらく車の速度計をそのまんま何にでも使ってる
車の速度計の単位 km/h が便利なのは、 平均40km/hで走れば1時間後に地図で40km先の
目的地に行けるのが計算せずに判るからだ。
テニス、卓球、野球では役に立たず相対的な比較の目安にしかならないのだが
今でも何故かスポーツ業界はm/sを使わない。
0027ご冗談でしょう?名無しさん垢版2020/05/08(金) 13:06:04.18ID:???
>>24
野球のフォースプレイの例ならば、塁間の長さ、特定走者の塁間時間、野手の投球速度
などを m,s,m/sの単位で以前から情報収集しておけば、打球を捕球した時点の位置情報から
ベテランでなくともフォースアウト可能かどうか判断でき、暴投の危険率も回避できる。
これがそれぞれの実用単位の便利性なのだ。
0028ご冗談でしょう?名無しさん垢版2020/05/08(金) 13:47:39.63ID:???
バットの重心に関してボールが及ぼす力のモーメントと手が及ぼす力のモーメントが釣りあって云々、ってはなしは
野球に関してはよく研究されているようなのでそういうのを参考にしたらいいと思う
バットの力学とかいう言葉でググればいい
0029ご冗談でしょう?名無しさん垢版2020/05/08(金) 13:52:13.40ID:???
>>24
言いたいことが分かりませんが、どう解析するべきか分からないので適当にパラメータを出した感じです

ともあれ曲がりくねって加速減速する車のスピードメーターをそんな指標にはしないのでは?
飛行機なら分かりますが・・・

m/sにしないのは単純にスピードガンの単位がkm/hで馴染みがあるだけだと思います
0032ご冗談でしょう?名無しさん垢版2020/05/08(金) 14:08:59.26ID:???
>>29
>適当にパラメータを出した感じです
それにケチつけてるのではない
>平均40km/hで走れば
が読めないのか、 平均で走る訓練をしたければラリー競技にでも参加すればよい。
当然だが、速度制限40km/hを順守して一般道路で平均40km/hで走るのは不可能だ
どうしても1時間で行きたい人は速度違反をする。
0034ご冗談でしょう?名無しさん垢版2020/05/08(金) 14:29:11.41ID:???
>>29
>m/sにしないのは単純にスピードガンの単位がkm/hで馴染みがあるだけだと思います
そこまで推測したなら、スピードガンの単位がkm/hになったのか推測したらどうだ

交通違反の速度検挙に使われてたとすれば、違反容疑者は車のkm/hの速度計との比較
でしか判断できない。
速度違反を通告して罰金を科すには、同じ実用単位の速度でなければ互いに不便だ。
0035ご冗談でしょう?名無しさん垢版2020/05/08(金) 14:36:54.61ID:???
>>33
車の速度計のメカニズムを知らないから”瞬時値”などと言ってるんだろが
速度計の表示速度は平均値だ。 ただし1秒よりは短いだけ。
つまり、km/hの様な大きなスケールにしたとき速度としての意味がある。
0038ご冗談でしょう?名無しさん垢版2020/05/08(金) 15:02:39.56ID:???
>>35
車の速度計の例えから、km/h が単なる表現形式の1つではなく
キロメートル、1時間の単位スケール以上で扱うのに便利な実用単位だろ判るだろう。
メートル、秒のスケールを扱うスポーツ種目でそのまんま慣習で使っても目安程度にしかならない。
0042ご冗談でしょう?名無しさん垢版2020/05/08(金) 15:26:45.54ID:???
秒速数十メートルじゃ迫力が足りないからスポーツじゃ使わないんだよ
スポーツって興行だよ? 選手でも監督でもないようなおじさんが実用だの何だの言ってシコシコ悦に入っているのはキモいな
0043ご冗談でしょう?名無しさん垢版2020/05/08(金) 15:34:00.67ID:???
>>33
>瞬時値
確定した瞬間の速度というのは、連続体近似した古典物理学の概念
ミクロ現象に拡大した量子論には無い、平均値(期待値)があるだけ

>>40
否定はしない、日本は超高齢化でアナログ・ボケー爺婆の慣習が蔓延ってるからな
イギリスやスウェーデンのようにウィルスによる自然淘汰に任せたほうがいいかも。
0048ご冗談でしょう?名無しさん垢版2020/05/08(金) 15:56:17.80ID:???
>>42
>スポーツって興行だよ
そうだな、160km とか分母の /h を無視すれば物凄いスピードだ。

聖闘士星矢に出てくる光速拳とか絶対零度拳とか物理無視だから大迫力がある
いまなら無料で見れる。
0054ご冗談でしょう?名無しさん垢版2020/05/08(金) 17:29:26.83ID:???
>>47
頭悪いな
統一されるのは平均の概念だ、古典物理学と量子論では計算方法が違うのが当然
同じく、確率の概念も同じだが両者では計算方法が全く違う。

>>50
頭悪いな、読解できんのか
投手が投げてからミットに収まるまでの時間は0.5秒でも見てる観客はその時間で
160kmの表示を無意識に関連付けるということだ。
0057ご冗談でしょう?名無しさん垢版2020/05/08(金) 18:23:05.31ID:???
>>54
ある老人がハイウェイを運転していた。
その時、妻が心配そうな声で彼の携帯に電話を掛けてきた。

「あなた、気をつけて。ルート280号線を逆走している狂った男がいる
ってラジオで言ってたわよ。」

老人は「ああ、知ってるわい。でも、一台だけじゃないんだよ。
何百台という車が逆走してるんだよ。困ったもんだ。」
0059ご冗談でしょう?名無しさん垢版2020/05/08(金) 18:52:30.25ID:???
変な本やサイト読んで一時的に頭ショートしてる中高生ならこんなもんでしょ
大の大人がこんなこと抜かしてたら義務教育の敗北だよ
0060ご冗談でしょう?名無しさん垢版2020/05/08(金) 19:07:15.33ID:???
>テニスで軽いラケットを使うと球威(球の持つエネルギー)に負けて打ち返しにくいというのは何となくわかるのですが
>同じ理屈で卓球でも重いラケットは球威に負けないと言う俗説があります
>個人的にはピン球の球威なんて知れているので軽いラケットを早く振った方が球威が出ると思うのですが

この質問が結局どういう問題かというと「ボールやラケットに、ボールやラケットの質量kg、ボールやラケットの速度m/s、以上の物理量が乗っることはあるかどうか」ということ

答えを言っておこう。不可能だ。

ボールについて語るが、
ボールの質量はボールの質量。ボールが飛ぶときボールの持っている運動量(kg)(m)/(s)の質量成分はボールの質量という定められた値。これが増加してしまうと無から質量が生み出されたことになる
速度も同じ。見た目の速度は見た目の速度。運動量(kg)(m)/(s)の速度成分はそのまま見た目の速度。増加する場合見た目の速度も変わる

だからボールが物体として質量が規定されて、見た目の運動として速度が規定される場合、それ以上それ以下の強力or弱い球というのは存在しない
ラケットも同じ。全ては質量の比と衝突時の速度

──なんてな。^1ならな

(kg^2)、(m/s)^2ならどうだと思う?
0061ご冗談でしょう?名無しさん垢版2020/05/08(金) 19:08:19.43ID:???
(m)/(s^2)=加速度。3s^2の入力で9m進む
(m^2)/(s)=減速度。9sの入力で3m^2進む
(m^2)/(s^2)=これは?3s^2の入力で3m^2進む=等速度。速度と相似。

(m/s) vs (m/s^2) = (^1) vs (^2)
乗数が上がるほど我が強い。乗数の低いものに打ち勝つ

(m^1/2)/(s)=加速度。3sの入力で9m^1/2進む
(m^1/2)/(s^1/2)=等速度

(kg) vs (kg^2) =??
(kg^1/2) vs (kg) =??

ボールの質量は変わらない。決まった値。しかしその質量の役が変わる。物体既定の質量kgの2つ分。質量が二重にかかる

さて、人間にこれが不可能だろうか?思ったより力の強い奴、重い拳、重いスマッシュ……
加速度m/s^2を作り出せるように他は作り出せないのか?

野球ボールに圧力をかける=(m)/(s^2)
シャトルを素早くコンパクトに投げる=(m^1/2)/(s)=端折り
ハンドボールをうまく投げる=(m^2)/(s)=ボール全体に速度を伝え、働かせる(ビーチボールなどは特に)

人を殴るときは?ものを叩くときは?足の小指をぶつけるときは?歩くときは?走るときは?跳ぶときは?
人間は人間の体のスペック以上の行動をしている
0062ご冗談でしょう?名無しさん垢版2020/05/08(金) 19:09:46.13ID:???
^2……これはあくまで物理量2個分。質量10kgの腕は10kg^2=100Pという決まった値でしか叩けない。

──^xが整数しか許されないならな
kg^1.012などの中途半端も存在する。これが思ったより〜、人間の動きの不安定さ。誤差。

「軽いラケットで打つと弱い」?「重いラケットなら強い」?そんなことはない。人間の体の動かし方。「重いラケットの方が力が加えやすい」だけ。ピンポン球の球威?「ピンポン球は軽すぎるし、中が中空なので空気に物理量を付与しにくい」だけ

プレイヤーの体重、腕力、ラケットの重さ、ボールの重さで「効率」は変わるが、「効果」は体の動きがもたらす「動力」

結局何が言いたいかというと、重いラケットを買え。
0066ご冗談でしょう?名無しさん垢版2020/05/08(金) 19:54:22.02ID:???
俺を理解してくれない奴は頭悪い理論ほんとすき
なんでこう長ったらしくて分かりにくい文章を書くんだか
日本語不自由すぎるだろ
0069ご冗談でしょう?名無しさん垢版2020/05/08(金) 22:10:32.61ID:???
よくあるバンド理論の説明
http://web.tuat.ac.jp/~katsuaki/B060516appendix.pdf

図6に対してk=π/aの自由電子が正電荷の周期的ポテンシャルによってエネルギーが大きくなるのはなぜか?
負電荷をもつ電子に対して正電荷を導入したのになぜ?
0072ご冗談でしょう?名無しさん垢版2020/05/09(土) 02:42:34.19ID:???
この動画とかない?

>量子多体傷跡状態は、熱平衡化を示さないという点で、従来の統計力学の常識に反しているうえ、
>中には「温水中の氷が解けたり凍ったりを繰り返す」ような異様な振る舞いをするものもあり、大きな注目を集めています。
https://www.s.u-tokyo.ac.jp/ja/press/2020/6864/
00732行ずつ文字数揃えた垢版2020/05/09(土) 04:41:10.28ID:???
>>71
ボール・ラケット・腕の質量比は「効率」
ボールの威力・ラケットの威力は「効果」
重いラケットの方がスイングが遅くなるが、これは「効率」
体の動かし方によりラケットスピードが維持できる「効果」
球威、ラケット威力を乗せたければ「効果」的に体を動かすこと
「効果」を発揮するためには「力の込めやすく使いやすいラケット」
重いラケットというのは適切ではなかった
だが重いラケットを振るために力を込める
動かしにくいほど動かすために物理量を込める
だから重いラケットの方が簡単に球威を出せる
一方器用な人間の場合軽い方が可変的に扱いやすい場合もある
どちらにしても体の動かし方「動力」を意識的に扱えるかどうか

つまるところ「本人の扱いやすい重さ」の上で「効率」を考えるべきということだな
体の動かし方を工夫せず筋力・瞬発力で打ちたいというなら軽いラケットでよい
0075ご冗談でしょう?名無しさん垢版2020/05/09(土) 09:31:03.24ID:???
>>70
>爺ちゃん ... 絶望的にわかってない
貶しで終わり、アホレスの見本
アホだから短いスレの内容が読解できず何も書けない。
0076ご冗談でしょう?名無しさん垢版2020/05/09(土) 09:40:16.41ID:???
>>73
誰が読んでも「スポーツ指導」だろ
>動かしにくいほど動かすために物理量を込める
誰が読んでも物理じゃないからテニス板でやれ。
0083ご冗談でしょう?名無しさん垢版2020/05/09(土) 12:26:55.82ID:???
>>82
ちがう
ギャップが開くとき、下は自由電子よりも安定化するが上は自由電子よりも不安定になるでしょ
引力ポテンシャルを入れているのにどうして不安定化という質問
0085ご冗談でしょう?名無しさん垢版2020/05/09(土) 12:49:27.77ID:???
エネルギーが大きくなる/小さくなるを不安定化/安定化と言っただけ
言葉が気に入らないならそう読み替えてくれ
0086ご冗談でしょう?名無しさん垢版2020/05/09(土) 12:58:17.37ID:???
>>83

外場によって縮退が解けたときのエネルギー固有値(のいくつか)が、縮退が解ける前のエネルギー固有値より大きくなるのはどうしてか、ということ?
0087ご冗談でしょう?名無しさん垢版2020/05/09(土) 13:13:52.19ID:???
>>86
ポテンシャルを入れることを外場と言うならそういうこと
引力ポテンシャルだから安定化する以外あり得ないと思うが
0088ご冗談でしょう?名無しさん垢版2020/05/09(土) 13:21:15.24ID:???
固有値問題考えてるから別にエネルギー増えても問題ない気がするんですけど

調和振動子考えたとしても、ゼロ点エネルギーが最低になって、自由粒子のエネルギーの最低値0が最低じゃなくなりますよね
引力ポテンシャルなのに最低エネルギー増えてますよ
これも気持ち悪いんですか?
0089ご冗談でしょう?名無しさん垢版2020/05/09(土) 13:25:58.19ID:???
引力ポテンシャルという言葉が曖昧だったかもしれない
“常にマイナスの値をとるポテンシャル関数”を引力ポテンシャルと書いていた
図8みたいなポテンシャルのことだ
0091ご冗談でしょう?名無しさん垢版2020/05/09(土) 13:31:49.45ID:???
ポテンシャルの符号なんて関係ないですよね
エネルギーの原点の位置変えれば、正にできますよ
大事なのはポテンシャルの形ですよね
0093ご冗談でしょう?名無しさん垢版2020/05/09(土) 13:35:50.04ID:???
V=-1/rの形のポテンシャルは、下駄を履かせてV=-1/r+E0としても何処にも何も影響しないわけで、
“常にマイナスの値をとるポテンシャル関数”というのは何だろう?
0096ご冗談でしょう?名無しさん垢版2020/05/09(土) 13:42:32.66ID:???
それをしてなんの意味があるの?

“常にマイナスの値をとるポテンシャル関数”の定義が危ういのはわかったから本題に戻らせて
0098ご冗談でしょう?名無しさん垢版2020/05/09(土) 13:46:31.92ID:???
たとえ下駄を履かせたとしても自由電子のポテンシャルよりも小さくなっているのにエネルギーが上昇する理由がわからない
0100ご冗談でしょう?名無しさん垢版2020/05/09(土) 13:49:20.03ID:dQU+a0Qn
結晶でなくても例えば二原子系で結合軌道と反結合軌道が出てくるのが分からんというなら疑問としては一貫している
0102ご冗談でしょう?名無しさん垢版2020/05/09(土) 13:58:01.65ID:dQU+a0Qn
誰か分からんからID出せ
0103ご冗談でしょう?名無しさん垢版2020/05/09(土) 13:59:43.31ID:???
>>98
エネルギーは本当に上昇してるの?
同じ波数kでも、より高いエネルギーを持つ電子があるわけで、そういった電子がエネルギーを失って
一つ下の、エネルギーが上昇して見える所のエネルギー準位に落ち込んでいるんじゃないの?

図の6は、「この波数のときはこのエネルギーを持つ」という波数とエネルギーの関係を表すグラフであって
特定の電子の状態がこう変化するというグラフではないわけだし。
0105◆dBYSNQKzkkgD 垢版2020/05/09(土) 14:04:11.85ID:???
>>103
それは流石に苦しい話です
無限小のポテンシャルでも同じ話なので
0107◆dBYSNQKzkkgD 垢版2020/05/09(土) 14:08:50.37ID:???
69,81,83,85,87,89,90,92,94,96,98,99,101
はぼくです
安価が打ち込めないのでこれで勘弁してください
0108◆dBYSNQKzkkgD 垢版2020/05/09(土) 14:10:45.36ID:???
>>106
無限にゼロに近いけどゼロではないこと
さすがにほぼ自由な電子知ってたら>>103はおかしいってわかるでしょ
0109ご冗談でしょう?名無しさん垢版2020/05/09(土) 14:20:04.12ID:???
>>108
「ほぼ自由な電子」は知らない。
おかしいというなら引っ込めるけど、問題となる電子が何かからエネルギーを受け取っているのは確定なのか?
0110◆dBYSNQKzkkgD 垢版2020/05/09(土) 14:24:22.76ID:???
>>109
たとえば図8ならk由来のバンドがφ-で、k+G由来のバンドがφ+を構成するのは間違い無いと思う
きちんと説明するのはまだむずかしい…
すまん
0111ご冗談でしょう?名無しさん垢版2020/05/09(土) 14:25:19.42ID:dQU+a0Qn
NFE近似も知らないド素人が引っ掻き回すと無駄に荒れるだけだからやめろ

結局周期ポテンシャルVの一次摂動で固有エネルギーがk^2/2m±V (k=π)に分裂する直観的理由が分からんって質問か?
0112◆dBYSNQKzkkgD 垢版2020/05/09(土) 14:28:52.32ID:???
>>111
図8の説明にある分裂する理由はわかるけど、分裂した先で自由電子よりも高いエネルギーをもつ理由がわからない
0113ご冗談でしょう?名無しさん垢版2020/05/09(土) 14:33:06.18ID:dQU+a0Qn
それは数学的にか直観的にか
0114◆dBYSNQKzkkgD 垢版2020/05/09(土) 14:36:01.44ID:???
よく分からないからどっちも聞いてみたい
0115ご冗談でしょう?名無しさん垢版2020/05/09(土) 14:42:25.64ID:dQU+a0Qn
式(12)までの数学的導出も理解できないってことか?
0116◆dBYSNQKzkkgD 垢版2020/05/09(土) 14:46:13.10ID:???
そこはわかると思う
すまん、じゃあ直感的のほうを聞きたい
0117ご冗談でしょう?名無しさん垢版2020/05/09(土) 15:07:48.81ID:???
頭が悪い奴ばっかだな 直感もわかんのか
シュレディンガー方程式で
自由空間中の電子と、結晶中の >ほぼ自由な電子 はポテンシャル付きの解が周期関数でも別物。
そのポテンシャルが非常に小さい極限をとっただけだ。

結晶物質中の電子であるにもかかわらず結晶原子で散乱されずに自由に動け回れるという意味で、抵抗が非常に小さい導体物質が実在する理由と、同時にエネルギー順位の
バンド構造が生じることを証明したことになる。 量子力学の威力は凄いね。
0118ご冗談でしょう?名無しさん垢版2020/05/09(土) 15:12:11.45ID:dQU+a0Qn
波数 π の一次摂動を入れたら陽イオンの位置で腹を持つ状態と節を持つ状態が現れて
引力相互作用だから後者のエネルギーが高くなるというだけの話だが
どこに納得ができないんだ?
無摂動状態(空格子近似の自由電子)なら空間的に一様な存在確率を持つ状態だというのが理解できてないのか?
0119ご冗談でしょう?名無しさん垢版2020/05/09(土) 15:14:39.02ID:???
君ら頭硬すぎ。

|ψ(x)|^2が原子核の位置で極小≒クーロン引力を受けていない≒ポテンシャル導入前とエネルギーが同じ
と考えるんだよ。

k=π/aでEが+Vと-Vに分裂するんじゃない。k=π/aでEが+0と-2Vに分裂するんだ。
k=0でE=0だった状態がE=-Vになってるんだよ。
0122◆dBYSNQKzkkgD 垢版2020/05/09(土) 15:20:00.77ID:???
>>118
陽イオンの位置で節をもつとしても自由電子のエネルギーと比べて大きくなるのはおかしいんじゃないのって話
その二つでエネルギー差が生まれるのはわかる

>>119
|ψ(x)|^2が原子核の位置で極小≒クーロン引力を受けていない

がわからない
クーロン引力が働いてなかったらただの自由電子じゃん
電子分布も一様だよ
0124ご冗談でしょう?名無しさん垢版2020/05/09(土) 15:20:24.48ID:dQU+a0Qn
>>120
一様な存在確率を持つ状態と比べてイオン近傍の存在確率が減少すれば相対的にエネルギーが上昇するというのは当然だと思うが
どこに違和感を覚えるんだ?
0125ご冗談でしょう?名無しさん垢版2020/05/09(土) 15:20:55.51ID:dQU+a0Qn
>>122
>陽イオンの位置で節をもつとしても自由電子のエネルギーと比べて大きくなるのはおかしいんじゃないのって話
その根拠は?
0127ご冗談でしょう?名無しさん垢版2020/05/09(土) 15:22:32.34ID:???
無限遠静止電子をエネルギーの基準に取ると
真空中を運動する自由電子はプラスのエネルギー
原子核に束縛された電子はマイナスのエネルギー
その区別をもう一度考え直せ
0129ご冗談でしょう?名無しさん垢版2020/05/09(土) 15:24:19.07ID:dQU+a0Qn
>>126
比較すべき「自由電子」は空格子近似のそれであって
その力学的エネルギーは(無限遠をポテンシャルの原点として)負になっているということを理解してるか?
0130◆dBYSNQKzkkgD 垢版2020/05/09(土) 15:26:38.75ID:???
>>129
ポテンシャル関数が自由電子のそれよりも常に小さな値だから

に直せば伝わる?
0131ご冗談でしょう?名無しさん垢版2020/05/09(土) 15:27:11.98ID:???
クーロン引力のような中心力を受けながら運動する場合はビリアル定理によって運動エネルギーの2倍のポテンシャルエネルギーの安定化を受けているぞ
0132ご冗談でしょう?名無しさん垢版2020/05/09(土) 15:28:42.57ID:dQU+a0Qn
>>130
その「自由電子」は真空中の電子だろ
バンド理論でいう自由電子とは別物
0135◆dBYSNQKzkkgD 垢版2020/05/09(土) 15:34:10.52ID:???
>>132
ポテンシャル関数ってのは結晶の周期ポテンシャルのことだけど話あってる?

空格子近似の力学的エネルギーが負になるってのは正直よく分からん
式6のことなら明らかに正だと思うが
0136ご冗談でしょう?名無しさん垢版2020/05/09(土) 15:37:19.44ID:dQU+a0Qn
>>135
あっている

式(6)は原点を取り直している
式(12)の導出は波数 π 以外の成分を無視している
0137ご冗談でしょう?名無しさん垢版2020/05/09(土) 15:37:19.55ID:???
「運動エネルギー10eVの自由電子」と同じ運動エネルギーを持つ結晶中の電子は,
運動エネルギー10eVに加えてポテンシャルエネルギーを-20eV受けている(無限遠基準で-10eV)
これが±5eV分裂して-5eVと-15eVになっても「運動エネルギー10eVの自由電子」からすればどっちも安定化しているだろ?
0139◆dBYSNQKzkkgD 垢版2020/05/09(土) 15:40:52.51ID:???
>>136
真空のときにE=h^2k^2/2m
空格子近似でE=h^2(k+G)^2/2m

比べたら後者の方が大きい気が
どっか勘違いしてる気がするけど、どこかは分からない
0141ご冗談でしょう?名無しさん垢版2020/05/09(土) 15:45:35.56ID:dQU+a0Qn
>>139
それらは真空中の自由電子のエネルギーよりも周期ポテンシャルの波数 0 成分 V_0 の分だけ低い
0144◆dBYSNQKzkkgD 垢版2020/05/09(土) 15:50:50.48ID:???
>>140
図6のbとcでは原点がずれているってことか?

>>141
空格子近似ってポテンシャルゼロのことだと思ったけど波数0のエネルギーはいるの?
0146ご冗談でしょう?名無しさん垢版2020/05/09(土) 15:55:12.54ID:dQU+a0Qn
>>144
何度も言うように
原点を取り直している
0148ご冗談でしょう?名無しさん垢版2020/05/09(土) 15:57:52.14ID:dQU+a0Qn
>>119は陽イオンから受けるポテンシャルの波数 0 成分と π 成分の区別が出来ていないから論外
0151◆dBYSNQKzkkgD 垢版2020/05/09(土) 16:03:55.15ID:???
>>146
何度もすまん
原点を無限遠にしたときに真空中の電子のエネルギーはh^2k^2/2mってのはあってるよね?
>>141だとこっちにも-V0するように見えたけど
0152ご冗談でしょう?名無しさん垢版2020/05/09(土) 16:06:50.02ID:dQU+a0Qn
>>151
あっている
固体中に束縛された自由電子(>>145の言うようにジェリウム模型)は -V_0 だけ真空中の電子よりエネルギーが低い
0153◆dBYSNQKzkkgD 垢版2020/05/09(土) 16:10:52.77ID:???
>>152
まぁわかった
図6のaとbではV0だけ原点がずれているという話なんだな
0155ご冗談でしょう?名無しさん垢版2020/05/09(土) 16:12:53.87ID:???
結論
均質なポテンシャル(ジュリウム)を基準にして、周期的なポテンシャルを考えると相対的なエネルギーが分裂するというお話でした
0156ご冗談でしょう?名無しさん垢版2020/05/09(土) 16:13:46.80ID:???
まだやってんのか
結晶中の電子は周りの格子のポテンシャルで余分なエネルギーが有り
つまり、真空中の電子と異なる有効質量を持っている。
0159◆dBYSNQKzkkgD 垢版2020/05/09(土) 16:17:40.43ID:???
空格子近似では周期的なポテンシャルは入れないが、一様なポテンシャルが入っていて(陽イオンを一様に分布させることに対応、ジュリウムモデル)V0だけ安定化する
って認識でいいか?
0161ご冗談でしょう?名無しさん垢版2020/05/09(土) 16:19:22.32ID:dQU+a0Qn
>>159
よい
0164◆dBYSNQKzkkgD 垢版2020/05/09(土) 16:24:59.47ID:???
>>161
さんくす
空格子近似の理解が間違っていたんだな
スレ消費してしまったすまん
0174ご冗談でしょう?名無しさん垢版2020/05/09(土) 20:35:25.97ID:B3+oodHg
ゲージ変換=位相変換がわからない。直感的なイメージってないのかな?
0176ご冗談でしょう?名無しさん垢版2020/05/09(土) 22:56:57.08ID:???
アーノルドにラグランジュ方程式の導出は変分を一次まで取ればいい理由が証明つきで載ってました

それはそれとして一般相対性理論の破綻が予想されている超高エネルギー領域でそもそも解析力学の方法が使えるのかとか(多様体に基づくから使えそうではあるけど)
対称性はあるのかとか、変分2次3次が効いてきてネーターの定理が破綻しないのかとか考えてるんだけど

なんかわかりやすい文献やサマリーない?
弦理論はこの辺に答えてくれそうもない
0177ご冗談でしょう?名無しさん垢版2020/05/09(土) 23:00:37.59ID:???
変分二次とって初めて効いてくる項があるというのは、時間の二階微分以降がラグランジアンに含まれるかどうかって話になるんじゃないですかね


qとq•の他にq••とかq•••とか含まれるかどうかって話なら結構探しやすそうな気がします
0178ご冗談でしょう?名無しさん垢版2020/05/09(土) 23:06:25.27ID:???
>>177
専門が非線形光学だったので単純に超高エネルギー領域ではラグランジュ方程式の導出においても2次3次が無視できない大きさの量になるのではおと思っています

ただ、根本的に方程式が変わってしまうため違和感が根強く、なんかいまいちはっきりした考えが持てないんですよね。
多様体に埋め込んで議論するとしても多様体自体を考え直す必要に迫られるし明らかに今と全然違う物理になっちゃう
0179ご冗談でしょう?名無しさん垢版2020/05/09(土) 23:07:39.93ID:???
q••含む場合のオイラーラグランジュの話はありますよ
解析力学の本の1番後ろとかに乗ってたりします
0180ご冗談でしょう?名無しさん垢版2020/05/09(土) 23:11:56.70ID:???
>>179
あ、ねこ本に載ってた気がしてきた。
でも知りたいのは解析力学を議論するベースとなる多様体がどう変わるかだからちょっと違うかな

それを元に一般相対性理論まで行くと時空の破綻とか対称性の破綻とかが見いだせるんじゃないかと思うんだけど、感覚だけで書いてるから計算しないと
0181ご冗談でしょう?名無しさん垢版2020/05/09(土) 23:24:06.59ID:???
解析力学の多様体的な扱い詳しくないですけど、あれって速度を接空間の元とみなすってだけの話なんじゃないですか?
加速度考えるときは接空間のコピーでも考えれば良いのではないでしょうか

数学的な定式化が難しくなるかもしれないですけど、本質はあまり変わらない気がします
0182ご冗談でしょう?名無しさん垢版2020/05/09(土) 23:27:10.00ID:???
>>181
あ、言われてみればそうかも

議論に乗せるべき多様体自体が変わってしまうと意識してたけどそんなこともないかもしれん
0185ご冗談でしょう?名無しさん垢版2020/05/10(日) 17:34:46.37ID:RC0fRR/l
単振動の質点においてこの質点の位置の確率と速度の確率が大きいのは粒子がどのあたりにいるときか、と問われました。
位置の確率はわかるのですが速度の確率ってどういう意味ですかね?
0187ご冗談でしょう?名無しさん垢版2020/05/10(日) 17:39:44.13ID:???
>>185
速度を求める測定をしたときに速度がvである確率P_vを求めよってことじゃない?
位置の確率なら波動関数の二乗がそれに相当するよね
0188ご冗談でしょう?名無しさん垢版2020/05/10(日) 17:43:30.77ID:RC0fRR/l
>>187
単振動ならその確率は常に一定ではない?
0191ご冗談でしょう?名無しさん垢版2020/05/10(日) 18:17:46.10ID:RC0fRR/l
>>189
どうして?
0192ご冗談でしょう?名無しさん垢版2020/05/10(日) 19:48:57.21ID:???
>>185
意味わからんからdxdp=hから速度の確率のようなものを出して提出して
「この質問意味わかんなかったんですけど」
って聞くかな俺なら
0193ご冗談でしょう?名無しさん垢版2020/05/10(日) 20:00:14.42ID:???
古典論の場合、ある位置で観測される確率は P(x) dx 〜 1/|v| ・dx だろうけど
ある速度で観測される確率 P(v) dv はどうなるの?
0195ご冗談でしょう?名無しさん垢版2020/05/10(日) 20:44:00.39ID:efvKVGge
いまだに確率が存在すると思っているアホザルどもには
心底笑わされるわ。
お前ら本当にアタマ悪くておもろいよな。

いっぺんググってみろよ。
まともな定義は一切存在してないから。

確率とは
・定義不可
・検証不可
・作成不可
の3拍子そろった人類の妄想である。

まずはその確率とやらの
定義から言ってみろや。

とたんに不可だと気づくわな。
普通の知能があればだがな。
アホザルどもが。

二重スリット実験で
都合の悪い不細工な干渉縞がいっこうに発表されんのは
いったいどういうことなんだ。

確率なんだろ、ああ?
だったら不細工な模様も観測されるはずだぞ、確率事象だったらな。
つまり「二重スリット実験は確率事象ではなく、恣意的事象である」と
証明してるようなもんだわ、アホザルどもが。

きれいな干渉縞が出るまで恣意的に測定系をいじりまくっているのである。
当たり前だがその時点で確率事象ではないのだ。やらせである。
アホザルにはこれがいっこうに理解できないのだ。

くっくっく
0196ご冗談でしょう?名無しさん垢版2020/05/10(日) 20:49:16.58ID:efvKVGge
いやあ、まともな人間なら
サイコロの確率でつまづくわな。

サイコロの出目の確率は「1/6」であるとされておる。
ところがこの1/6とはいったい何のか、まるっきり定義不可なのだ。
ググってもまともな定義がまったく存在せん。

よくもまあ、確率などという妄想に
騙されてきたもんだ。
そんなもん、この宇宙のどこにも存在せんわな。

くっくっく
0199ご冗談でしょう?名無しさん垢版2020/05/10(日) 21:57:17.19ID:???
確率論では確率を「定義」してから議論を進めます。
サイコロの目のでる確率は1/6であることは「仮定」ないし「前提」です。
頭の悪い人はそこがどうしても理解できないのです。
0200ご冗談でしょう?名無しさん垢版2020/05/10(日) 23:03:10.03ID:???
>>174
数学的な定義ががあって
それを現実でも適用できるとモデルとして使ってるだけで
現実では定義できないとか意味わからないことを言う余地はない
0201200垢版2020/05/10(日) 23:04:14.29ID:???
前安価で返事しようとしたのが残ってた
>>174ごめんね
0202ご冗談でしょう?名無しさん垢版2020/05/10(日) 23:53:44.65ID:???
統合型リゾート法ではカジノが一部解禁されるが
日本では1000年前から現代まで金銭賭博を国家的に禁止されて地下にもぐるしかなく
宝くじ保険業界は確率データ一を隠して公表しないため、ほとんどの一般人が確率論を知らない。
カジノやサイコロ賭博では、出目の人為的操作が一切禁止され約1/6でなければイカサマになり殺人事件も起きる。
保険業界、カジノは偶然現象を信じ、確率論を信頼する人が行う事業といえる。

パチンコは外観は確率的に見えるが店も客も人為的操作が可能でそのためか
法律上も賭博とは見なされずコロナ蔓延でも堂々と営業できる。
日本人のおかしな論理でした。
0207ご冗談でしょう?名無しさん垢版2020/05/11(月) 01:09:26.44ID:???
シュレーディンガーの猫、
猫ではあんまりなので、音信不通の知人や最近見かけなくなったタレントでもいいのでしょうか
0211ご冗談でしょう?名無しさん垢版2020/05/11(月) 15:12:54.79ID:+zMZPnYK
>>210
sinカーブとは限らんぜ cosの場合だってある
0214ご冗談でしょう?名無しさん垢版2020/05/11(月) 18:11:11.92ID:???
重要 【 丸 投 げ 禁 止 】

4. 宿題を聞くときは、どこまでやってみてどこが分からないのかを書く
0220ご冗談でしょう?名無しさん垢版2020/05/11(月) 19:24:53.66ID:???
>>213
これ本当は2ページ目に書いてるみたいに何がなんの変数になってるのか考えて計算しないといけないのに、微小量だからーとか言って勝手に割り算したりなんなりしてるからわけわからないことになるだけだと思うんですけど
0229227垢版2020/05/11(月) 20:58:56.92ID:???
>>226
君の計算の悪いとこもいった方がいいか
複素部分というか位相部分ともいうかもしれんがそいつの絶対値は1なので
下から2式めからtanδ自体はだせるけど
そいつは絶対値1である必要があるので分子にルートの部分をくくりださないといけない
結局割られて分母にルートがつく
0231ご冗談でしょう?名無しさん垢版2020/05/11(月) 21:58:17.82ID:Sja7oIDY
自分はこう考えました
zはdx変化してからdy変化するのではなく
dx,dy同時に変化するのだと
z=φ(x,y)として
φ(x,y) → φ(x+dx,y+dy)
続きは添付画像で
http://imgur.com/ioBwzfu.jpg
0233ご冗談でしょう?名無しさん垢版2020/05/11(月) 22:04:47.24ID:???
>>231
いや、そこら辺の議論に疑問を持ってる人はいないのではないでしょうか?

最後の式がまずあって、そこから例えば両辺dxで破ったらどうなるのーって感じかと思います
0241ご冗談でしょう?名無しさん垢版2020/05/12(火) 14:33:33.06ID:???
この証明じゃ一変数関数の単なる微分と偏微分を混同してるみたいで気持ち悪いという意見が出ました!
だからそんなことが起こらないように別の方法で証明を考えたみました!

ということを言ってるだけのpdfじゃないの?
0242ご冗談でしょう?名無しさん垢版2020/05/12(火) 14:56:39.04ID:86RkkZdJ
質問です

この動画、フェイクですよね?
https://youtu.be/5_Pbb1Ywo18

実質的にはカップを上下に2つ重ねて二本管を挿したの同じですよね
すると噴水側の管が上のカップの水面の高さまで水が来る事はあっても、噴水にはなりませんよね
合っているでしょうか?
お願い致します、エロい人
0247ご冗談でしょう?名無しさん垢版2020/05/12(火) 20:28:20.32ID:???
>>244
四角形を書きます

右は正方形、左は長方形、2つで大きい四角形を作ります

4m/s東に歩いて北風…正方形+長方形横の長さ4m

1m/s東に歩いて北西の風…長方形の左端から右端に歩いて正方形に到達。正方形の対角斜め右下45度矢印、これが北西の風。長方形の横は1m

正方形の縦の長さ3m

問題の風=大きい四角形左上から右下矢印。つまり北西よりやや西寄りの風。矢印、大きい四角形の対角を求める

縦3m 横4m 対角は

風の向きの平行四辺形
0252ご冗談でしょう?名無しさん垢版2020/05/12(火) 23:29:10.49ID:???
>>242
フェイクとは?

サイフォンの応用だと思うけど
最初はBのボトルの水がCに落ちてて
その過程でAの噴水を通ってる

次はBとCに差がないけどAの水位とAの出口に位置エネルギーがあって
その水がボトルに落ちてると思う
でも完全密閉だと流れないはずだから空気漏らしてるのかな
0255ご冗談でしょう?名無しさん垢版2020/05/13(水) 00:41:58.21ID:apx0Ky14
>>244 解答の図をみると、風の向きが北東に書いてある

北東の成分なんて出てくるかな
0258ご冗談でしょう?名無しさん垢版2020/05/13(水) 07:01:19.84ID:???
>>252
永久機関っぽい機構で液体に色がついてる以上、ポンプが入ってるとか疑うんじゃないかな
まあ、ボトルに温度差をつけてるとかしたら水ぐらい噴出しそうではあるけど、よくわからん
0259ご冗談でしょう?名無しさん垢版2020/05/13(水) 07:57:16.66ID:zcIwK4FJ
ああ、永久機関に見えるってことか
Bの水が無くなったら終わりだよ
0261ご冗談でしょう?名無しさん垢版2020/05/13(水) 11:36:26.97ID:???
>>260
やっぱり勝手に書き換えてるとしか思えませんね

y=xとしましょう
z(x,y(x))=x+y(x)=x+x=2x

dz/dx=2

∂z/∂x=1

こうなるべきだと思うのですが、あなたはどう思いますか
0263ご冗談でしょう?名無しさん垢版2020/05/13(水) 12:17:47.33ID:???
つまり、yが何かの関数になったら、dを∂に置き換えることはできないということですね


元々の熱力学の話でも、もろに、独立変数だったものがなにかしらの関数に置き換わってたりするんですけど、割れないということですね
0264ご冗談でしょう?名無しさん垢版2020/05/13(水) 12:34:21.34ID:???
>>213
みたいな,物理の教員が書いたものより
数学の本,読んだ方がいいんじゃないか
数学の本では,dxで割るなんてこと書いてある本はないぞ
0266ご冗談でしょう?名無しさん垢版2020/05/13(水) 12:39:18.47ID:???
それなりの計算をしないといけないのに、dを∂に置き換えるとか誤魔化そうとしてるのがあなたなんじゃないんですか?
0267ご冗談でしょう?名無しさん垢版2020/05/13(水) 12:41:39.49ID:???
無条件で置き換えるなんてことは誰も言ってません
ほかの変数を固定すれば偏微分になるだけです
勝手にy=xとか後付けしても何も矛盾は生じません
0272ご冗談でしょう?名無しさん垢版2020/05/13(水) 13:10:36.45ID:???
>>271
それは私が求めたやつですよ?

あなたはそれなりの計算をすると求められると言ってましてね

早くその計算を見せて欲しいんですけど
0275ご冗談でしょう?名無しさん垢版2020/05/13(水) 15:23:29.36ID:???
JJサクライの量子力学の本を定価で買いたいんだけど、通販じゃ無理?
大学生協も図書館も閉まってるから入手手段が通販しかないのに、ぼったくり価格では買いたくない
0280ご冗談でしょう?名無しさん垢版2020/05/13(水) 17:52:07.99ID:???
>>277
dyはどこに行ってしまったのですか?

てかもう面倒なんで答え言いますけど、

dz/dx=dx/dx+dy/dx
これが答えですよね

yが独立変数とみなせるのなら、dz/dx=∂z/∂x

yがxの関数になってるのなら、dz/dx=∂z/∂x+∂z/∂ydy/dx

こうなるわけですよ

つまり、割ってもいいですけど、その際はなにが独立変数であるかを全て明確にしないといけないということです
で、dxは偏微分ではなく、全微分として考えなければならない

なんでわざわざdxと書かれてるかわかっていないのですよあなたは
0281ご冗談でしょう?名無しさん垢版2020/05/13(水) 17:55:34.17ID:???
>>276
amazonで見てみると
出品 amazonの 6050円が定価なんだろうけど「一時的に在庫切れ; 入荷時期は未定です。」
他のは "新品"が12286円とか12588円になってる
他のサイトは見てないけど >>275が言ってるのは他も同じようなことになってるってことじゃないかな
0296ご冗談でしょう?名無しさん垢版2020/05/14(木) 01:16:48.13ID:???
電子の移動の相互作用が力を発生する理由。フレミング的なこととか理論とか数式とかじゃなくて、実際どういう力学現象が起こってるの?電子がなにしてるの?
0299ご冗談でしょう?名無しさん垢版2020/05/14(木) 01:21:17.45ID:???
電子が磁場中を動くと力を受けるという現象は、一番難しく言えば量子電磁気学というもので記述されます
0300ご冗談でしょう?名無しさん垢版2020/05/14(木) 01:40:29.63ID:???
じゃなくて、微粒子の力学的仕組み。カラクリ機構。つまり人間がミクロの背丈だったら何を目にするのか
0304ご冗談でしょう?名無しさん垢版2020/05/14(木) 03:01:10.84ID:???
ヒッグス機構=質量だけを得るのでヒッグス粒子の司るものは質量だけ
エレクトロン機構=力とか得るので光子が司るものは質量だけじゃない
0305ご冗談でしょう?名無しさん垢版2020/05/14(木) 03:07:21.73ID:???
というか光子はデフォで光速度の量をもつ
逆に、光子は質量を持たないのに電気が発生する力には質量がある、これいかに
0306ご冗談でしょう?名無しさん垢版2020/05/14(木) 07:52:31.06ID:jVgJ7HT1
>>298
dF=Ids×Bで
磁場は定義されるんだから、
「電子に力が働けば磁場がある」という解釈が
論理的に正しいんだよ。

dFを電子1個に直したものがローレンツ力で
df=ev×Bだ。

お前らは常に論理の逆を行ってるのは
サルだからだ。

くっくっく
0307ご冗談でしょう?名無しさん垢版2020/05/14(木) 07:54:09.72ID:jVgJ7HT1
訂正だ。

× df=ev×Bだ。
〇 f=ev×Bだ。

くっくっく
0309ご冗談でしょう?名無しさん垢版2020/05/14(木) 08:07:06.38ID:jVgJ7HT1
[問題]
磁場の定義式dF=Ids×Bからローレンツ力を求めよ。

(答え)
dF=Ids×B
 =(idS)ds×B
 =(eNv)dSds×B
 =ev(NdSds)×B

上式で(NdSds)は電流素片中の電荷の総数なので
1個当たりに働く力はこれで割って

f=ev×Bとなる。
これがいわゆるローレンツ力であり、
磁場の定義式を電荷1個当たりに変形したものにすぎない。

アホザルどもには
これがいつまで経っても分からないのだな。

くっくっく
0310ご冗談でしょう?名無しさん垢版2020/05/14(木) 08:14:34.28ID:jVgJ7HT1
ちなみに
アホノシュタインのバカもこれが分かっていない。
ローレンツ力が本質だと思っていたから
相対論というトンデモ論を平然と垂れ流しできたワケだ。

くっくっく
0313ご冗談でしょう?名無しさん垢版2020/05/14(木) 14:49:34.59ID:???
>>310
くっくっくは相間だけでなく物理現象の(マクロ)因果律を無視してオレ様解釈する。
f = m*a の因果律は力が原因で加速度が応答 f -> m*a が正しく、f <- m*a は間違い。
加速度から力を求める物理問題は、応答から原因を計算すると解釈される。
電源電圧Vの回路が閉じられる(ON)が原因でRIがその応答になる。
>>298
>電子は磁場の中を動くと力を受ける
は因果律的にも正しい。 具体的には
慣性運動の電子が磁束Bの領域に入る -> 電子に垂直方向のローレンツ力が発生する
-> 電子は垂直方向に加速度運動する。 という因果律に従う。
0321ご冗談でしょう?名無しさん垢版2020/05/14(木) 15:55:52.94ID:???
922 名前:ご冗談でしょう?名無しさん[sage] 投稿日:2020/05/05(火) 11:14:53.53 ID:???
自発的対称性の破れは、
完璧な対称性があったのにそれが破れたのではなく、
対称性の不完全さゆえに破れただけなんだよ。
もともと完璧な対称性などあり得ないというのがこの理論の核なんだよ。


925 名前:ご冗談でしょう?名無しさん[sage] 投稿日:2020/05/05(火) 12:01:26.76 ID:???
>>922
の様なオレ様解釈する奴が5chに多い。

南部博士の一般人向け説明によれば
「(完全な)対称性が自発的に失われたのと、始めから無かったのとは同等ではない。
 (完全な)対称性が自発的に破れた物理的ななごりが存在する。」


931 名前:ご冗談でしょう?名無しさん[sage] 投稿日:2020/05/05(火) 14:37:45.72 ID:???
>>925
いやいや南部が間違えてるだけ
完全な対称性など理論上のものであって
現実にあることは実証されていない。
0324ご冗談でしょう?名無しさん垢版2020/05/14(木) 16:30:54.88ID:jVgJ7HT1
面倒だから概要だけ書いといてやる。
お前らは基本を知らなさすぎだ。しょーもない教科書にしか接していないからな。
自力での勉強がまったく足らんのよ。だからサルなのだ。

歴史的には磁荷があるとして論理展開されてきたが
今はこういうふうに論理展開するのが正しい。ただし、その辺の教科書は
EーH対応とEーB対応を混在させて書いてるのでメチャクチャだから
まったく話にならん。

・磁場の定義式 dF=Ids×B、つまり電流に力が働けば磁場があるとする。
・2本の直線電流に働く力は実験により F=k・I1・I2/r
・このことから電流自身が磁場を作るというビオサバールの法則dB=μ0I/4Π・ds×r/r^3が得られる。

・この法則を変形するとrotB=μ0i、またはB=rotAとなる。
・B=μ0HとするとrotH=i、つまり真電流だけが作る場をHとする。
・強磁性体があれば電流に働く力は強められるのでB=μ0H+Jとして磁場を加算する。
・rotB=μ0rotH+rotJ=μ0i+rotJ=μ0i+μ0im(磁化電流の定義)
・特にJ=χHならばB=μ0H+χH=(μ0+χ)H=μH、よってrotB=μrotH=μi
・そうするとrotB=(μ0+χ)i、これがμ0i+μ0imに等しいので
 χi=μ0imとなり磁化電流は真電流に比例する。その関係は強磁性体なら一般的にi<<im

これをさらっと理解していないとまともな論理展開などまったく不可能だ。
お前らは土台がまるっきり存在していないアホザルなんだよ。

くっくっく
0325ご冗談でしょう?名無しさん垢版2020/05/14(木) 16:38:22.89ID:jVgJ7HT1
>>311
電流と電流の間には力が働く。
磁石には磁化電流が流れていると解釈する。
これが土台だ。

だから空中の単独電荷の流れは電流ではないので
相互作用としてはクーロン力しか働かんのだ。

単独電荷同士の間ではローレンツ力は存在しない。
これに気づかなかった20世紀の物理屋どもは
マヌケしかおらんわ。

気づいておれば相対論など容認されなかったのにな。
実にアホザルしかおらんかった。哀れよのうー

くっくっく
0326ご冗談でしょう?名無しさん垢版2020/05/14(木) 16:42:57.69ID:???
>>324
そんなめちゃくちゃ古い物理学の話を何度やったって無駄

現代的には解析力学を用いた定式化でやりなさい
こっちのがスマートで
古典力学電磁気学や相対論まで同じように定式化できる
電磁気学をつくるラグランジアンにおいて
物理変数として電磁場があって
もちろんそっからオイラーラグランジュで
荷電粒子との対応がわかって
それを測定する方法もそっからわかると
0327ご冗談でしょう?名無しさん垢版2020/05/14(木) 16:44:28.68ID:jVgJ7HT1
電流があれば、空中にある単独電荷の流れでも
その電流との相互作用でローレンツ力が働くのだ。
磁石に対しても同じだ。

ローレンツ力も電磁誘導も変位電流も物質が必要なのである。
空中に単独で存在する電荷の流れだけでは発生しないのだ。

こんな簡単な、そして素直な考察が
誰一人として出来なかった20世紀の物理屋どものアタマの悪さには
本当に驚嘆するわ。

何を食ったら相対論みたいな一発で虚構だと分かるうんこ論を
鵜呑みに出来るんだろうな。ワシがおったらアタマしばき倒しておるわ。

くっくっく
0329ご冗談でしょう?名無しさん垢版2020/05/14(木) 16:51:42.04ID:???
まずくっくっくに言っておく
(というか何度も何度もいってるけどおじいちゃんじゃ理解できんか)
電磁場が実際には存在しないとか存在するとかではなく
物理として電磁場を用意した段階で物理的にはあるんだよ
むしろそういう話は哲学的にあるかないかの話になるんで哲学板にいってこいアホ
0330ご冗談でしょう?名無しさん垢版2020/05/14(木) 16:54:00.91ID:???
くっくっく は始めからねじ曲がって腐った老木だから間違い探し用ていどに使えるが
物理学の基本的な内容もすっ飛ばし、自発的対称性の破れとかを喚くアホが多すぎ
物理板レベルなら南部博士の解説本のエッセンスで十分、アホはそれすら理解不能。
0332ご冗談でしょう?名無しさん垢版2020/05/14(木) 17:55:10.08ID:???
そうでもないですよ
人によります

むしろ、定義にこだわらないで自分の思ったこと思った通りにダラダラ垂れ流す人が多いんじゃないですか?

数学板の安達さんみたいな人は定義の重要性を絶対に理解しようとしないわけです
0334ご冗談でしょう?名無しさん垢版2020/05/14(木) 18:50:23.41ID:???
光速に近づくにつれて質量が増加するというのは実験からも実証されてる事ですが

これは対象が向かってくる時も離れていく時も観測者からは同じように質量が増加しているのでしょうか?

それとも観測者が対象を光速方向に加速させようとした際に質量が増加してるのでしょうか?
その際は減速や打ち返す場合は対象の質量が減少しているのでしょうか?
0341ご冗談でしょう?名無しさん垢版2020/05/15(金) 00:29:45.22ID:???
>>334
>光速に近づくにつれて質量が増加する
物体の加速度が力に比例しなくなる事実のニュートン力学的な表現であって今では使われなくなった。
0342ご冗談でしょう?名無しさん垢版2020/05/15(金) 04:39:40.34ID:???
もし空間のもつれが質量になるなら、誰かより速いって相対が質量増大させるのって考えにくいけどどうなの?絶対なら空間との差だから質量増大わかるけど

時間もなんかのもつれで実は絶対の可能性ってない?
0348ご冗談でしょう?名無しさん垢版2020/05/15(金) 11:04:58.69ID:7GMzJM7D
>>346
電子絡めてよろしくお願い申し上げます。
0349ご冗談でしょう?名無しさん垢版2020/05/15(金) 11:11:17.00ID:7GMzJM7D
真空のマイナス準位に詰まった電子を蹴り上げると、陽子電子対が生じるとか言う話よりは、
そもそも電荷とは何だろうかにウェイトを置いて頂きたいです。
0350ご冗談でしょう?名無しさん垢版2020/05/15(金) 11:14:11.82ID:???
>>344
同じ大きさの力を加えて、運動速度だけでなく縦方向の加速度と横方向の加速度が違う量を
定まった「質量」とは呼べないだろ。
運動速度で変化しないのは根源的な「静止質量」。 ついでに「電荷」も変化しない。
0351ご冗談でしょう?名無しさん垢版2020/05/15(金) 11:17:14.35ID:???
>>349
電荷にしても質量に関してもそうなんやけど
パラメータとして与えなあかん量がなんなのかと言われてもよくわからんという話ですね

もちろん物理の未解決問題というか目的としてはより上位の理論を考えてより少ないパラメータからこの電荷や質量を計算できる仕組みを考えることができたならばその意味は見やすくなると思いますけど
0353ご冗談でしょう?名無しさん垢版2020/05/15(金) 12:40:04.83ID:???
電荷は保存則の対象であり保存則に対応する対称性から導かれるゲージ場の源
エネルギー運動量が時空から導かれる重力場の源であるのと同様
保存則⇔対称性⇔ゲージ場⇔ゲージ場源の保存則 は一般的なゲージ原理
0354ご冗談でしょう?名無しさん垢版2020/05/15(金) 13:49:48.74ID:7GMzJM7D
点状粒子と電磁場との結合の強さは∞で、それが近傍の真空を壊し、その分極がまた真空を
壊し・・その極限として、(外から)観測されるのが(素)電荷ということではいけませんか?
逆に、電磁場がきゅーっと強まって、そこのδ関数みたいなのが電荷で、その近傍が電子に
見えるとかじゃ変でしょうか?
0355ご冗談でしょう?名無しさん垢版2020/05/15(金) 13:52:29.97ID:???
電荷っていう粒子が何かあるわけじゃないんですよ
電荷は粒子の持つ性質です
色みたいな感じですかね
電子の色は何色ですかーと言ったら-1ですし、陽子の色は何色ですかーと言ったら+1なわけです
0357ご冗談でしょう?名無しさん垢版2020/05/15(金) 14:12:56.00ID:7GMzJM7D
何というか、場の破綻した点が場に対して持つ量が例えば電荷で、その点が粒子に見えるというか、
定式化できなくてすみません。
0366ご冗談でしょう?名無しさん垢版2020/05/15(金) 16:37:26.18ID:7GMzJM7D
電場と磁場の見え方は速度で変わります。
0370ご冗談でしょう?名無しさん垢版2020/05/16(土) 07:35:04.82ID:+x86wQo4
>>345
電磁気力の結合定数
0372ご冗談でしょう?名無しさん垢版2020/05/16(土) 11:28:44.77ID:???
物理学は図書館や博物館のような分類作業ではない
>原子分子物理学
ならば量子論(第一原理)を基礎に理論展開する物理の領域といえる
0375ご冗談でしょう?名無しさん垢版2020/05/16(土) 12:26:45.50ID:???
>>373
量子力学理論と基礎方程式が与えられただけでは、多電子の原子や分子のエネルギーを簡単に計算などできない。
境界条件で実際の現象はがらりと変わる、解くほうが遥かに難しい。
個々の問題で解く方法を考え出し、実際の物理実験と一致して初めてキミは認められる。
0377ご冗談でしょう?名無しさん垢版2020/05/16(土) 12:56:58.80ID:???
ある演算子の固有状態であるということはその演算子を固有状態で対角化できるということと同値なんですか??
0379ご冗談でしょう?名無しさん垢版2020/05/16(土) 13:32:21.17ID:???
>>361
4元電流ベクトルや4元運動量ベクトルはローレンツ変換する
そのベクトルの時間成分は電荷や質量に対応するが
それが変化するからと言って電荷や質量が変化するとは(現在は)言わない
0380ご冗談でしょう?名無しさん垢版2020/05/16(土) 13:43:25.45ID:???
>>376
キミの反応から、シュレディンガー方程式が出ただけか、記事を検索しただけで
その問題が解決したと思い込む文系?のアホ
0385ご冗談でしょう?名無しさん垢版2020/05/16(土) 14:51:01.52ID:???
分子物理学(ぶんしぶつりがく、英語:molecular physics)とは、分子性物質のマクロな物性と、原子・分子の相互作用・統計的に処理された分子の運動との関係を研究する学問である。

実験的に物性を測定し、分子構造等との関係を調べる(実験)
統計的な処理を用い、ミクロな分子ダイナミクスとマクロ物性との関係を理論的に記述する(理論)
分子動力学の原則に基づき計算機によってマクロ物性を予測する(シミュレーション)
と様々なアプローチがとられているが、それぞれは相互に刺激しあい発展している。
0389ご冗談でしょう?名無しさん垢版2020/05/16(土) 15:41:24.81ID:???
奈良県立医科大学が新型コロナウイルスに規定の濃度のオゾンガスを約1時間噴霧。
感染力を持ったウイルスが最大で1万分の1にまで減少することが確認された。
次に病室などの臨床現場に近い環境で、効果があるかどうか検証する必要がある。

これが科学的方法なのだよ、オゾンの強い酸化作用から効果が有りそうだと推測だけ発表
しても公には認められない、検証が無ければな。
0395ご冗談でしょう?名無しさん垢版2020/05/16(土) 16:27:37.93ID:???
数学では方程式と一般解を求めればある意味で問題解決だが
現実の物理学的問題は境界条件を課した難しい特殊解を解く開発能力が必要
簡単には波動方程式の一般解だけ見ても、現実の弦の振動数は説明できない。
0396ご冗談でしょう?名無しさん垢版2020/05/16(土) 16:47:15.61ID:umirH/r/
フーリエ変換でこれを理解してないと式の変形ができないと思い知った

f(x)=∫[-∞,+∞] {F(+k)exp(+ikx)}dk

=∫[0,+∞] {F(+k)exp(+ikx)}dk
+∫[-∞,0] {F(+k)exp(+ikx)}dk

=∫[0,+∞] {F(+k)exp(+ikx)}dk
+∫[0,+∞] {F(-k)exp(-ikx)}dk

この部分がどの参考書でも省略されてるから自力で導き出すのに苦労した
0398ご冗談でしょう?名無しさん垢版2020/05/16(土) 17:03:20.03ID:???
マルチ

0268 あるケミストさん 2020/05/15 19:30:49
量子化学ってタイトルじゃないけど
高柳「原子分子物理学」朝倉書店
http://www.asakura.c...n/978-4-254-13681-4/
は前期量子論とか面倒な所を飛ばしてすぐに水素原子に入る本だから化学向けに結構良い本だと思う
分子間力までカバーしてるし
0399ご冗談でしょう?名無しさん垢版2020/05/16(土) 17:09:19.65ID:g777QU0o
圧力鍋の説明でよく解らなかったのでお願いします。
http://www.apajapan.org/JPCC/poc.html

> 概ね圧力50〜150kPa、温度にして110~130℃弱で調理するよう設計されており、
これは大気圧を足して、1.5〜2.5気圧の意味かと考えたのですが・・・
> 消費生活用製品安全法では、9.8kPa以上のゲージ圧力で使用するように設計〜(中略)〜「PSCマーク」を表示
これは1.1気圧にも満たない設計に対してお墨付きを与える・・・では、おかしいですよね。
どう解釈したらいいんでしょうか。

同法(https://www.meti.go.jp/product_safety/consumer/system/01.html)を見ると、確かに「9.8キロパスカル」とあります。
0401ご冗談でしょう?名無しさん垢版2020/05/16(土) 17:12:49.58ID:???
単純すぎる話だけど
テンソル積だけでは表現しきれない量子状態が量子エンタングルメントであるという話はどの現代であれば教科書にも載っています。
しかし、その量子状態はテンソル積だけでは表現できない、ということを証明するのは実は計算機科学のSATに相当する問題でNP完全なはずです

そこまで説明した上で例えばしかし実はエンタングルメントにあることを証明するのは現代の計算では不可能であるということまで説明しているテキストを見た記憶がありません

以前この質問をしたらエンタングルメントエントロピー計算すりゃ分かるからいいだろと言われ、それはそうだと納得はできたのですが、なんか釈然としません
0402ご冗談でしょう?名無しさん垢版2020/05/16(土) 17:26:40.64ID:???
>>399
「内容積が10リットル以下のものであって、9.8キロパスカル以上のゲージ圧力で使用するように設計したもの」だけがPSC認証が必要で
そうでないものは要らないということでは?

あと法解釈なら法律関係の掲示板の方が良いとおもう
0403ご冗談でしょう?名無しさん垢版2020/05/16(土) 22:26:46.50ID:ZY0dXVFH
>>353
適当なこと言ってんじゃねーぞサル。
電荷保存則はただの前提条件だぞ。

ρdv=ρ’dv’
こうしないと辻褄が合わないから
そうしてるだけで、辻褄合わせを原理という言葉に
置き換えんなよアホが。

くっくっく
0404ご冗談でしょう?名無しさん垢版2020/05/16(土) 22:34:56.01ID:ZY0dXVFH
>>396
k=−K、dk=−dKとしてすべて置換してから
最後にkと書き直せばいいんだよな。
偉いぞサル

くっくっく
0406ご冗談でしょう?名無しさん垢版2020/05/16(土) 22:53:44.87ID:ZY0dXVFH
>>399
それでいいだろ。
ゲージ圧+大気圧=絶対圧
https://d-engineer.com/fluid/zettaiatu.html

ちなみに
空気1m^3の質量は1kgだから
重力加速度9.8をかけると約10N/m^2だな。

これが10kmの上空まで積み重なってるとして
10N/m^2×10000=100000N/m^2=10万パスカル=100キロパスカルだな。
割とどうでもいいわ。

くっくっく
0408ご冗談でしょう?名無しさん垢版2020/05/16(土) 23:04:05.84ID:ZY0dXVFH
>>407
だから鍋の中の圧力が
大気圧の1.1倍以上なら危険だとみなすんだろ。
それ以下なら割とどうでもいいんだろうな。

くっくっく
0409ご冗談でしょう?名無しさん垢版2020/05/16(土) 23:10:55.37ID:ZY0dXVFH
しかしお前ら
ホントにつまらん話ばっかだよな。
もっと心に響く本質的なことを書いてみろや。

なんかおもろいことないか
たまに見に来ても
なーーーーーんもないわ。

お前らのは表面的なことばっかで
実につまらんわ。
じゃあな。
せいぜい頑張れよサルども

くっくっく
0412ご冗談でしょう?名無しさん垢版2020/05/16(土) 23:14:23.57ID:???
でも戻ってこずにはいられないんだろうな
サルと思ってる連中と死ぬまで戯れるしかないんだよな
なんと哀れな人生か
0413ご冗談でしょう?名無しさん垢版2020/05/16(土) 23:16:59.34ID:ZY0dXVFH
>>401
何回も何回も書いてやってるが、
遠隔作用だってーの。
それを認めたら何のことはないわな。

まったくもって
進歩しない自分で考えることができないサルだよな。

くっくっく
0414ご冗談でしょう?名無しさん垢版2020/05/16(土) 23:32:15.16ID:???
>>408
圧力鍋の強度を議論する際の「圧力」は、一気圧下の「差圧」が重要であって、
真空との差圧である絶対圧を基準とした「気圧比」が圧力鍋の強度を議論する規準には
ならないと思います。
「絶対圧」にこだわる物理的理由があれば説明していただけますか?
0416ご冗談でしょう?名無しさん垢版2020/05/17(日) 00:06:43.14ID:???
鍋の破壊強度を論じるには、鍋の内外の「圧力差」が重要であって、
その「圧力差」と、「真空と1気圧の差」とを比べる意味は無いと思います。
0418ご冗談でしょう?名無しさん垢版2020/05/17(日) 05:46:19.15ID:???
じゃあ気分転換にポエム。量子論について。量子論はこれでいいのか

自分が解る初歩的なところだけど外積と内積の話。常識の範囲だけど外積と内積の生じるための1つの最低条件
・内積…aとbに順序がない
・外積…aとbにaがbを生じさせてるという順序がある
内積を生じるのは力の平行四辺形とか対等な関係が主(かどうかはわからないが)
外積を生じるものには電気の回転が磁気を生じてるという電磁気のような複雑性を伴うものが多い

・フレミングの左手で中指の電気の周りを磁気が「右ネジ」に発生して人差し指の磁気と重なり力の親指の根元の方が順方向で磁気が強くなり先の方が逆方向で弱め合いその差で力が生じる、という初歩のイメージは、
・人差し指の磁気を中心にすると、人差し指の磁気の周りを電気が「左ネジ」に発生して中指の電気と重なり親指の根元の方が順方向で電気が強くなり先の方が逆方向で弱め合い力を生じる、
という風に電気を主格にすると右ネジに磁気、磁気を主格にすると左ネジに電気というどちらが先かプライマリセカンダリの優劣を持つ。順序を持っている2要素→外積を生じる最低条件の1つ。と、常識であるのは勿論分かってる

ようするに電気現象、電子や電荷、量子の振る舞いには順序問題が必ず含まれる。だから量子の計算は順序(因果)関係を几帳面に扱わなければならず、順序(因果)を正しく扱わなければ答えが変わってしまう。ここからが本題
0419ご冗談でしょう?名無しさん垢版2020/05/17(日) 05:47:34.17ID:???
それをよく示すもの、その1つが「電子の振る舞いを行列の計算で表せる」というもの
行列で有名なのが行と列に順序があり交換ができないという性質
足し算が項の総加算、かけ算が項の分配計算、累乗が項の入れ子計算、足し算とかけ算には順序がなく、累乗になり入れ子構造になると順序問題が出てくる

・分配・入れ子などはその計算層の特有性質→計算性
・計算層とは→計算がどのような構造で処理されるか
・順序も→計算性
・行列、集合、合理、論理なども→計算層

行列に順序があり、量子の計算に順序問題がある。そして電子の振る舞いを行列で記述できる。これは行列が電子の順序問題を正しく扱えるということ
電子の位置・運動量とその他の量、交換不可能なこれを行列が正しく扱える
計算法自体に順序が含まれている行列はそこまで気を遣わなくて良いが、順序が交換可能な他の計算法で計算する場合、これを必要以上に気遣う必要がある


おそらく常識であるから量子の理論と計算もそこのところも几帳面に建てられているだろう。しかしそれならばと思う。不確定性原理には言及されていない事柄がある
不確定性原理の否定をするわけではないが前述の通り量子の振る舞いに順序問題が内包されているため、順序・因果関係を正確に想定しなければ誤差を生んでしまう
すなわち何が言いたいかというと「不確定性原理を否定するわけではないが、不確定性の中に回避可能な確定性が存在する…それは順序問題である。これにより不確定性はもっと小さな値になる」
不確定性が実は順序問題を含めた異なる計算層の計算性によってもたらされる確定性を読み解けない故の白旗か、あるいはその上で不確定性が実在するのかは解らない。それは今後解き明かされるのを待つとして、
それでもこの常識範囲の注意点がもし常識でないのなら今一度、全ての理論について順序と因果を精査し検証する必要がある。インテリジェンスのない無学で初歩の自分には無理だが
以下全ての理論…前提や原理からそれは本当に正しいのか
0420ご冗談でしょう?名無しさん垢版2020/05/17(日) 05:48:14.46ID:???
・振る舞いが全て確率で決まる…これは計算層が違うので既存の計算で表せないのを擬似的に表すためのツールというだけで、量子が確率存在というわけではないのでは。そして確率(仮)にも順序はないのか?
(確率自体は順序交換可能(順序なし)。むしろ順序を扱わないために確率としてる可能性)
・重ね合わせ(量子状態)…確率にしても波動にしてもその計算層は?重ね合わせもツールにすぎないのではないか。重ね合わせは相互作用しないのか、それは異なった計算層の違った物理だろう
・もつれ…かけ算物理以外の物理が存在する。本当に観測が媒体なのか、観測だとしてもどんな物理?観測以外にも思考・認識などは媒体になりえないのか、どこまでが人間の観測力でどこからが宇宙の自己認識?
・波動性…計算性により色々な未知の性質がでるのだろう
・不確定性…前述の通り計算性を考慮すると今よりある程度確定性を持たせられる(今より小さくできる)

と、そう主張したいなら実際に理論や計算で示せと言われても自分には無理。聞き流したい奴は聞き流せとしか言えない。あとは、

相対性理論
・光速不変…光はミクロで速度はマクロ、ミクロとマクロの異なる計算層の互換性→不変なのには何か理由がある。光が神だからじゃない
・光速度最高速…計算層の限界が上限だろうがこれが上限か?
・相対性…絶対性もあるし、他の属性もある。計算層ごとに属性が違う
・時間遅延…時間が相対層だと誰が決めた?互換性で見た目そう見えたとしても、時間はかけ算的な物理相互作用を起こさない

などなど解る範囲の学がなくても解る部分だけ指摘できる
という量子論は根元から改良余地があるという主張のポエム
0423ご冗談でしょう?名無しさん垢版2020/05/17(日) 08:00:20.98ID:???
圧力って理科学の世界だと絶対圧が標準的でゲージ圧があんまり出てこないな
工業分野だとゲージ圧が標準的で絶対圧は〜kPa.Absって明記する事が多い
0424ご冗談でしょう?名無しさん垢版2020/05/17(日) 08:05:35.89ID:???
C=A×Bという関係があって実験でCを得るために複数回のAとBを測定したとしてCを求める場合はAの平均値とBの平均値を出してから上の関係に代入してCを求めるべきですか?それとも測定で出たAとBそれぞれの値を使ってCを求めてから平均値を出すほうがいいのでしょうか
0426ご冗談でしょう?名無しさん垢版2020/05/17(日) 08:26:58.21ID:???
>>422
そんな数学者のお気持ち表明みたいなことを実証科学の物理屋が言うなよw
その上でいうとむしろポエムなのは弦理論だな
0429ご冗談でしょう?名無しさん垢版2020/05/17(日) 10:44:52.31ID:F57aymOm
内部結合
0430ご冗談でしょう?名無しさん垢版2020/05/17(日) 10:49:02.16ID:F57aymOm
特殊相対論を肩に乗せ飛躍した量子力学
拡大する場の量子論に呑み込まれるか一般相対論
0432ご冗談でしょう?名無しさん垢版2020/05/17(日) 14:13:49.94ID:HVm2kQ/A
>>395
【本日のテンプレ】
〜だが
〜必要
〜できない(←何故か必ず《否定》が入るんだよなw)
0434ご冗談でしょう?名無しさん垢版2020/05/17(日) 16:10:40.87ID:???
>>431
例えば長方形の面積を2つの辺を測って求めるとするなら
2辺を同時に測ってるとかなら先にかけるほうがよさそう
だがまず縦を数回横を数回測るってんなら平均するべき
0435ご冗談でしょう?名無しさん垢版2020/05/17(日) 16:52:45.00ID:???
塩素を含むフロンガスがオゾン層を破壊するという理論ですがこれって実証されてるんでしょうか?

比重の重いフロンガスが成層圏まで昇るとは考えにくいし
仮に上がるとしても塩素を含む他の軽いガスでも同じ事が言えような
0438ご冗談でしょう?名無しさん垢版2020/05/17(日) 17:35:50.51ID:wVFZYVq2
>>375
【テンプレ・コレクション】
〜だけでは〜できない《否定》
〜(はるかに)難しい《困難》
〜は認められる《ご褒美・動機付け》
0439ご冗談でしょう?名無しさん垢版2020/05/17(日) 17:49:38.83ID:???
>>433
実験的にはそうだけど
実はテンソル積(大雑把に言って掛け算)だけではその式を表現できないってのを証明するのは計算機科学の言葉に翻訳するとNP完全問題のSATを解くのに等しいのよ

だから、ある状態がエンタングルメントしているかどうかをほんとうの意味で式で示すのは意外と大変って話(現状全パターン試すしかない)
0440ご冗談でしょう?名無しさん垢版2020/05/17(日) 17:56:55.93ID:???
直積状態で掛けることは十分条件であって
エンタングルしていることの定義はエンタングルメントエントロピーが非零であること
0442ご冗談でしょう?名無しさん垢版2020/05/17(日) 18:40:46.98ID:Tw45Kd+H
E=mγc^2が質量とエネルギーが等価だと言われるのがいまいちよく分かりません。
静止系即ちγ=1となる系を選べば確かにmc^2分のエネルギーがあるように見えますが、
エネルギーの基準は任意に選ぶことができるはずです。

なので、その静止系でE=0となるように基準を選べばよいのであって、質量とエネルギーの等価性は
そこからは導出できないように思うのです。
E=mγc^2は単に運動エネルギーが相対論ではmv^2/2ではなく、mγc^2−mc^2であると
言っているに過ぎないのでしょうか?
0443ご冗談でしょう?名無しさん垢版2020/05/17(日) 19:25:18.67ID:???
質量0の粒子Aと、質量mの粒子Bがあるとしましょう

EA=0、EB=mc^2

エネルギーの基準を変えて、Bのエネルギーが0になるようにしてみましょう

EA=-mc^2、EB=0

エネルギーの基準を変えたとしても、質量があると質量がない時よりはエネルギーはあるということですね
0444ご冗談でしょう?名無しさん垢版2020/05/17(日) 19:48:00.12ID:???
>>442
その疑問自体は正しい、解説本的な説明だけではE=mc^2の正確な意味を理解できない。
熱力学の知識が有れば理解しやすいが、静止物体の内部エネルギーEと静止質量mの関係を示す。
運動物体の全エネルギーは E + K になる。 Kは運動エネルギーだが単純なmvではない。
特殊相対性理論(ローレンツ変換)では E, m が不変量になるから不定性が無い。
つまり、物体の内部エネルギーの変化(温度変化、原子の励起など)があれは静止質量が変化する。

ニュートン力学(ガリレイ変換)では内部エネルギーは不定性があるから基準を選ぶ必要がある。
0447ご冗談でしょう?名無しさん垢版2020/05/17(日) 20:02:08.45ID:???
E=mc^2の正確な意味を理解すると相対性論が絶対理論で、ニュートン力学が相対理論に見えるから可笑しい。
0448ご冗談でしょう?名無しさん垢版2020/05/17(日) 20:08:43.29ID:???
場の量子論まで言わなくても、粒子の崩壊とか考えないとエネルギー原点の取り方はどうでもよくなると思う
0449ご冗談でしょう?名無しさん垢版2020/05/17(日) 20:20:51.50ID:???
ぶっちゃけ摂動展開とか完全に摂動0次をどう取るかは人間が勝手に決めることであってエネルギー原点がどうこう以前の問題だからな
0450ご冗談でしょう?名無しさん垢版2020/05/17(日) 20:50:28.90ID:T9bTcB9w
丸投げはスレチかもしれんがまじでわからん
優しい人おしえて
https://imgur.com/gallery/doZaonP
0451ご冗談でしょう?名無しさん垢版2020/05/17(日) 21:33:10.34ID:???
量子力学で、エネルギーを誤差ゼロで測定するには、不確定性関係により無限の時間が必要である、というのは誤りですか?
0455ご冗談でしょう?名無しさん垢版2020/05/17(日) 23:11:52.69ID:???
>>453
東北大学の事情は知らないけど何でバンバン成果上げてるのにずっと助教なの?
普通の大学なら量子エネルギーテレポーテーション発見した時点で教授だろあの人
0457jdn垢版2020/05/18(月) 01:51:35.41ID:???
質問です
相対性理論によると速度によって時間が変わります
全ての物事は相対的にできています
相対性とは座標の原点を本人にしても相手にしても空間にしても相関が変わらないということです
どこに原点を打っても相手との関係が現象を起こします
これが時間と空間の歪みです
そこで質問です
回転していた円盤が急に止まった場合、相手との関係、相対性が急に変わってしまいます
円盤の時間と空間が歪みますし、観測者にも原点の場所によらないので歪みの影響が出ます
これは円盤が止まったときに正体不明の 場・界 の変化が伝播したように見えます
その正体は時間と空間です
すると全ての物体は時間と空間の場を常に出してるということになりますか
物体は電気磁気によらず質量の発する重力にもよらず空間と時間を発しているのでしょうか
0458ご冗談でしょう?名無しさん垢版2020/05/18(月) 01:52:38.81ID:TPqN6Xqs
E^4 = m0^2*c*4 + p^2*c^2 = (m0γ)^2*c^4 = m^2*c^4
0459ご冗談でしょう?名無しさん垢版2020/05/18(月) 01:59:08.67ID:TPqN6Xqs
回転していた円盤を急に止めると、円盤がリーマン面みたいに歪んじゃうと思います。
リーマンショックと言うかどうかは知りませんが。
0462jdn垢版2020/05/18(月) 12:40:46.73ID:???
>>460 場というのは間違いなのでしょうか
0465ご冗談でしょう?名無しさん垢版2020/05/18(月) 13:18:08.54ID:???
>>442
特殊相対論で分かるのは4元運動量ベクトルがローレンツ変換することだけ
4元運動量ベクトルの空間成分が3次元の運動量で E=mγc^2 は時間成分
質量0の光子でも鏡で閉じ込めて往復させれば合計の運動量は0にできて
時間成分の E=エネルギー だけ残すことができる
その時の系の質量は E=mc^2 から求める m に一致する
0466ご冗談でしょう?名無しさん垢版2020/05/18(月) 13:22:47.32ID:???
>>457
そもそも円盤が回転してる状態で時空は歪んでる
止まったら歪が変化するだけ
歪が変化すれば重力波が出るが
定常回転してる間は何も出さない
0467ご冗談でしょう?名無しさん垢版2020/05/18(月) 13:37:34.94ID:???
>>465
とどのつまり、物体の質量なるものは素粒子の質量の総和であり
素粒子の質量はゲージ場(光子の類)がヒッグス場に閉じ込められたエネルギー
キミたちはみんな閉じ込められたエネルギーが作りだしたゴーストなのだよ。
幽霊らしく楽しく生きよう
0469ご冗談でしょう?名無しさん垢版2020/05/18(月) 13:46:23.91ID:???
現代物理学に対して古典物理学って言い方は良くないよな
普通に量子力学とニュートン力学って言えばいいし誤解を招かない
相対性理論なんてどっちにも含まれるし他の力学でもどっちも使う事が多い
(熱力学とか流体力学とか
0473ご冗談でしょう?名無しさん垢版2020/05/18(月) 14:07:50.45ID:???
>>472
Wikiには古典物理学とは量子力学を含まない物理学の事をさしてて
現代物理学の対義語ではないとある

そもそも古典力学と古典物理学ってどう違うかいまいち分からないし
0474ご冗談でしょう?名無しさん垢版2020/05/18(月) 14:14:24.19ID:???
>>473
ほんまやな
調べるとマクロを対象としたのが古典物理学だとかいてるわ
現代物理学は対義語でないみたいに書いてるな
0475ご冗談でしょう?名無しさん垢版2020/05/18(月) 14:19:39.61ID:???
1900年頃に前期量子論が始まり、1925−26年に量子力学が完成されて後
量子力学から見てそれ以前の従来の物理学を古典物理学と呼ぶようになった。
フランス大革命以前のアンシャンレジーム(旧体制)と似たようなもの
0476ご冗談でしょう?名無しさん垢版2020/05/18(月) 14:33:00.62ID:e/roHExR
物理学掲示板群 ttp://x0000.net/forum.aspx?id=2

学術巨大掲示板群: アルファ・ラボ ttp://x0000.net
物理学 化学 数学 生物学 天文学 地理地学
IT 電子 工学 国語 方言 言語学 など

ペンローズの量子脳理論
ttp://x0000.net/topic.aspx?id=3702-0
0477ご冗談でしょう?名無しさん垢版2020/05/18(月) 14:34:25.55ID:???
曖昧なら定義して使えばいいだけだけど
相対論とか現代物理とかの項目におかれてて、でも量子物理の文脈では相対論的な解でも量子効果がなければ古典解って扱われるな
0478jdan垢版2020/05/18(月) 15:31:19.18ID:???
>464
>465
>466
>468
ありがとうございます
0479ご冗談でしょう?名無しさん垢版2020/05/18(月) 17:10:08.78ID:???
まあ英語でmodernとかclassicalって言ってるのを訳しただけだろうけど
普通にマクロとミクロって言っとけと思う
0481ご冗談でしょう?名無しさん垢版2020/05/18(月) 17:50:03.10ID:???
馬鹿が多いな単なる呼び名の変更ではないのだよ、それが人間世界を変えてしまう
 平成生まれは知らんだろが、20世紀半ばごろまでは通信機、ラジオ、テレビや
初期のコンピュータは古典物理学の原理で作られた装置が使われていた。
 量子力学によって半導体産業が誕生し半導体装置に置き換えられ事務計算職は失業
して今は無い。いずれキミたちの多くが消えた電話交換手のように仕事を失うことになる。
コロナウィルスがそれを加速し、その後バラマキ給付などで最低生活するか一部の勝ち組になるか
キミの人間能力次第だ。
0482ご冗談でしょう?名無しさん垢版2020/05/18(月) 17:52:15.35ID:???
地球上の氷がすべて解けたとしても
海水面はあがらない変わらないって
聞いたんですが本当ですか?

地球温暖化で海水面が上がるって
新聞とかでよく見かけたんですが。
0483ご冗談でしょう?名無しさん垢版2020/05/18(月) 18:37:10.45ID:???
グリーンランド、南極の大量の氷は海水面より高い。
地球温暖化の海面上昇より前にウィルスかAIに滅ぼされるから心配すんな
0485ご冗談でしょう?名無しさん垢版2020/05/18(月) 18:45:35.12ID:HH+JKpD8
検察庁法改正案に抗議した人達
https://twitter.com/chibiinochi/status/1259651894214971392/photo/1

俳優    :城田優、小泉今日子、西郷輝彦、浅野忠信、井浦新
タレント  :高田延彦、杉本大輔
アーティスト:Chara、綾小路翔、オカモトレイジ、きゃりーぱみゅぱみゅ、しりあがり寿、大友良英、水野良樹、宮本亜門
政治家   :志位和夫、福島みずほ、立憲民主党(公式)

https://www.sankei.com/affairs/news/200512/afr2005120025-n1.html ほかより
https://twitter.com/5chan_nel (5ch newer account)
0489ご冗談でしょう?名無しさん垢版2020/05/18(月) 19:48:29.22ID:???
>>486
マクロの実生活はミクロ生物とミクロ科学技術に支配されている
外見は変わらん装置でも古典物理学の原理ではナノテクAIは不可能なのだよ。
21世紀の半導体集積技術、デジタルAI技術で日本人と日本企業は敗北した
殆どの馬鹿は大学出ても3K仕事で家賃も払えなくなるかもな。
俺は普通の知能しかないが日本の全盛期でウハウハだったからもういらん。
0490ご冗談でしょう?名無しさん垢版2020/05/18(月) 19:56:03.40ID:???
>>488
海の面積は3億6000万平方キロメートル
南極の面積は1600万平方キロメートル
おおよそ3.9%の比率

南極の氷が1m解けるだけで40mm上昇する
南極の平均氷床は厚さ2400mほど
2400m x 0.039 = 93.6m

たった93mなら安心だね
0491ご冗談でしょう?名無しさん垢版2020/05/18(月) 19:56:31.04ID:???
>>488
メートル単位で上がるだろ、南極、グリーンランドの氷が平均数100メートル標高として
陸地面積と海洋面積で簡単に計算してみればよい。
0493ご冗談でしょう?名無しさん垢版2020/05/18(月) 20:11:41.07ID:???
ちょっと質問だけど現代物理では先に多様体という数学を詰める容器を想定するの?
それともその先まで進んでて、何らかの前提計算が多様体を仮定し、その中に物理法則を詰めていくって考えるの?
0494ご冗談でしょう?名無しさん垢版2020/05/18(月) 20:13:10.67ID:???
>>492
まあ一番寒い地域は平均マイナス50℃以下だからな
そこの氷が全て解けるという事は他の地域もプラス50℃くらい気温が上がってるだろう

気温+50℃に比べれば海面上昇+90mとか些細な問題
0495ご冗談でしょう?名無しさん垢版2020/05/18(月) 20:22:13.24ID:???
>>493
一般相対性理論が正しいことを信じるならば、時空はリーマン多様体によって記述されなければなりません

多様体ってのは、簡単に言えばxy座標で場所表しましょうということなのですから、それほど不自然な仮定でもないわけです
0496ご冗談でしょう?名無しさん垢版2020/05/18(月) 20:26:57.90ID:???
>>494
そんな単純ではない、南極大陸が極度に低温なのは南極大陸を周回する海流が原因という
気候変動で世界の大海流の流れが少し変われば気温が大きく変わる。
0500ご冗談でしょう?名無しさん垢版2020/05/18(月) 21:26:44.19ID:???
質問 
なぁみんな 前スレさいごの電気の単位書き込まれたの考えてみないか?
ですかか?

質問
なぁみんな 暇つぶししないか?
ですか?
0502ご冗談でしょう?名無しさん垢版2020/05/18(月) 22:15:23.73ID:???
>>490-492
理科の実験だけどさ、
1. コップに水をいれて氷をうかべる。
2.水面の高さを計測する。

氷が溶ける前 も 溶けた後も水面は同じ高さ。

この理屈から、浮いている氷は地表に出ている部分と沈んでいる部分で
浮力のバランスが一定だと分かる。
水面の氷が溶けたらその分、水が増えるが、浮力を維持するために
沈み込む部分の体積が小さくなるので、そこで相殺される。

海中内の氷が溶けたら、その氷が存在していた空間が
空きスペースになり、そこが水と置き換えられるだけ。
よって、水面は変わらない。
0507ご冗談でしょう?名無しさん垢版2020/05/18(月) 22:24:52.33ID:???
・海中の氷 … 溶けたらそのスペースが水に置き換わる
・海面に浮いている氷 … 溶けたら
その増えた水の体積を打ち消すように、氷塊の沈み込み部分の体積が小さくなる。
浮力のバランスがとれており、全体としては変わらず。

↑ 海水と接している氷については全てこの2つで相殺される。

よって、海水と接していない (海水による浮力を受けていない氷) 独立した氷が
溶けた分だけが液体の増加量として、海水面の上昇に影響を与えうる。

となると、氷床2000m とか意味のない数字になる。
海水から独立している氷塊の体積分だけを考えればいい。
0508ご冗談でしょう?名無しさん垢版2020/05/18(月) 22:29:38.08ID:???
>>505

海水に接していたり、
浮力の影響を受けている氷は溶けても溶けなくても同じ。

氷が存在していた部分の空間を無視しないで。
0509ご冗談でしょう?名無しさん垢版2020/05/18(月) 22:31:55.94ID:???
当然だが北極の氷も海水と塩分濃度が異なるので実際には海面上昇する
これがどれくらい深刻かは各自計算するべし
0510ご冗談でしょう?名無しさん垢版2020/05/18(月) 22:31:58.50ID:???
コップに水と大きい氷をいれて底と接地させて…南極大陸とする。
コップに氷をいくつか浮かべて…北極の氷や氷塊とする。

水面の高さを測ると…氷が溶ける前も溶けた後も変わらない。
0513ご冗談でしょう?名無しさん垢版2020/05/18(月) 22:40:36.03ID:???
いや、割とマジで
俺の言っていること可笑しいか?
アルキメデスの原理からしてそうだろ。

氷が解けるということは水の量が増える、
しかし、一方で氷の量が減る。
容積で見ると、この2つは相殺する。

海面と接しておらず、浮力の影響をうけない
独立した氷塊 から溶けでた水だけが海面上昇に寄与する。
0515ご冗談でしょう?名無しさん垢版2020/05/18(月) 22:52:09.27ID:???
>>509
水→氷…海面に出ている氷の分と同じだけ、氷から水になると体積減るからプラマイ0
塩水→純氷…塩水の浮力の関係で海面に出る氷が増えるが、氷から塩水の体積減少率はそこまで変わらない→氷が出すぎてる分だけ海面上昇

ってこと?
0516ご冗談でしょう?名無しさん垢版2020/05/18(月) 22:52:18.16ID:???
誰も俺に賛同しねぇんだな…
天動説を唱えた時のガリレオみたいな気分…

「大陸上にあり浮力から独立した氷」
が溶けた分だけだぞ?
それだけで海水面を上昇させるって尋常じゃないぞ。

海の広さから見たら、微量だ。
せいぜい、数mm だろ?
0518ご冗談でしょう?名無しさん垢版2020/05/18(月) 22:54:02.27ID:???
例えば、エベレストの頂上にある氷を
持って帰って、日本に来て、海に放り込めば、それは海水面の上昇につながる。

独立した氷ってそういうのだぞ?
0519ご冗談でしょう?名無しさん垢版2020/05/18(月) 22:57:53.10ID:???
>>517
逆に言うが 相殺しない のは、
南極大陸、および、グリーンランドの地上部にあり、
かつ、海水の浮力の影響を受けていない氷塊 のみだ。

そういうわけで、北極など浮いている分は
全て無視できる。

独立していないあらゆる氷で相殺が起こる。
溶けて水が増えるというのは、
氷が減ることの埋め合わせに過ぎない。
0526ご冗談でしょう?名無しさん垢版2020/05/18(月) 23:27:10.69ID:???
海面の下の氷=水槽の水が押しやられる
海面の上の氷=水槽の水は押しやられない
海面の下の氷=水を湛える領域の形状
海面の上の氷=水を湛える領域外
海面の下の氷=海面の高さに関係
海面の上の氷=海面の高さに無関係
海面の下の氷=が溶けると体積差で海面低下
海面の上の氷=が溶けると水になった分の水が加水

南極大陸の上の氷=100%→水へ─体積減少0+100%
海の中の氷=浮力の差で出ている○%→水へ─マイナス体積減少+○%
0527ご冗談でしょう?名無しさん垢版2020/05/18(月) 23:27:14.94ID:???
>>444
>特殊相対性理論(ローレンツ変換)では E, m が不変量になるから不定性が無い。

これはどういうことでしょうか。
エネルギー基準を任意にとってE=mγc^2+αとしたとき、ローレンツ不変性を保つためには
α=0でなければならないということでしょうか?
0528ご冗談でしょう?名無しさん垢版2020/05/18(月) 23:27:41.98ID:???
>>525
その氷の体積は
1600万 m^2 の面積の上に
高さ 20億 m の氷の塊があることになる。
さすがに標高 20億 m はおかしい。

2大部分は水中に沈んでいるな。
0531ご冗談でしょう?名無しさん垢版2020/05/18(月) 23:45:04.16ID:???
>>525
ごめん、 高さ 2000m余り だったわ。

南極大陸
・面積 A = 1400万 (mxm)
・氷床の体積 V = 3000万 (mxmxm)

よって、高さ H = 2140 m

これを全て独立した氷だとすると、
溶けた場合に海に与える影響は…

面積比 で 南極大陸 : 海 = 1: 26
なので
2140m の 1/26 で 82 m

海の全体を82m押し上げることになる。

マジかよ…
0532ご冗談でしょう?名無しさん垢版2020/05/18(月) 23:46:43.55ID:???
>>531
仮に氷床のうち、独立した氷を
少なく見積もって 1/10 としても、
全てが解ければ 8m 上がる

割と深刻だな…、水泳教室通うか…
0533ご冗談でしょう?名無しさん垢版2020/05/18(月) 23:54:19.02ID:???
水に浮いてる氷が融けても水面の高さが変化しないってのはアルキメデスの
原理を習った中学生でも知ってることなので、埋め合わせだの相殺だのと
拙い説明されなくても、そんなことはみんな百も承知だろ
(実際は海水(塩水)に浮いてる淡水の氷が融けると少し上昇するが、それはさておき)

>>490でまさに南極の(この人の言い方では相殺しない)氷だけからくる
上昇分が見積もられてるのに何が不満なんだ?
0534ご冗談でしょう?名無しさん垢版2020/05/18(月) 23:59:10.79ID:???
>>526
塩水と純氷の(質量差)浮力の差で浮いてる氷が余計に出てるから加水量が多くなって体積減少を上回り海面上昇するってこと
0535ご冗談でしょう?名無しさん垢版2020/05/19(火) 00:10:23.40ID:???
誠にお騒がせしました m(__)m

>>533
ごめんっつってんだろが。


厚さ平均 2000mの氷床のうち、
水上に出ているのが何%かが分からないのが怖い。
5% なら4 m、 10% なら 8m 、50% なら40m
メートル単位でも凄いんだな。

応用問題だけどさ、
南極の独立した氷を全て溶かすとして…
どれだけの気温上昇が必要なんだろう…。
0537age垢版2020/05/19(火) 00:23:59.18ID:2C8u9IiH
さてこれやる?

・・・・・・・・・ここから引用・・・・・・・・・

N/A^2=μ、N/V^2=ε 則
(kg^3)(m^2)誘磁│1/(kg^3)(m^2)透磁μ
(kg^3)(s^2)誘電ε│1/(kg^3)(s^2)透電
リスト1
√(kg)(m)/(s^2)
││
├÷1/(kg^3)(m^2)・・・・・(kg^2)(m^3/2)/(s)│電流
├÷(kg^3)(m^2)・・・・・1/(kg)(m^1/2)(s)│正孔

├÷(kg^3)(s^2)・・・・・(m^1/2)/(kg)(s^2)│電圧
└÷1/(kg^3)(s^2)・・・・・(kg^2)(m^1/2)│電荷
リスト2
√(kg)(s)/(m^2)
││
├÷(kg^3)(s^2)・・・・・1/(kg)(s^1/2)(m)│反荷
├÷1/(kg^3)(s^2)・・・・・(kg^2)(s^3/2)/(m)│磁洞

├÷1/(kg^3)(m^2)・・・・・(kg^2)(s^1/2)│磁荷
└÷(kg^3)(m^2)・・・・・(s^1/2)/(kg)(m^2)│反洞

電荷×m/s=電流
正孔×m/s=電圧
電流×電圧=電力
電荷×正孔=kg/s

(kg)(m)/(s^2)ニュートン
(kg)(s)/(m^2)ヒキニートン
0538ご冗談でしょう?名無しさん垢版2020/05/19(火) 01:01:02.17ID:???
>>537 言っとくがお前のそれを認めてるわけではないからな。勘違いするなよ
あくまで不明だ
やるなら質問から仕切り直しな
0539age垢版2020/05/19(火) 11:33:49.96ID:mMnXKRh2
質問
電気をkg、m、sのみで表してみないか?
ですか?

質問
電気の単位を考えてみる
ですか?

質問
W(kg)(m^2)/(s^3)→を2つ(電流と電圧)に分解しなさい
1/c(m/s)^2→を2つ(誘電率と透磁率)に分解しなさい

A(電流)×V(電圧)はW(電力)ですが、上の表が正しいならa(電荷)×v(正孔)=(kg/s)になります。これは正しいでしょうか

上の表が正しいなら、V(電圧)/A(電流)もしくはv(正孔)/a(電荷)でΩ=1/(kg^3)(m)(s)になります。これは正しいでしょうか
ですか?

質問
ゆくゆくは実証してみよう
ですか?

質問
理論と絡めてみるのも面白い
ですか?

質問

ですね
0541age垢版2020/05/19(火) 12:57:32.02ID:mMnXKRh2
(kg)(m^2)/(s^3)を分解

(kg)×1
パターン
kg×1/2│kg×1/2
kg×1│kg×0
kg×3/2│kg×−1/2
kg×2│kg×−1

(m)×2
パターン
m×1│m×1
m×2│m×0
m×3│m×−1
m×3/2│m×1/2
m×5/2│m×−1/2

1/(s)×3
パターン
s×2│s×1
s×3│s×0
s×5/2│s×1/2
s×3/2│s×3/2

この組み合わせを考える
0543ご冗談でしょう?名無しさん垢版2020/05/19(火) 14:11:09.70ID:???
3元馬鹿がまた荒らしてるな  SI実用単位をやめてヤードポンド、尺貫法に戻せと騒ぐバカと同じ。
結論から言えば3元単位系の実用価値はない。
SI実用単位では 電圧x電流=電力 J/s でAは積で消え、日常便利に使ってる。 同じく
抵抗xコンデンサ=時間 s になる。 自分で確かめられるが電気理論を知らないと便利に使えない。

3元単位系は理論物理的な価値もない。
理論物理では自然単位系が便利であり、長さ、時間、質量のどれか一つを単位にすればよい。
さらにプランクスケールを取れば単位が定められる。ファインマン曰く、すべて長さに還元される。
0544age垢版2020/05/19(火) 14:23:05.99ID:???
>>543 わかったよ。了解。偽でFAね。そういうことにしといてやるよ

>3元単位系は理論物理的な価値もない
これについてそう証明できる高度な理論を語れれば完璧なんだけどな。こっちも納得するから

上のポエマーの方が説明してるだけマシ
0547age垢版2020/05/19(火) 15:16:43.76ID:mMnXKRh2
簡単な証明でもするか

前提・(電力)(kg)(m^2)/(s^3)は3元単位。それ以外の単位は介在しない。3元のみ

前提・(電流)Aはしばしば(強度)Iと解釈される

疑問・なぜ電流は(強度)のみの単位で、電圧は(強度)(質量)(距離)(時間)の全ての単位で構成されるのか。俗に言えば差別

条件・(電流)×(電圧)で(電力)が構成されるなら(電力)の3元が両者に同じように分配されてなければならない

条件・その上で(電流)と(電圧)とで(強度)などが打ち消されて(電力)が3元となることはありえる
しかし電流はそもそも流れなのに(m/s)を持たず(強度)Iのみで構成されるとは考えられない
ないし電荷には周りに力を及ぼす性質がある。力を及ぼす性質に(質量)の介在しないことはありえない

結論・(電流)に(強度)などの打ち消されるものがあったとしても、(3元単位)が介在しないことはありえない→よって(電力)の(3元単位)を分配することは物理的に意味がある

仮説・(電力)の見た目は(3元単位)だが(他の元)が同時に介在している可能性もある。それは通常使う…つまり古典物理に還元するのに考慮する必要がないため見えなくても問題ないのだろう
(電流)(電圧)を古典物理の外で使う場合(他の元)も必要となるのだろう。もちろん(仕事)も
0548age垢版2020/05/19(火) 15:33:20.43ID:???
(光速)cが単位ってこれ(m/s)だぞ?絶対の物差しかもしれないけど
つまりcを単位にしてる時点で3元単位を肯定していることに気づいていない。他の元なら換算可能でない
(光速)cが(電気的な単位)×(m/s)であればそれは(光速)でなくともよく、1/(m/s)^2を誘電率透磁率に分配する時点で電気の単位には(3元単位)が含まれる
ならば電流にも3元単位が含まれる。含まれないなら光速じゃなくて電流か電荷を絶対の基準にすればよい。
この電荷を割り出すために光速を基準に使っても(m/s)は要らないから1/√εμ=cと定めなくてよい。1/εμにcの中の電気的性質を分配するだけで光の速度と整合性をとらなくてよい
光の速度を気にしてる時点で3元単位を含む
そして(電気の単位)は(3元単位)と違ってよく見えないのと、(電力)や(力)という古典物理に還元する場合、(電気の単位)は必要ない。この主旨は(電気)の中の(電気の単位)以外の(3元単位)を換算してみようという主旨
0550ご冗談でしょう?名無しさん垢版2020/05/19(火) 16:12:07.59ID:???
>>548
>(電気の単位)以外の(3元単位)を換算してみようという主旨
従来からの3元単位系の内容と同様で、新しい事実は何もない。

>(光速)cが単位ってこれ(m/s)だぞ
勘違い。
相対性理論の光速cは普遍定数 もちろん相対性理論を承認するのが前提。
これで時間は長さに還元される。
真空の誘電率、透磁率とは直接関係がなく、電磁気学で1/√εμ=cと関係付ける。

同じく量子論のℏも普遍定数 量子論(量子力学)を承認するのが前提。
素電荷を定数1にでき、同様に質量(エネルギー)が長さに還元される。
つまり、自然単位系は1元単位になる。
0554ご冗談でしょう?名無しさん垢版2020/05/19(火) 16:57:45.58ID:???
>>550 わかったよ。わるかったな。わるかったなってことにしとくよ

ついでにこっちの言葉に反論って形じゃなくてお前自身の理論を1から通しで聞きたいんだが。「物理的意味がないこと」を「文章の意味で説明・証明」してくれ

まぁ、どっちみち暇つぶしにはならんか。質問取り下げるよ
0555ご冗談でしょう?名無しさん垢版2020/05/19(火) 17:04:34.20ID:???
プランクスケールの物理解釈を単位次元とみるか無次元とみるかの違いか
万有引力定数が普遍定数かどうかも怪しい
0556ご冗談でしょう?名無しさん垢版2020/05/19(火) 17:11:31.48ID:???
もちろん特定分野の物理単位系を無次元化すると数学処理が簡単になることをその人たちは知っている。
0557ご冗談でしょう?名無しさん垢版2020/05/19(火) 17:20:47.86ID:???
wikipediaレベルでも単位系の話なんざ腐るほどあるので見てみるといい
要は単位系とは何に使うに便利か、という話

こういう目的でこういう単位系を提案したい、っていうなら意味はある話。目的をはっきりさせないといけない
たとえば機械設計なんかでは過去、工学単位系という力をkgfで量る単位系があった。これは静力学では便利な面もある
理論物理的なら自然単位系がいい。理論計算の時に物理定数をいっぱい書かなくて済むからな
物理実験でSI単位系がいつもふさわしいかはわからんが、測定値を換算する手間を考えてSIで落ち着いたのだろう
過去腐るほど単位系はあった。私見だが、多分新たな単位系の提案は不要だろう
0559ご冗談でしょう?名無しさん垢版2020/05/19(火) 17:52:30.70ID:???
>>557
一応工学単位というのは単体での測定が可能」かどうかという目安がある
あとは計量法でお金に絡む物は単独単位になる場合がある
仕事柄勉強したけど光と色の単位は鬼門だったな

まあ物理学とはちょっと毛色の違う話だけど
0560ご冗談でしょう?名無しさん垢版2020/05/19(火) 18:21:01.73ID:???
>>558
無関係じゃないだろ
>>550
の3元単位系から1元単位系への説明から c,h が普遍定数であるとして正当化してきた。
普遍定数でなければ、正確な単位変換ができない。
 2007年には原子干渉計を用いた測定による万有引力定数として、G =6.693(21)×10−11 m3 s−2 kg−1
というそれまでの測定結果(6.67259(30)×10−11 m3 s−2 kg−1)とは著しく異なった値が
サイエンスに報告された。 wikiより

>>553 >MKSならc=h=G=1とすればすべて無次元になる
との主張は 数学的にはそうなると言ってるだけ。
0562ご冗談でしょう?名無しさん垢版2020/05/19(火) 18:33:18.72ID:???
アインシュタインの一般相対性理論の「第1近似としてのニュートンの理論」中で
万有引力定数が 6.7 と2桁で粗く書かれ 当時の高い精度値を使ってない。
アインシュタインが普遍定数と考えていなかったのか非常に興味がわく。
0563ご冗談でしょう?名無しさん垢版2020/05/19(火) 18:51:19.46ID:???
強い力が近距離で弱くなるとか、弱い力の到達距離が10^-15cm などどうやって測るのか?
素人にはさっぱり解らんが、エネルギー(質量)が長さの単位に還元できることを知れば
超大型加速器で加速した素粒子衝突のエネルギーから換算できることが推測できる。
0564ご冗談でしょう?名無しさん垢版2020/05/19(火) 18:52:35.10ID:???
>>560
ゼロ元単位系にするには関係ないという話だろ
[G]を1にしたときの単位元の取り方というだけ
例えば6.7×10−11 m3 s−2 kg−1を1にしたときに単位変換は正確に行える
この数値が将来変わっても同様

ちなみに自然単位系を0元ではなく1元単位系としてる文献ってあるの?
0566ご冗談でしょう?名無しさん垢版2020/05/19(火) 19:14:57.83ID:???
>>563
すくなくともLHCなどの大型加速器の衝突実験での測定に万有引力定数など使わないだろうから
SI単位系の精密計測をc,hの1元単位系に換算して使うと推定できる。
0568ご冗談でしょう?名無しさん垢版2020/05/19(火) 19:32:23.62ID:???
>>566
LHCのでも重力作用が観測されるエネルギーの10^-15にしかならないから無いと同じ。
リサちゃんとか災害妄想学者がブラックホールが発生して地球が無くなるとか騒いだが
全部フェイクだった。消えたのはリサちゃん達の方だった。
0569ご冗談でしょう?名無しさん垢版2020/05/19(火) 20:42:02.45ID:???
>>564
素粒子理論は「自然単位系」って称してc = h/2π = 1 ≠G じゃない?
Gまで1にしたら馬鹿でかいエネルギー単位になるし不便だってのもあるだろうけど
0570ご冗談でしょう?名無しさん垢版2020/05/19(火) 20:51:21.47ID:???
【悲報】長期ネットストーカー天羽優子氏、とうとう発狂して意味不明な言葉を並べ始める【精神疾患】

609 名前:名無しさん [sage] :2020/05/19(火) 17:05:23.01 ID:X1XjpvJz
>>607-608
おまえはID変えて連投してないで
特異の詩吟でも吟じてろ
0571ご冗談でしょう?名無しさん垢版2020/05/19(火) 21:54:23.97ID:???
>>529
なるほど…。

単純に

E=mc^2+mv^2/2+…

の展開形を見るとニュートン力学の運動エネルギーの拡張だということはすぐに気づきます。
初項のmc^2は意味のない付属品であって基準点の任意性に吸収されると解釈するのが
ニュートン力学からの推論からは普通のように見えますが、
これに積極的な意味を見出して静止エネルギーと宣言したのは相当勇気の要ることだったんじゃないですかね。

当時はまだ実証できる物理的な証拠は何もなかったはずなので。
0572ご冗談でしょう?名無しさん垢版2020/05/19(火) 22:00:37.43ID:???
>>571
アインシュタインがいなくても特殊相対論は数年の間にできたであろう
とか言われてるけどやっぱ難しかったと思うね
0573ご冗談でしょう?名無しさん垢版2020/05/19(火) 22:29:27.38ID:???
>>569
現在か近未来でも地球上で検証実験不可能なプランクスケールの理論単位に実用性はない
アインシュタインの一般相対性理論でもc=1でしかない。
5ch物理板レベルが妄想しても死ぬまで実際に使うこともないから金縁の額にでも書いて飾ればよい。

地球上で原子・素粒子理論の検証実験等で実用的な自然単位系にするには微細構造定数を使う。
0575ご冗談でしょう?名無しさん垢版2020/05/19(火) 22:53:56.32ID:???
>>571
1903年にソディにより放射性崩壊(ラジウムの崩壊)が発見されて、1905年の6月に特殊相対性理論の論文が、
9月にE=mc^2の等価性の論文が受理されてますね
後者の論文ではラジウムを見ればこの等価性を検証できると提案しています
おっしゃるとおりです
0576ご冗談でしょう?名無しさん垢版2020/05/19(火) 23:10:18.25ID:???
>>574
>プランクスケールは関係ないよ
大有りだよ、物理学は数学と違って物理解釈できないと単に全部1にするだけではダメだ
仮に単位の桁を変更して実験のスケールに合わせてたとしても
万有引力定数(普遍定数がどうかも定かでない)の精度が悪すぎて実用にならない。
0577ご冗談でしょう?名無しさん垢版2020/05/19(火) 23:13:56.64ID:???
単位系絡みの雑学本一般書をざっと見繕って探したけど最後の巻末のエピローグ的にゲージ原理や繰り込みの「お話」ぐらい入れればいいのに。
0580ご冗談でしょう?名無しさん垢版2020/05/19(火) 23:40:40.18ID:???
初歩的な質問で恐縮なんですがお願いします。
液中での浮力についてです。
液中で受ける浮力は物体が液体を除去する体積✕液体の比重✕重力加速度となると思います。
例えばピンポン玉の体積が33cm^3だとして一般的な水の密度を1g/cm^3であれば浮力は33/1000✕9.8(N)だと思います。
そこで質問なのですが、このピンポン玉は33gまでのオモリを吊るしても水面に浮いていられると解釈しても問題ないでしょうか。便宜的にピンポン玉の質量を無視した場合と考えたとしたらの話でよいのですが。

質問が下手ですみません。
よろしくお願いします。
0582ご冗談でしょう?名無しさん垢版2020/05/19(火) 23:55:20.42ID:???
>>581
ありがとうございます
そうですね重りも浮力を受けますが無視してよいと言う場合はどうでしょうか
実際にピンポン玉と浮力ゼロの錘30gを質量ゼロの糸で繋いで水に入れたら浮きますでしょうか沈むでしょうか
0583ご冗談でしょう?名無しさん垢版2020/05/20(水) 00:07:32.15ID:???
>>582
ピンポン玉が沈む高さが直感的に不明なら解るが、浮き沈みは自明だろう
ピンポン玉が全部沈んた時の全体の力を考えればいいだけ。
0584ご冗談でしょう?名無しさん垢版2020/05/20(水) 00:36:38.40ID:???
>>583
浮くという事で納得しました
ありがとうございました
何かと言うと釣りの仕掛けをピンポン玉を利用して作ろうとしておりました
0586ご冗談でしょう?名無しさん垢版2020/05/20(水) 03:06:08.96ID:???
南極の氷が溶けて水面が上がるのが事実ですが、
実際に溶けているんですか?
すべてが溶けるために必要な温度上昇は何度くらいですか?

海水面が上がった原因が、
「人類による温暖化で南極の氷を溶かした」
という説がどうしても私には信じられません。

人類の産業活動によるエネルギーなんて
太陽のエネルギーと比べれば微々たるものじゃないですか?

地球や太陽活動の変化が原因だと疑う方が合理的だと思うんです。
0587ご冗談でしょう?名無しさん垢版2020/05/20(水) 03:25:34.42ID:???
>>586
地中に眠ってた炭素が二酸化炭素になっているし
森林伐採などで光合成で減らす量も減ってる
それがつみかさなればそらっていう
0589ご冗談でしょう?名無しさん垢版2020/05/20(水) 06:00:30.72ID:???
>>580
その計算だと単位合わない

>>586
二酸化炭素だけ騒がれてるけど単に熱だけでも温暖化する。宇宙空間への放熱量を生成熱が上回る
海面の高さと人類の活動を照らし合わせて時間的に相関性が見えるなら(それだけでそうとは言い切れないが)熱単体でも温暖化の1要素になっていると予想できる
0590ご冗談でしょう?名無しさん垢版2020/05/20(水) 07:08:03.51ID:kPz8ygG4
質問です
夕空はなぜ赤いのか、下の説明を読んでもイマイチ納得できません
https://global.canon/ja/technology/s_labo/light/001/01.html

1)朝夕の空が赤くなるのは、青い光が散乱され切って赤やオレンジの
色が強調されるからとありますが、赤い光が届くだけなら太陽は赤く見える
けど空は赤くならないのではないでしょうか?
大気を通る距離が長くなると青い光が減衰して、赤い光の散乱が青い光の
散乱より近くまで発生し、それが見えるからという説明ならまだ理解できますが、
そうではないのでしょうか?

2)そもそも太陽が月のように黒い空に丸い部分だけ明るくならないのは、
平行に地表に降り注いだ太陽光があらゆる方向で散乱するからだと理解して
いますが、それでいいでしょうか?
その場合、大気を通る距離が長くなると赤くなって見えるなら、なぜ真昼でも
地平線方向は赤くならないのでしょうか?

3)大気を通る距離が長くなるほど長い波長の光が届くなら、朝夕の空は
天頂(青)から地平線(赤)にかけて虹色のようにグラデーションに
なるのではないでしょうか?
そうなっているように見えないのはなぜでしょうか?

こんな疑問がわきました
小学生レベルの質問ですみませんが、もし分かったら教えてください
0592ご冗談でしょう?名無しさん垢版2020/05/20(水) 07:15:17.51ID:???
>>586
南極の氷はまだまだ溶けない
マイナス20℃がマイナス15℃になっても氷は氷のままでしょ?

もっと緯度の低い地域や山での積雪が減るのが先

それとは関係なく年間2mmくらい海面上昇してるけど、これは8000年前から続いてると見られてる
0594ご冗談でしょう?名無しさん垢版2020/05/20(水) 07:40:15.95ID:???
>>590
グラデーションというなら宇宙から地球半球を見て昼の地域は青く夕方の地域は赤いだろうけど…

でもそうだな夕方は赤いのに朝焼けって赤くないよな。ドップラー効果って方がまだ信用できるかもしれん
0595ご冗談でしょう?名無しさん垢版2020/05/20(水) 07:50:47.28ID:???
>>594
朝焼けが赤くなりにくいのは大気のチリや水蒸気が少ないから
夕方は昼に温められたチリが舞い上がって乱反射して赤くなりやすいため

朝焼け夕焼けの無い白夜はずっとオレンジだよ
0596ご冗談でしょう?名無しさん垢版2020/05/20(水) 07:51:00.97ID:???
>>594
ドップラー効果だとすると、大気の西から東に吹くローテーション(偏西風など)によって大気の粒子に反射する光が赤に傾くか…朝焼け青くないな違うな
0597ご冗談でしょう?名無しさん垢版2020/05/20(水) 08:00:54.47ID:kPz8ygG4
>>595
ありがとうございます
朝夕の空が赤くなるのは大気のチリや水蒸気が原因ではなく
ガス(窒素、酸素)の分子がレイリー散乱するからみたいですが
やはりチリや水蒸気も関係するのでしょうか?

アイスランドの白夜の空
https://icelandguide.net/2019/06/white-night/

ここでは朝焼け夕焼けみたいに見えていますね
地平線方向にはグラデーションがあるようにも見えます
空が赤く見えるのは雲の反射も大きいですね
0598ご冗談でしょう?名無しさん垢版2020/05/20(水) 08:04:00.68ID:???
>>595
水分量によって色が変わるなら雨の日とか太平洋の真上とか砂漠の上とか、昼間・朝・夕方地域によって空の色が微妙に異なることになる
0599ご冗談でしょう?名無しさん垢版2020/05/20(水) 08:22:57.44ID:kPz8ygG4
>>595
このサイトを見ると朝焼けと夕焼けの違いは上昇気流による水蒸気やチリが
関係するとありました
https://gimon-sukkiri.jp/sunrise_sunset/

しかし具体的なメカニズムはイマイチ曖昧です
水蒸気やチリが赤色の散乱をより多く起こすというより太陽光全体を
減衰させるので、より短い距離で赤色のレイリー散乱を観察しやすいから
夕焼けの方が暗く赤く見えるのではないでしょうか
0601ご冗談でしょう?名無しさん垢版2020/05/20(水) 09:11:16.13ID:kPz8ygG4
>>600
朝夕の違いは暗さと赤さの両方が観察されるようです
太陽光の赤色への偏位は大気の距離と関係(比例?)するようです
以上を考えると全体の強度が落ちればより短い距離で赤色への偏位が
観察されるのではないかということです

つまり朝日ももっと距離があれば赤く暗くなりますが
夕日は水蒸気とチリのせいでより短い距離で赤色への偏位が起こっている
と考えられるのではないでしょうか

大気によって青色のみ減衰するメカニズムはどのようなものでしょうか
青色のみが減衰するのではなく、距離に比例して波長の短いものから
徐々に減衰していくのではないでしょうか?

逆に水蒸気やチリが赤色の散乱を起こすとすると
そのメカニズムはどういうものでしょうか
0602ご冗談でしょう?名無しさん垢版2020/05/20(水) 10:12:33.48ID:???
朝焼けと夕焼け赤色青色とかの現象論で解釈しようとする人は物理に向かない。雑学向き

物理学の基本は物理量同士の数量関係を記述してから物理説明するからその現象全てが
一つの物理理論で説明できる訳でもない。
0603ご冗談でしょう?名無しさん垢版2020/05/20(水) 10:49:06.17ID:kPz8ygG4
>>602
個人的には向く向かないということにはあまり興味がありません
説明は様々な概念レベル・スケールで行うことが可能です
細かい数式を理解するにも大雑把な理解が助けになることもあります
空がなぜ青いかも「レイリー散乱」という概念である程度は理解できます
(「概念」というのも極めて人間的な理解の方法です)

数量関係を記述すれば良いというならシミュレーションとコードを
見せてはい説明と言えばいいことになります
それはただの逃げではないでしょうか
0604ご冗談でしょう?名無しさん垢版2020/05/20(水) 11:16:08.28ID:???
国家プロジェクトとしての英才教育施設みたいなのが無いのは何故?

量子力学や相対論が、鶴亀算に比べて概念的にとくべつ難しいとは思わない。
理解するための前提知識が多いから難しいと感じるだけだ。
つまり、英才教育をすれば小学生でも理解可能ということ。

だから幼稚園児とかの時点で施設に集めて泊まり込みで洗脳し勉強させれば、いわゆる「早熟な天才」は量産可能なはず。
12歳で量子力学の教科書を書いた少年がいたが、あれ以上の逸材を作るのも容易だろう。
ではなぜ、そういった施設がないのか?
人権の問題があることは分かるが、中露は躊躇しないだろうしアメリカだってやっててもおかしくない。
しかし、そのような話は寡聞にして聞かない。

早熟な天才がその後も天才か、というのは確かに疑問だが様々なデータが示しているように早熟な天才はその後の人生においても成功を収めることが多い。
人権の問題はあるにしろ、人類の発展のためにこのような施設は是非とも必要なのではないだろうか?
むしろ、目先の人権のために発展の速度を遅らせるというのであれば、それは愚かであり、統治者としての義務を怠っているとさえいえる。
0606ご冗談でしょう?名無しさん垢版2020/05/20(水) 12:15:20.15ID:kPz8ygG4
>>590のような質問に日本語でスッキリ答えられないとすると
その原因はひょっとすると
空の色の物理現象についてあまり考えたことがない
空の色の物理現象についての既存の理論について理解が不十分
空の色の物理現象についての既存の理論が不十分・間違っている
空の色の物理現象について完璧に理解しているが言語化が出来ない
などではないでしょうか?

ほぼ完璧に理解している上に言語化して説明も出来るという方がいれば
大尊敬します
理解は不十分だけど一緒に考えようという人がいれば友達になれるでしょうw
0607ご冗談でしょう?名無しさん垢版2020/05/20(水) 13:13:08.70ID:???
水蒸気(と書いているが実際は水滴のことだろう)やチリによる散乱はミー散乱となり、
レイリー散乱とは波長依存性が異なる
なのにレイリー散乱の波長依存性だけで考えていればわけわからなくなって当たり前。
それぞれのメカニズムを知りたければレイリー散乱やミー散乱そのものを調べればよかろう。
自分の勉強不足を他人の説明不足のせいにするな
0608ご冗談でしょう?名無しさん垢版2020/05/20(水) 13:22:27.84ID:kPz8ygG4
>>607
大して知識がないから質問してるんですよw
返ってくる答え(?)に関してはそれは違うんじゃないですかと
再質問してるだけです
それを「勉強不足」と返しても何の意味もないと思いませんか?
0609ご冗談でしょう?名無しさん垢版2020/05/20(水) 13:26:15.80ID:kPz8ygG4
「レイリー散乱」「ミー散乱」を調べればよかろうというのは
何の説明にもなっていません
おそらくご本人もよく分かってないんではないでしょうか
0611ご冗談でしょう?名無しさん垢版2020/05/20(水) 13:31:10.78ID:kPz8ygG4
それが何か関係ありますか?
またそれは何を根拠に測定されたのでしょうか
科学的態度に問題があるんじゃないでしょうかw
0613ご冗談でしょう?名無しさん垢版2020/05/20(水) 13:44:20.91ID:???
質問に対して何を調べればわかるかを答えるのも立派な回答だろ
ここは子供電話相談室じゃないんだよ
理解する気のないやつにこれ以上ボランティアで付き合う義理もねぇ
0614ご冗談でしょう?名無しさん垢版2020/05/20(水) 13:50:02.72ID:kPz8ygG4
少なくとも現時点では私の質問には満足に答えられていないわけですから
私より賢いと主張するのは無理があるでしょうね
別に喧嘩を売る気はありませんが
分からないなら分からないで構わないのに
さも分かってる風を装って他人をバカにする人は私もバカにします
0615ご冗談でしょう?名無しさん垢版2020/05/20(水) 14:04:28.30ID:OqIRmPqR
物理学掲示板群 ttp://x0000.net/forum.aspx?id=2

学術巨大掲示板群: アルファ・ラボ ttp://x0000.net
物理学 化学 数学 生物学 天文学 地理地学
IT 電子 工学 国語 方言 言語学 など

ペンローズの量子脳理論
ttp://x0000.net/topic.aspx?id=3702-0
0616ご冗談でしょう?名無しさん垢版2020/05/20(水) 14:17:19.28ID:???
>>606
>空の色
個人の視覚から一般化した概念 可視光線の一定波長と強度に対応しない
科学的なモデルでみれば可視光線のスペクトル分布と個人の脳神経網の情報処理など
多数の複雑な現象で(補色も見える)色彩学の分野ともいえる物理学だけで説明出来ない。
>ほぼ完璧に理解している上に言語化して説明も出来るという方
存在しない。
物理学の議論や理論には物理量の再現性が必須だからその様な個人体感表現は排除しなければ議論にならない。
物理のテーブルに乗せたければ簡潔化して
例えば、光線波長が650nmと450nmで窒素ガスの散乱を計算する理論と方法の問題になる。
0619ご冗談でしょう?名無しさん垢版2020/05/20(水) 15:01:22.67ID:kPz8ygG4
>>616
確かに色の知覚は脳で起こる現象で物理現象だけでは記述できませんが
いわゆる平均的な色の見え方というのは標準化されていますから
(例えば色度計として測定装置に標準的な知覚特性が組み込まれています)
青とか赤とか述べても話が通じるのでそこは本質的な問題ではないと思います

知覚取扱不可能論は数量関係至上論と同じ逃げだと思います
別に神の如く現象を全て正確に捉え切るというのではなく
だいたいこういうことが起こっていると理解できればいいのです
0620ご冗談でしょう?名無しさん垢版2020/05/20(水) 15:40:10.96ID:???
>>590
1) 正しい
2) 地平線方向に太陽がないから。
赤より青の方が大きく散乱するから
太陽から目に直進する光は赤が多くなり
太陽以外の方向から来る光は青が多くなる(青空)
3) 逆
太陽に近い方向から来る光は離れた方向より赤が多いので
逆になるが、直進方向と比べて微々たる差だから気づきにくい
グラデーションは宇宙空間の方が観測しやすい

散乱の原因は大気の屈折率変動と大気中の微粒子がある
青空・夕焼けの原因は大気密度変動による屈折率の方
水蒸気の存在は屈折率に大きく影響する
0622ご冗談でしょう?名無しさん垢版2020/05/20(水) 16:10:05.75ID:???
>>619
>だいたいこういうことが起こっていると理解できればいいのです
>>620
分かり易い文章なので使わせてもらうと
>赤より青の方が大きく散乱するから
>太陽に近い方向から来る光は離れた方向より赤が多い
>太陽以外の方向から来る光は青が多くなる
何かの物理理論からの結果だけを並べているだけ。
0624ご冗談でしょう?名無しさん垢版2020/05/20(水) 16:54:31.21ID:???
>>599
まず昼間の空が青いのは遥か上空でレイリー拡散が起こり、それが青色として地上に届いてるから
夕刻は太陽が大気に対して斜め(長い距離)に進みため、青い波長の光は減衰してしまう
結果、減衰しにくい赤い波長が届く

でも夕刻と朝方ではチリや水蒸気の量が違い、減衰量に違いだ出る
朝方は澄んでいて減衰が起こらない=上空で起こるレイリー拡散の青が届きやすい
夕刻はチリ水蒸気が舞っていて減衰が多い=青が届きにくい

ミュー拡散の話をしてる奴がいるけど、雲が白いとかそういう話だからここでは関係ないと思う
0627ご冗談でしょう?名無しさん垢版2020/05/20(水) 18:16:37.79ID:???
この際だから流れに乗って>>624に質問したいんだけど地球大気レンズ球効果(プリズム効果)みたいなのってまったくの無関係?
0628ご冗談でしょう?名無しさん垢版2020/05/20(水) 18:28:36.08ID:???
>>624
>でも夕刻と朝方ではチリや水蒸気の量が違い、減衰量に違いだ出る
レイリー散乱は散乱体の大きさが波長に比べて無視できるほど小さいときの話。
波長程度より大きくなればミー散乱になる。
分子による散乱なら疑いなくレイリー散乱だが、チリや水蒸気も話題にしているときに
その大きさについて何も言及せずにレイリー散乱だけで論じるのはおかしいだろ
0629ご冗談でしょう?名無しさん垢版2020/05/20(水) 18:41:06.69ID:kPz8ygG4
>>620
ありがとうございます
簡潔かつ要領を得た答えですね

1)は概ねこの考えでいいようですね
2)、3)については自分がその後考えた結論とは少し違います
私は以下のように考えました

2)昼間に地平線方向が赤く見えないのは見えている範囲が狭いから

地表のある場所から地平線によって切り取られる空の範囲は限られているため
地球の端の方(経度90度先)で起こっている赤色の空まで視界に入らない
(光ファイバーなどを通して無理やり見れば赤く見えると思います)
またレイリー散乱によって生じた空の青さは遠方でも周波数が偏位する
ことはないので距離を考慮に入れる必要はない(青空も大気を長く通れば
赤色に偏位するかと疑ってました)
したがって見えている範囲で赤い空を見ることが出来ないというものです
(いかがでしょうか?)

3)朝夕の空は虹色のグラデーションの様にずばり「見える」

「レイリー散乱」のwikiページの写真には見事な虹のグラデーションが
写っています
https://ja.wikipedia.org/wiki/%E3%83%AC%E3%82%A4%E3%83%AA%E3%83%BC%E6%95%A3%E4%B9%B1
(逆になるというのはイマイチよく分かりませんでした)
0630ご冗談でしょう?名無しさん垢版2020/05/20(水) 18:43:03.71ID:kPz8ygG4
>>624
「減衰しにくい赤い波長が届く」というのが曖昧で分かりにくいというのが
質問の趣旨の一つでした

また青い光の届く量の違いだけで朝夕の違いが説明できるということですね
それはそれでありのような気もしますね(ただ下に説明とは違うようです)
0631ご冗談でしょう?名無しさん垢版2020/05/20(水) 18:43:51.88ID:kPz8ygG4
>>626
これ完璧な答えではないでしょうか

「明け方や夕方も日中と同じようにレイリー散乱による青色光の散乱が
起こっている。しかし,日中と違うのは,図 5 に示すように,太陽光が
地表に届くまでに大気の中を通り抜ける距離が長いという点である。

青色光は大気圏に入るとすぐ散乱されるため,長い距離を進み散乱が
繰り返されるうちに減衰し,我々の目に届く前に失われてしまう。
その結果,残った赤色光が目に届くことになるが,大気中に存在する
水蒸気や塵によってミー散乱が起こることで太陽だけでなく周りの空も
赤く染まる。また,ミー散乱を起こすサイズの大きな粒子は低層に沈んで
いることが多く,上空には少ない。このため,日中の青空のように空全体に
赤色が広がるわけではない。

一方,朝焼けと夕焼けでは, 夕焼けの方が赤色が強烈なイメージがあるかも
しれない。 日中は人間の活動や水分の蒸発も多く,大気の対流も盛んである。
それゆえ,明け方よりも夕方の方が散乱を起こす大気中の粒子が多く,
サイズも大きいため,西の彼方から 大気中を長距離進んできた赤色光が
より強く散乱される。」

ミー散乱も関係するみたいですね
0637ご冗談でしょう?名無しさん垢版2020/05/20(水) 21:27:12.14ID:kPz8ygG4
僭越ながら自分なりに皆さんの回答を採点してみましたw

よい回答(形式が整っている・正解が含まれる)80点
620、626

ふつうの回答(議論として成立・情報が含まれている)60点
594、595、596、598、600、607、624、628

だめな回答(無意味な還元論・知覚論・意味不明)40点
602、616、622

ゴミ(無内容・スレ汚し)0点
610、613、618、621、623

以上回答の全ヴァリエーションが含まれていたような気がします
「ふつうの回答」以上は普通に有益です(ありがとうございました)
興味深いのは「だめな回答」でこれよくある(頭の悪い人などが)
陥りやすいパターンではないでしょうか
今後の参考にして下さい
0639jdn垢版2020/05/20(水) 21:35:18.92ID:???
>>631
赤い色の夕焼けって空全体についてるよな?
昼間の青い空ってレイリー散乱だよな?これも空全体に色がついてるよな?
あれ?夕方の空って赤色散乱してるじゃん?
赤色散乱してなかったら空全体赤くならないよな?散乱する青色なくなったら散乱できないから宇宙の黒色が見えるよな?太陽昇ってるのに夕方空黒いはずだよな?
あれ?じゃあ空気って青も赤も散乱するのか?んなら昼間ってピンクの空になるよな?なんで空が青いんだ?
青が散って赤だけ残って空にスクリーンのように映る?なら空はスクリーンだと仮説を立てよう
昼のスクリーンは散乱した青を映す?夕方のスクリーンはなにを映すんだろうな?
こう考えられんか?スクリーンが映し出してるの今入射してる光だと?
さて虹ってどうやってできるんだろうな?水のタマがプリズムになって光が分裂するんだったよな?大きな水のタマがあれば大きなグラデーションができるな
ところで質問だ
地球ってのは大きな空気のタマか?それともただの青いボールか?
グラデーションってのは綺麗だよな。映画で見ると感動してしまうよな。
そういうことだ
0640jdn垢版2020/05/20(水) 21:38:10.07ID:???
遅かったな
0641ご冗談でしょう?名無しさん垢版2020/05/20(水) 22:30:50.12ID:kPz8ygG4
>>639
「だめな回答」w
0645ご冗談でしょう?名無しさん垢版2020/05/20(水) 23:24:07.78ID:B7MW2bt6
アホすぎる質問で恥ずかしいのですが、どなたかお願いします
帰宅後にスマホをアルコールで消毒するのですが、その直後に充電を始めます
ときどき電化製品のケーブルを差し込むときにパチッと火花がとぶことがありますが、あれが万一起こった場合、スマホにアルコールがついていると危険ですか?
引火する可能性はありますか?
0646ご冗談でしょう?名無しさん垢版2020/05/20(水) 23:48:31.14ID:???
あります
というか火花が散るプラグ又はコンセントはどこかおかしいので使うのをやめた方が良いのでは?
0647ご冗談でしょう?名無しさん垢版2020/05/21(木) 00:21:15.26ID:???
>>645
直流電源が入ってる機器をスイッチON状態でプラグを刺すとコンデンサーチャージ
で瞬間的に大電流が流れるため火花が飛ぶ。
アダプタ電源などはスイッチが無いので火花を飛ばしたくなければスイッチ付きの
電源タップを使えばよい。
0648ご冗談でしょう?名無しさん垢版2020/05/21(木) 01:25:38.32ID:HsBgcWzU
凄く知りたいのですが、
とてつも無く長く丈夫な棒を動かしたら光速を超えるのでしょうか?
※指で空をなぞっていて気になりました
0649ご冗談でしょう?名無しさん垢版2020/05/21(木) 01:59:33.03ID:???
>>648
これよく言われるけど、棒使わなくても単純に宇宙の中心で回ってる星からみて宇宙の果てか壁が光速より遥か速さで回ってるように見えるなって今想像してみて思った
宇宙の果ても相対速度の範疇なのかね
何億光年先の星もそうだろうし、観測してみたり理論考えてみた人っているのかな
0650ご冗談でしょう?名無しさん垢版2020/05/21(木) 02:13:57.17ID:???
>>648
超えません
実際の棒は原子が繋がって出来てます
棒が動くというのは、端っこの原子を動かしたらその動きが隣へ隣へと繋がっていく現象のことです
動きの伝わるスピードは光速を超えることはありません

>>649
そもそも、遠くにあるものの速さを測るというのはどのようなことなのでしょうか
それを突き詰めると一般相対性理論にたどり着きますね
速度をどのように定義するかによって、光速を超えるかどうかの結果が変わります
0651ご冗談でしょう?名無しさん垢版2020/05/21(木) 02:20:44.51ID:HsBgcWzU
>>650
棒などの物質は、光速を超えられないという事なのですか
ありがとうございます、すっきりしました
0652ご冗談でしょう?名無しさん垢版2020/05/21(木) 02:24:56.27ID:???
>>650
単純に図面引くとか
幾何的に外周へ行くほど距離が長くなるんだから回転すると自分を静止として相対的にその距離を遠くの星が走ったように見える

まあ見えるだけか
0653ご冗談でしょう?名無しさん垢版2020/05/21(木) 02:43:06.06ID:???
周りの星がついてくるように(光速限界)空間と時間が歪むなら宇宙の回転する全星合わせてどんだけ複雑な時空歪曲しなければならないのか

まあそんなことも当然のことなんだろうけど
0661ご冗談でしょう?名無しさん垢版2020/05/21(木) 04:01:05.53ID:???
>>660
スカイツリーくらいならスカイツリーに時計置けばいいですけど、宇宙の果ての速さを測るときは宇宙の果てに時計を置くわけにはいきませんよね
0664ご冗談でしょう?名無しさん垢版2020/05/21(木) 09:40:18.27ID:???
>>662
>光子時計 ->  光格子時計
現実の宇宙の時計を簡単に説明すれば
個々の観測者の時間(局所時間)とは観測者と一緒に運動する原子の一定振動から出る
光の振動数のに基づいている。それをカウントした値で時間経過を比較できる。
>>661
量子論によれば同一の原子は完全に同一の振動であるから宇宙の果てでも原子(さらに素粒子のスピン)
があれば局所時間が存在し時間経過している。
0667NAS6 ◆n3AmnVhjwc 垢版2020/05/21(木) 11:57:19.46ID:???
頭悪いからだろ
光を測るのに光を使うから光速度不変
いわゆるドップラー効果しか分からない
音しかわからんコウモリにしたら音速不変なだけだろ
0668NAS6 ◆n3AmnVhjwc 垢版2020/05/21(木) 12:03:52.78ID:???
物体に対して伝搬速度Aの検出法をつかったら検出法の速度を超えて観測出来ないだけですよ
0669NAS6 ◆n3AmnVhjwc 垢版2020/05/21(木) 12:15:18.84ID:???
目の錯覚物理
光のドップラー効果、相対論
音の錯覚物理
音のドップラー効果
0672NAS6 ◆n3AmnVhjwc 垢版2020/05/21(木) 12:21:19.71ID:???
例えば銃で撃たれたのは分からんかもしれないけど
腹に穴が開いてるのは分かるだろみたいな世紀末救世主がそんなセリフ言ってただろ
0673NAS6 ◆n3AmnVhjwc 垢版2020/05/21(木) 12:23:08.73ID:???
670
メトロノームで音時計でも作れよ
同じことだろ
0674NAS6 ◆n3AmnVhjwc 垢版2020/05/21(木) 12:26:20.31ID:???
ソナーは実際に周波数が変化してるけど
光で分かる奴には錯覚なんだよ
0676NAS6 ◆n3AmnVhjwc 垢版2020/05/21(木) 12:36:24.79ID:???
653
だからどこかで回転している奴のために全宇宙が合わせて光速度で動くよてアホか
それは錯覚であってだな
夜振り向いたら遠くの星が超光速で動いたことになるけど
そいつの錯覚なだけ
0679NAS6 ◆n3AmnVhjwc 垢版2020/05/21(木) 12:48:33.13ID:???
一光年先の星をみて百八十度振り向いたら二光年動いた
振り向くのに五秒だから超光速
なんてそいつの見た目の錯覚だけだろ
0680NAS6 ◆n3AmnVhjwc 垢版2020/05/21(木) 13:12:19.06ID:???
自己中な傲慢な奴には理解出来ないかもな
お前が回転しただけてのが
0681NAS6 ◆n3AmnVhjwc 垢版2020/05/21(木) 13:22:19.77ID:???
量子力学のなんちゃら解釈やらシュレ猫やらは観測したから確定する
池沼の頭のなかは欧米にエベレストが見つかるまで世界最高峰はエベレストじゃないらしい見つかる見つからないに関わらず決まってることが理解出来ないらしいな
0682NAS6 ◆n3AmnVhjwc 垢版2020/05/21(木) 13:30:21.70ID:???
シュレ猫なんて箱の中の事情をしらない観測者からみたら生と死が重なった池沼説明だろうが
猫本猫からしたら生きていて死んでる重なった状態なんてまるきりありませんわ
0683ご冗談でしょう?名無しさん垢版2020/05/21(木) 13:39:50.43ID:???
>>679
>一光年先の星をみて百八十度振り向いたら二光年動いた
>振り向くのに五秒だから超光速
その様な回転運動する観測座標系であっても「一般相対性理論」では光速不変では
ないから成り立つし、論理矛盾も無い。
>なんてそいつの見た目の錯覚だけだろ
それはNAS6が「一般相対性理論」ではない自己の座標系による判断か
 普通の一般人ならば慣性系が絶対基準のニュートン力学または
相間でないが特殊相対性理論の立場の判断ならば超光速運動とは認めない。
0684NAS6 ◆n3AmnVhjwc 垢版2020/05/21(木) 13:40:26.07ID:???
事情をしらない野次馬博士の証言より
被害猫の証言の方か分かりやすい
0686NAS6 ◆n3AmnVhjwc 垢版2020/05/21(木) 13:47:39.19ID:???
だからさ一光年先の星の運動は観測者の観測に全く関わりないの
もし見えたならそう見えただけ
0687ご冗談でしょう?名無しさん垢版2020/05/21(木) 13:47:57.42ID:ocugDxoz
物理学掲示板群 ttp://x0000.net/forum.aspx?id=2

学術巨大掲示板群: アルファ・ラボ ttp://x0000.net
物理学 化学 数学 生物学 天文学 地理地学
IT 電子 工学 国語 方言 言語学 など

ペンローズの量子脳理論
ttp://x0000.net/topic.aspx?id=3702-0
連続と離散を統一した!
ttp://x0000.net/topic.aspx?id=3709-0
0688NAS6 ◆n3AmnVhjwc 垢版2020/05/21(木) 13:52:17.36ID:???
一光年先の星の運動は観測者の観測座標系とは無関係ですよ
0689NAS6 ◆n3AmnVhjwc 垢版2020/05/21(木) 13:56:49.47ID:???
相対性原理も
観測者の観測座標系と
一光年先の星の運動系に
優劣はないなんて言いそうで池沼丸出し
シュレ猫と同じ
傲慢
0690ご冗談でしょう?名無しさん垢版2020/05/21(木) 14:06:34.07ID:???
>>683
現代でも「一般相対性理論」で恒星の日周運動を見ている人は世界中に0.01%もいないだろが
面白いことに事に近代化以前は天動説があたりまえで99.99%の人が疑問を持たなかっただろう
現代と中世以前で比率が逆なのはニュートン力学の刷り込み学校教育の成果といえる。
将来、義務教育で「一般相対性理論」が必須科目になれば再逆転するだろうが
学生の知能が付いていけないのでならない。
0691NAS6 ◆n3AmnVhjwc 垢版2020/05/21(木) 14:16:41.39ID:???
つか683は回転変換も知らずに相対論やってんの?
原点からx=1を見て
百八十度回転したらx=-1だよな
それがいまは光年単位の話で二光年動いたろ
回転するのに五秒だから超光速
これは観測者の観測座標系の話
一光年先の星の運動系はそんな観測座標系は知ったこっちゃない
0692NAS6 ◆n3AmnVhjwc 垢版2020/05/21(木) 14:32:41.88ID:???
量子力学でも相対論でも
お前がそう見えたからって
お前が見た対象がなんで実際にそう振る舞わなきゃならんのよ
まず自己中の池沼脳を先になんとかしてくれ
てわけ
お前の見る見ないと
見る対象はほとんど無関係なんだわ
とかく物理になるとそう見えたからて言い出す
0693ご冗談でしょう?名無しさん垢版2020/05/21(木) 14:34:41.64ID:???
>>691
太陽系から6光年のバーナード星系の惑星の知的生物は誰でも一般相対性理論が理解でき
太陽系が超光速で動くのが当たり前と習ってるかもな。
0694NAS6 ◆n3AmnVhjwc 垢版2020/05/21(木) 14:38:28.96ID:???
お前がアイドルが愛してるて歌ってんのを聞いても
お前とアイドルは関係ないの
シュレ猫の博士ってこの意味分かるの?
0695ご冗談でしょう?名無しさん垢版2020/05/21(木) 14:42:56.01ID:???
>>629
その写真では太陽に近い方向は赤く、離れた方向は青い
ならば太陽が上にあれば、上が赤く地平線が青い
それが「逆」の意味
0696ご冗談でしょう?名無しさん垢版2020/05/21(木) 14:55:53.16ID:???
>>694
物理的に完全に隔離されてれば、関係ないのは事実。
ファンは当然関係付けようと電磁気的通信で相互作用を図るだろ
それでもダメなら直接握手会とかで接触による相互作用を計画するだろ
それでも相互作用が足らないキチガイは犯罪行為で広範囲な相互作用を計画する。
この例えがオマエにわかるか?
0698ご冗談でしょう?名無しさん垢版2020/05/21(木) 15:15:39.57ID:???
>>697
スタートレックの光子魚雷は実在する核魚雷がヒントだろう。
核魚雷の水爆弾頭ではなく物質・反物質爆弾にすれば殆どの爆発エネルギーは光子になる。
0699NAS6 ◆n3AmnVhjwc 垢版2020/05/21(木) 15:30:41.57ID:???
いやアイドルは池沼だし興味ねーわ
それよかアイドル満足させられるほど
おらに甲斐性ねーわ
低みの見物している身としては異性はコスパ重視
0700ご冗談でしょう?名無しさん垢版2020/05/21(木) 16:03:17.02ID:???
シュレディンガーの猫という誤用されまくる思考実験

量子力学を認めなかったシュレディンガーの呪いの言葉なのに
0702NAS6 ◆n3AmnVhjwc 垢版2020/05/21(木) 16:21:06.73ID:???
ん?だから
観測問題で観測したから有るって分かる
って話だろ
お前にあわせて有ったり無かったりする素粒子なんて妄想だろ
0704ご冗談でしょう?名無しさん垢版2020/05/21(木) 16:32:46.19ID:Vdug/Tty
>>695
ありがとうございます
そういう意味なんですね

その場合ひとつ>>631でも自分の中で解決していない問題があります
夕日の青色光は大気圏に入るとすぐレイリー散乱され(そして減衰され)
残った赤色光は大気の低層でミー散乱され太陽近傍を赤くします
その時、空の青と赤の間は虹色のグラデーションを作りますが
緑色光は大気のどの辺の位置で何散乱されて見えるのでしょうか?
「大気中層の比較的小さいゴミ・水滴によるミー散乱が赤より広い領域に広がる」と考えていいのでしょうか
0707ご冗談でしょう?名無しさん垢版2020/05/21(木) 17:00:21.02ID:Vdug/Tty
>>706
ありがとうございます
こういう現象もあるんですね!

ただ私が言っているのは一瞬の現象ではなく
下の写真のように持続して出現する緑色光(虹色グラデーション)のことです
https://upload.wikimedia.org/wikipedia/commons/thumb/5/5e/SDIM0241b.jpg/800px-SDIM0241b.jpg

うっすらと緑色の帯が青とオレンジ(黄)の間に水平に見えていますよね
これは空が赤みがかってから消えるまでずっと見えていると思います
これがどうして見えるのかということです
0708ご冗談でしょう?名無しさん垢版2020/05/21(木) 17:23:09.40ID:Vdug/Tty
ちなみに太陽光の大気への入射による屈折(プリズム作用)によって
虹色のグラデーションが起こるとすれば
これとは上下の向きが逆だと思います
0712ご冗談でしょう?名無しさん垢版2020/05/21(木) 17:57:07.58ID:Vdug/Tty
仮に緑色が単独ではっきり認められない場合でも
赤がオレンジから黄へのグラデーションになっていますから
少なくとも緑色光成分が赤と青に間に存在することになります
0714ご冗談でしょう?名無しさん垢版2020/05/21(木) 18:15:49.22ID:???
>>711
フォトショで開いてサンプルしたけどGのピークはほぼ無かったよ

予想だけど
太陽光には全ての色の波長がある
昼はレイリー散乱で青は強く見える
夕刻は大気を進む距離が長いので青色が段々と減衰する
青と緑と赤が釣り合った場所は合成色で白く見える
青が減衰すると残る波長は緑と赤
だから白の次は緑と赤の合成色である黄色が見える
そこから緑が減衰して最後は赤だけが残る

こんな感じではなかろうか
0716ご冗談でしょう?名無しさん垢版2020/05/21(木) 18:24:35.93ID:Vdug/Tty
>>714
その可能性もありますね
ただしそに場合も緑が何故そこにあるのかは説明が必要です
青(レイリー散乱)と赤(ミー散乱)の散乱は説明されていますが
緑の散乱は説明がなされていません
散乱がなければ光線は全て太陽の円の中に収まります
0718ご冗談でしょう?名無しさん垢版2020/05/21(木) 18:36:27.44ID:Vdug/Tty
考えられる説明は以下などです
1)青・緑・赤は全てこの順番の強度(広さ)でレイリー散乱している
2)緑は大気の中層で小さいチリにより広く、赤は大気の低層で大きいチリにより
狭く(緑に比べて)ミー散乱している

どちらも起こっているとしても、どちらがより大きいのかという疑問も湧きます
0719ご冗談でしょう?名無しさん垢版2020/05/21(木) 18:43:42.34ID:Vdug/Tty
>>706
「グリーンフラッシュ」に関する記事のリンク先の英文記事の一部が
イマイチ理解できません
https://www.popsci.com/green-flash-sunset/

At sunset, colors fade at varying rates. Just before the sun vanishes,
the red light’s shallower ripple causes it to shoot overhead and miss
your eyes. Green, with its steeper wavelength, remains the sole color
survivor, if only for a second.

「the red light’s shallower ripple causes it to shoot overhead」
これどういう意味でしょうか?
0721ご冗談でしょう?名無しさん垢版2020/05/21(木) 19:06:02.96ID:???
単に、赤い光の波はあまり屈折せず、頭上を飛んでいくってことじゃない?
ttps://www.animations.physics.unsw.edu.au/jw/light/mirages-green-flash-sky-colours.htm#3
0723ご冗談でしょう?名無しさん垢版2020/05/21(木) 19:17:32.00ID:???
>>718
自然光は青の波長だけ、赤の波長だけってことはなく
緑の波長も混じってそれが乱反射してる
白く見えるのはその証拠(緑の成分がなければ白が見えない)

緑を含まない光を作ろうとすると単一波長レーザー

青く見えるレイリー拡散だって青のピークが高いだけで乱反射して色々な成分を含んでる
プリズムみたいな分光の原理とは違って綺麗に中間色が出るわけじゃなく、見えるのは合成光
0724ご冗談でしょう?名無しさん垢版2020/05/21(木) 19:33:32.23ID:Vdug/Tty
>>723
分かります
程度問題ですよね
しかし疑問は緑色光の散乱はレイリー散乱が大きいのかミー散乱が大きいのか
ミー散乱の場合それは大気のどの辺で起こるのかという点にあります
ここが未だに曖昧です
0725ご冗談でしょう?名無しさん垢版2020/05/21(木) 20:00:37.75ID:???
>>724
ミー散乱は透明であるはずのものが白く見える現象だよ
緑はこの際関係ないかな

緑はレイリー散乱でピークを出す物質があるわけではなく大気による短波長光の減衰だと思う
0726ご冗談でしょう?名無しさん垢版2020/05/21(木) 20:06:08.49ID:???
なぜか夕焼け論議が伸びてる
5chレベルでも寺田寅彦一門の現象論的な情緒物理が日本人は好むようだ
だれも現実のスペクトル分布を持ち出さず個々に勝手な推測してるだけ。
複雑な散乱吸収を数値解析するための光散乱のモンテカルロ法も知らない
時代遅れのアホども
0727ご冗談でしょう?名無しさん垢版2020/05/21(木) 20:15:01.03ID:Vdug/Tty
>>725
緑色光においてレイリー散乱は主に減衰に関わっているであろう
というのは理解できます

一方ミー散乱が白くするという説明は少し違うのではないでしょうか
ミー散乱は波長依存性が低いというだけで来た光を散乱させるだけです
(全波長の光があれば結果として白く見えるだけ)
夕空が赤いのは減衰せずに残った赤色光がミー散乱しているから
理解しています

おそらくきれいな空気の大気下層側では少し弱くなった緑とあまり弱くなって
いない赤がミー散乱しており
緑の方が広く拡散するのでグラデーションが生じるという説明なら理解できます
この場合もなぜ緑の方がより広く散乱するのかは説明が必要です
0730ご冗談でしょう?名無しさん垢版2020/05/21(木) 21:01:30.17ID:???
例えばだけどR,Gは反射しないけどBだけは反射するミラー(3パターン全部)
みたいな理論の上では存在するけど工学的にはまず作れないものってどんなのあるの?
その中でも無理やりにでも作れれば科学が進むようなのは単純なアイテムでも結構ありそうなイメージだけど
0731ご冗談でしょう?名無しさん垢版2020/05/21(木) 21:27:54.27ID:Qd9M9+Nn
大学一年生で運動方程式までやりました
x、y軸を取りt=0に(0,ho)の場所から質量mの小球を初速度vo=(vo,0)
で発射した。小球に働く重力をF=(0,ーmg)とする。

1 x、y座標系での小球の運動方程式をもとめよ
2 x、y成分について一般解を求めよ
3 初期条件を入れて任意定数の値を決定せよ
4 x、y座標系と平行なx’、y’座標系は小球の初速度と同じvoで動いていて
  t=0で2つの座標系の原点は一致したときx’、y’座標から見ると小球の
  運動はどのようにみえるか
5 x’、y'座標系での小球の位置ベクトルをr’(t)=x’(t),y’(t)として
  運動方程式を求めよ
6 5の運動方程式の解から4が正しいことを確認せよ
0732ご冗談でしょう?名無しさん垢版2020/05/21(木) 21:28:54.70ID:Qd9M9+Nn
1番目から分からないので全て教えてほしいです
0733ご冗談でしょう?名無しさん垢版2020/05/21(木) 21:40:14.90ID:Qd9M9+Nn
ma=ーmgになりましたが一般解がわかりません
0734ご冗談でしょう?名無しさん垢版2020/05/21(木) 21:43:42.28ID:???
微分方程式の解き方はわかりますか?
簡単にいったらaを積分してやるとvになるし
さらに積分するとxとかyになります
0735ご冗談でしょう?名無しさん垢版2020/05/21(木) 21:52:43.16ID:Qd9M9+Nn
微分方程式はやりました
0736ご冗談でしょう?名無しさん垢版2020/05/21(木) 21:54:35.62ID:Qd9M9+Nn
微分方程式使うんですか
全然分かりませんでした
0738ご冗談でしょう?名無しさん垢版2020/05/21(木) 22:12:18.30ID:Qd9M9+Nn
1番目はあってますよね?
2回微分を積分とはどういうことですか
加速度aが変位xの二回微分はわかります
3番目は問題の意味がわからなくて
どこに代入すればよいですか
0739ご冗談でしょう?名無しさん垢版2020/05/21(木) 22:19:23.34ID:Qd9M9+Nn
もしつながってなければ4以降おねがいします
0740ご冗談でしょう?名無しさん垢版2020/05/21(木) 22:22:49.15ID:Xm+X7nIn
(1)〜(3)は時間微分を「 ’ 」「 ’’ 」で表す。

(1)
mx’’=0
my’’=−mg

(2)と(3)
積分してx’=C、t=0ではx’=C=v0
さらに積分してx=v0・t+D、t=0ではx=0より
x=v0・t

積分してy’=−gt+C、t=0ではy’=C=0
さらに積分してy=ー1/2gt^2+D、t=0ではy=D=h0より
y=ー1/2gt^2+h0

(4)
y’軸の負方向にh0から自由落下しているように見える。

(5)
ガリレイの相対性原理より
x=x’+v0・tより、x’=x−v0・t=v0・t−v0・t=0
y=y’より、y’=ー1/2gt^2+h0
これが(4)である。

くっくっく
0742ご冗談でしょう?名無しさん垢版2020/05/21(木) 22:29:47.26ID:Qd9M9+Nn
>>740
ありがとうございます
相対性理論がないと解けないの
ですね
0743ご冗談でしょう?名無しさん垢版2020/05/21(木) 22:33:11.71ID:Xm+X7nIn
>>742
ガリレイのだぞ。何も難しくはない。
教科書にあるはずだ。
あのアホノシュタインの相対性原理とは違うからな。

くっくっく
0744ご冗談でしょう?名無しさん垢版2020/05/21(木) 22:37:08.24ID:cbRBAMpA
摩擦は無視できるものとする
物体Aと物体Bがあり質量をそれぞれm1、m2としたとき、物体1にFを与えたとき物体1から物体2に与える力はどうなりますか?
また逆に物体2にFを与えたとき物体2から物体1に
与える力はどうなりますか?

加速度をaとおいてそれぞれ運動方程式
をたてましたがf1が打ち消しあってしまいます
0745ご冗談でしょう?名無しさん垢版2020/05/21(木) 22:37:41.77ID:???
まじめに999の発言読んでる人がいるとも思えないけど
彼、相対論が間違ってるって言ってるように読める?
間違ってるって言おうとして結果的に相対論は正しいって結論を言ってるようにしか読めない場合が多いんだけど
0746ご冗談でしょう?名無しさん垢版2020/05/21(木) 22:45:03.94ID:Xm+X7nIn
「ガリレイの相対性原理」は教科書では
「ガリレイ変換」と書いている場合がある。実質的にはこの「ガリレイ変換」で覚えておけ。
 
正しくは「ガリレイ変換」のもとで物理法則は不変であるというのが「ガリレイの相対性原理」である。
この「ガリレイ変換」というのは、座標変換は単に相対速度の加減算で行うことが出来るというものだ。
式で表すと、座標rと座標r’のガリレイ変換は

r=r0+v・t+r’(すべてベクトル)

ただこれだけのことであり、極めて当たり前の変換である。
くっくっく
0747ご冗談でしょう?名無しさん垢版2020/05/21(木) 23:02:41.54ID:Qd9M9+Nn
運動方程式までしかやってないですが
そんな知識が必要になるんですか?
0748ご冗談でしょう?名無しさん垢版2020/05/21(木) 23:03:15.38ID:Xm+X7nIn
ガリレイ変換
r=r0+v・t+r’(すべてベクトル)

これを時間微分すると
dr/dt=v+dr’/dtとなって、速度の加減算になる。

さらに時間微分すると
d^2r/dt^2=d^2r’/dt^2となり、この両辺にmを書けると
F=F’となってニュートンの運動方程式はガリレイ変換で不変であることが分かる。

つまり、運動方程式は自動的にガリレイの相対性原理を満たす。
ガリレイの相対性原理を適用するまでもなく、自動的に満たすのだ。

電磁方程式にガリレイ変換を施し、
さらにガリレイの相対性原理を適用すれば光速は
C’=Cーvとなって見る者によって変わるという結果が得られる。
導出は面倒なので省略する。勝手に調べろ。

運動方程式は自動的にガリレイの相対性原理を満たすが、
電磁方程式はガリレイの相対性原理を適用しなければ座標変換後の式が
得られないのである。もちろん、アホノシュタインの相対性原理でも同じである。

このことを明確に意識できている人間は世界に100人もおらん。
アホザルばかりなのである。

くっくっく
0749ご冗談でしょう?名無しさん垢版2020/05/21(木) 23:06:27.02ID:Xm+X7nIn
>>747
ガリレイ変換
r=r0+v・t+r’(すべてベクトル)
は絶対に必要で、高校物理で教えるぞ。

これを時間で1回微分、2回微分した形をちゃんと覚えておけ。
速度にあり、運動方程式になるのだ。
「回」の漢字は割とどうでもいいがな。

書き方がベクトルかそうでないかの違いだけだ。
この関係式は必須だぞ。

くっくっく
0750ご冗談でしょう?名無しさん垢版2020/05/21(木) 23:07:14.92ID:Xm+X7nIn
× 速度にあり、
〇 速度になり、

くっくっく
0751ご冗談でしょう?名無しさん垢版2020/05/21(木) 23:19:52.09ID:Xm+X7nIn
「アホノシュタインの相対性原理」とは
光速を不変とするためのローレンツ変換のもとで
物理法則も不変である、というものである。

電磁方程式にアホノシュタインの相対性原理を適用すると
もう1つの前提条件である「光速不変」が結果として得られるが、
これは循環論法だから当たり前である。
それは、ローレンツ変換に最初から光速不変が組み込まれているからである。

ほとんどのアホザルはこの極めて簡単なトリックに気づいておらず
いまだにアホノシュタインの相対論は正しいと信じ切っているのだ。

まことに
アホザルばかりなのである。

くっくっく
0752ご冗談でしょう?名無しさん垢版2020/05/21(木) 23:37:21.52ID:Xm+X7nIn
あと、インターネッツ上のクソサイトやなんちゃって専門書で
電磁方程式を不変とすると逆にローレンツ変換が得られるというのがある。
だからローレンツ変換とアホノシュタインの相対性原理は正しいというアホザルの主張な。

これ、論理を反転してるだけのマヌケ論法だから
良い子はあざ笑ってやれ。
本当にコイツらはアホザルだ。

くっくっく
0753ご冗談でしょう?名無しさん垢版2020/05/21(木) 23:44:20.78ID:wf9DlH5Y
変位を時間で積分した値ってどういう意味をなすんでしょうか?
気になって夜しか眠れません。
0754ご冗談でしょう?名無しさん垢版2020/05/21(木) 23:47:35.21ID:Xm+X7nIn

摩擦は無視できるものとする
物体Aと物体Bがあり質量をそれぞれm1、m2としたとき、物体1にFを与えたとき物体1から物体2に与える力はどうなりますか?
また逆に物体2にFを与えたとき物体2から物体1に
与える力はどうなりますか?


m1・a=F−F’
m2・a=F’

よってF=(m1+m2)aよりa=F/(m1+m2)
F’= m2/(m1+m2)・F

逆にした場合なんて
記号を逆にするだけで無意味だぞ。

んでこれの何が難しいんだ?
さっぱり分からんな。

ああ、m1+m2というふうに
くっつけて考えていい根拠がほしいのか。

くっくっく
0755ご冗談でしょう?名無しさん垢版2020/05/22(金) 00:25:55.40ID:vHrM9kt5
傾きθの斜面の上に質量mの物体が静止しているときの
静止摩擦力はumgcosθですか?静止摩擦係数u重力加速度gとする
0756ご冗談でしょう?名無しさん垢版2020/05/22(金) 00:48:02.43ID:???
>>755
一種のひっかけ問題みたいだけど
静止してるということは力が釣り合って0になってるということです
つまり、物体にかかかっている重力の斜面方向の成分と、静止摩擦力が釣り合って0
0757ご冗談でしょう?名無しさん垢版2020/05/22(金) 00:49:52.91ID:vHrM9kt5
mgsinθか
0758NAS6 ◆n3AmnVhjwc 垢版2020/05/22(金) 03:38:11.61ID:???
夕焼け物理はフレネル反射を媒体中から見た事ですよ
透過光が赤で反射光が青
なぜかはそれぞれ電界と磁界の平行成分の振り分け
だから大気が赤で真空が青って事
散乱でRGBがどうということじゃない
0759ご冗談でしょう?名無しさん垢版2020/05/22(金) 04:07:25.44ID:Z/uqjDmr
>>721
ありがとうございます
>>719の英文の意味はそういうことみたいですね

ただリンク先の説明を読んでも
赤色光の方が屈折が少なくて頭上を通り越しているなら
日没とともに頭の位置まで下りてきて
緑よりも後まで見えているはずではないかと考えて
まだしっくり来ていません、、、


「For observer at point a, the red light from the sun does not bend enough
to be seen, so his/her image of the sun will have no red light―for this
observer, the red image of the sun has already set.
Blue and green light do bend enough (the blue and green suns have not
yet set!), but the blue light is scattered out by the large thickness of
atmosphere traversed by the light. So the image is green for a short
time―typically a second or two.」

なぜredはhas alredy setでblue and greenはhave not yet setなんでしょう
逆にように思えます
(何が間違っているのでしょう)
0760NAS6 ◆n3AmnVhjwc 垢版2020/05/22(金) 10:08:12.49ID:???
ttp://natsci.kyokyo-u.ac.jp/~okihana/kaisetu/kussetu.html
青色光はなぜ屈折率が高いのか

青色光は振動数が高いから媒体中で遅くてだから屈折率が高いんだと
0761ご冗談でしょう?名無しさん垢版2020/05/22(金) 10:47:38.98ID:???
>>751
相間くっくっくの文から間違いを探して頭を鍛えよう。

>「アホノシュタインの相対性原理」とは
の部分がすぐに可笑しいと気づけばキミは物理の素質がある。
正しくは「特殊相対性理論の相対性原理とは」になる。
>光速を不変とするためのローレンツ変換のもとで
>物理法則も不変である、というものである。
でほぼマトモな文章になる。

「相対性原理」とは相対運動による物理法則(方程式)が同じ形になるという一般的な原理原則。
・ニュートン力学では慣性系のガリレイの相対性原理(ユークリッド空間の平行移動)
・特殊相対性理論では慣性系の特殊相対性原理
・一般相対性理論では加速度運動を含む相対運動の一般相対性原理
・宇宙論ならばコペルニクス原理など

以上のどれも物理学の理論上の基本仮説だからどれが絶対真理とかの基準はない。
つまり現実の個々の物理現象を解析するのにどの理論が最適かという使用者側の基準になる。
0762ご冗談でしょう?名無しさん垢版2020/05/22(金) 11:48:27.51ID:???
>>761
理論的に計算が可能だとしても弾丸の放物運動を求めるのに
特殊相対性理論を使って解こうとするアホはいない。また
シュレディンガー方程式使って解こうとするバカもいない。
0764NAS6 ◆n3AmnVhjwc 垢版2020/05/22(金) 12:07:16.79ID:???
f=- mG(1+2.5(v/c)^2)
万有引力を
これで解けばシュワルツシルト解で解いた事ですよ
解き方に頭の良さが出ちゃうね
0765NAS6 ◆n3AmnVhjwc 垢版2020/05/22(金) 12:30:55.81ID:???
ガリレイ変換つかプログラミングでは
同次座標とアフィン変換
を実際使うんだよ
0766ご冗談でしょう?名無しさん垢版2020/05/22(金) 13:09:06.52ID:???
>>759
a点からみた現象としては赤い光が消えて行って緑の光が現れて消えるわけで、英語の文章が正しいことは分かる
太陽がそのまま沈んでいけば光はa点より右のほうに行くわけで、それをsetで表しているんだろう

太陽からの光を追跡するより、目から太陽をたどる方がわかりやすいかも
こっちは日本語だし
ttp://oguriculture.web.fc2.com/syowa/greenflash/greenflash.html
0767ご冗談でしょう?名無しさん垢版2020/05/22(金) 13:12:58.78ID:???
>>704
夕焼け・青空にミー散乱は関係ない
ミー散乱の原因となる微粒子が影響するほどだと綺麗な夕焼けは見れない
またミー散乱もレイリー散乱も弾性散乱だから減衰も関係ない
夕焼け・青空の原因となるレイリー散乱は特定波長の散乱ではなく
赤より青の方が散乱しやすいだけの連続的散乱
緑色光は赤と青の中間にあるだけ
0768ご冗談でしょう?名無しさん垢版2020/05/22(金) 15:20:30.80ID:Z/uqjDmr
>>766
ありがとうございます
記事の図を見てひょっとすると光源が大きく平行光なので
誤解が生じたのかなと思ったのですが
やはりこの図は変じゃないでしょうか

図のように目に全ての波長の光が集まるなら
色のズレは起こらず白い(黄色い)太陽が見えるはずです
実際は太陽の上端から発する光は下から青→緑→赤の順に見えて
やはり青→緑→赤の順に没していくと思います
(最後にレッドフラッシュが見えるはず)

英語の記事の「光はa点より右のほうに行く」が理解できません
左に行くのではないでしょうか
(太陽が登るなら右に行くと思います)
0769ご冗談でしょう?名無しさん垢版2020/05/22(金) 15:24:24.34ID:Z/uqjDmr
>>767
>>631読まれましたか?
この説明と全然違います
もしレイリー散乱だけで説明出来るなら
太陽が沈むにつれて空は虹色に変化するはずですが
そうはなりません
おそらく間違っていると思います
0770ご冗談でしょう?名無しさん垢版2020/05/22(金) 15:26:49.36ID:Z/uqjDmr
虹色に変化するとは青空と夕焼けの間の時間帯には
緑焼けのような現象が起きるはずです
(そうはなりません)
0771ご冗談でしょう?名無しさん垢版2020/05/22(金) 15:31:51.55ID:Z/uqjDmr
>>769
>>631というより>>626
0772NAS6 ◆n3AmnVhjwc 垢版2020/05/22(金) 16:28:45.70ID:???
770
速度で屈折率が違うから
夕方朝方の太陽の角度で青と緑が反射するけど
太陽の角度と屈折率の関係から青と赤が反射する場合はないんだろ
緑は中間だからさ
0773ご冗談でしょう?名無しさん垢版2020/05/22(金) 16:47:41.50ID:Z/uqjDmr
確かに緑焼けというのは変ですね
あえて言えば黄焼け(青抜け)でしょうか
でも多分違うと思いますよ
反射の話は>>767には出て来ません
0775ご冗談でしょう?名無しさん垢版2020/05/22(金) 17:17:08.63ID:Z/uqjDmr
固定観念というより現時点では>>626を基準に話しています
そこと違うと指摘してるだけです
むしろ>>626の間違ってる部分を指摘してください
0776NAS6 ◆n3AmnVhjwc 垢版2020/05/22(金) 17:59:25.99ID:???
760の京都大学のリンク見た?
簡潔に理解出来るよ
0777ご冗談でしょう?名無しさん垢版2020/05/22(金) 18:07:49.14ID:Z/uqjDmr
屈折率については、ですね
夕焼けとグリーンフラッシュの理解に関しては
そこまで詳細な議論は現時点で特に必要ないんじゃないでしょうか
0778ご冗談でしょう?名無しさん垢版2020/05/22(金) 18:12:53.83ID:Z/uqjDmr
また屈折率を理解しただけでは
夕焼けとグリーンフラッシュは理解できないと思います
0779NAS6 ◆n3AmnVhjwc 垢版2020/05/22(金) 18:21:18.85ID:???
夕焼けは太陽の角度と屈折率の角度の関係で赤いんだろ
昼間は屈折率の影響が少なく
夕方は平行光源の太陽光において
青色光がより曲がるから青抜けして赤く染まる
0780ご冗談でしょう?名無しさん垢版2020/05/22(金) 18:24:12.18ID:Z/uqjDmr
>>779
まず>>626を読みましょう
0781NAS6 ◆n3AmnVhjwc 垢版2020/05/22(金) 19:02:56.28ID:???
赤焼けは青色光が曲がって青抜け
昼間はあんまし曲がらんから青空
水槽への光入射実験
ttp://quasar.cc.osaka-kyoiku.ac.jp/~fukue/EDUCATE/Ex2000/ExIV99110854.jpg
0783ご冗談でしょう?名無しさん垢版2020/05/22(金) 20:46:46.02ID:???
フレネル反射…暗さの説明
屈折率による分光…色の説明
屈折率の境界面が光る説…スクリーンの説明(空の色)←不明(実験

0784ご冗談でしょう?名無しさん垢版2020/05/22(金) 21:48:06.36ID:Z/uqjDmr
>>626が読める環境にない人もいるのかな?
0785NAS6 ◆n3AmnVhjwc 垢版2020/05/22(金) 22:29:04.09ID:???
ん?781の実験写真で
青は良く曲がるから散乱しやすく
光が進む媒体の中の距離が短いと青長いと赤
だけだろ
0786ご冗談でしょう?名無しさん垢版2020/05/22(金) 22:41:18.05ID:???
このNAS何番とかいう人は何がしたいの?
物理の素養ゼロなの丸出しで連投してるようだけど一体何が目的なの?
0787ご冗談でしょう?名無しさん垢版2020/05/22(金) 23:05:13.83ID:Z/uqjDmr
とっくにNG入り
0789ご冗談でしょう?名無しさん垢版2020/05/22(金) 23:35:59.72ID:Z/uqjDmr
しかしトリップ入れてるだけ誠実
ここはIDもワッチョイもないから人物の判別もNGもやりにくい
0790NAS6 ◆n3AmnVhjwc 垢版2020/05/23(土) 00:48:50.77ID:???
地表で十秒かけて一回転する観測者が一光年先の地平線すれすれの星を見る
このとき6.28光年の円周運動で十秒一回転なので超光速運動である
と、相対性原理で言えちゃうじゃん
この観測系は地上座標系と優劣ないっておかしいよな
0791ご冗談でしょう?名無しさん垢版2020/05/23(土) 04:37:55.78ID:???
超光速運動ができないのは局所慣性系内でだけの話なので
回転座標系で超光速になったからといって何の問題もない。
宇宙膨張で十分遠方が超光速になってよいのも同様

結論: NASは相変わらずのオタンコNASであった
0792ご冗談でしょう?名無しさん垢版2020/05/23(土) 09:18:17.79ID:???
>>790
その程度の内容ならば、NAS6でなくとも頭が良い中高生でも推測できる。問題はそれからだ

 物理学の基本理論とは別々の運動をしている観測者が観測した他の観測者(物体)の
位置・速度の観測値を座標変換して他の観測者の観測値と一致しなければ意味がない。
 どの座標系の観測者でも座標変換することで矛盾なく観測データを共有できるという
意味でも等価原理を基にして一般相対性理論の(難解な)座標変換が成功したのが判る。
0793ご冗談でしょう?名無しさん垢版2020/05/23(土) 09:34:16.36ID:???
2000年以上前にプトレマイオスが天動説で恒星と惑星の複雑な運動を周転円という
力学原理が不明な円運動を次々に付け足すことで現実の天体観測値と一致させることは可能だが
他の天体の観測者が観測するなとハナから無視、基本的な力学原理も無いから座標変換も不可能。
0796ご冗談でしょう?名無しさん垢版2020/05/23(土) 12:28:26.06ID:bIrrLi4e
>>792
何故か《個別の問題》についての《質問》なのに、
必ずもやっとした広大な《一般論》に《論点がズレる》

天然なのか、意図的なことなのか?
0797ご冗談でしょう?名無しさん垢版2020/05/23(土) 12:34:31.32ID:bIrrLi4e
>>762
【テンプレコレクション】
「〜を求めるのに」
「〜とするアホはいない」
「〜とするバカはいない」
0798ご冗談でしょう?名無しさん垢版2020/05/23(土) 13:30:19.80ID:???
物理学が他の自然科学とくらべて大きな特徴は少数の基本仮説から演繹して
広範囲な物理現象を説明できることにある。
個々の結果的な公式をいくら丸暗記しても理解したことにはならない。
0799ご冗談でしょう?名無しさん垢版2020/05/23(土) 14:13:26.23ID:???
>>798
物理学の著しい特徴をアインシュタインは最大限活用し、自分が創造した一般相対性理論
の上にマックスウェル方程式(基本は2式)を乗せ換えることに成功した時点で
100年後の科学技術でなければ製作出来ない重力波の観測装置やスカイツリーの光格子時計
による検証結果がどうなるか、彼にとっては自明の事実でしかない。
0801ご冗談でしょう?名無しさん垢版2020/05/23(土) 16:13:30.19ID:???
>>798
そんな事言ってるけどお前
例えば相対論の講義の試験で
「等価原理とはいかのものである。このアインシュタインの思いつきだけを根拠に一般相対性理論を構築しアインシュタイン方程式を導け。ただしテンソルの計算は以下の定義で与えられ、共変微分は以下の通りである」
って出題されたら解けるの?
0802ご冗談でしょう?名無しさん垢版2020/05/23(土) 16:42:21.05ID:YEUzyl/l
「グリーンフラッシュ」について理解できました

この動画では太陽が色別に沈んでいく順番が分かりやすく示されています
https://youtu.be/nMq3cqO__Yw
ここでも理由は「屈折率の違い」の一言で済ませていますが
考えるヒントになりました

「屈折率が高い光ほど後まで見える」というのは一見直観と矛盾しますが
(高い方から先に見えなくなるように思えます)
地平線をテコ(シーソー)の支点のように考えると理解できます
太陽が地平線の位置より下降するにつれて
地平線よりこちら側に届く太陽光の下限(影の上限)ラインは上昇していきます

下限ラインの位置が眼より上になるとそれ以降は見ることができなくなりますが
この時下への屈折が大きい光ほど眼の位置に達するのが遅くなります
(後まで見ることができます)
したがって緑色光がまだ見えているのに「赤色光が頭上を超えている」
(そしてそれ以降は一切見えなくなる)という状態が1、2秒起こります
(青色光は散乱により元から消失している)
これを「グリーンフラッシュ」と呼びます

この理解でいいのではないでしょうか?
0804ご冗談でしょう?名無しさん垢版2020/05/23(土) 17:01:31.76ID:YEUzyl/l
>>803
「だめな回答」のひとつのパターンですね
(難解な物理用語を持ち出すも中身ゼロ・ピント外れ)
これ割と多いような気がします
0805ご冗談でしょう?名無しさん垢版2020/05/23(土) 17:02:24.56ID:3+hChwkk
ガチャガチャのカプセルの残りが少ない時って
どうやってガチャガチャの空振りというか、お金入れてガチャガチャした時にカプセルが出て来ない現象を防いでるんですか?
(´・ω・`)
0806ご冗談でしょう?名無しさん垢版2020/05/23(土) 17:44:54.40ID:X7ruIWhl
なんか相対論がプチブームみたいなんで…。

「…宇宙船の長さは前後方向に … の割合で縮んでいる。
「縮んで見える」のではなく、実際に物理的に縮んでいるのである。」(金子隆一)
              別冊宝島116「宇宙論が楽しくなる本」宝島社1990 より。

この件、見解が二つに別れるのですが、さあ、正解はどっちだ!?
0807ご冗談でしょう?名無しさん垢版2020/05/23(土) 17:48:31.75ID:???
ちょっと質問ってかただの思いつきだけどだけど相対論は(t,x)に対する変換(=位置を変換するなら時間も同時に変換しないといけない)が基本的な原理だけど
何らかの時空の変数αがあって(t,x,α)に対する同時変換を行えばダークマターやダークエネルギーの問題は解決するって可能性はないの?

少なくとも弦理論見てる限り、あの理論がダークマターやダークエネルギーの問題を解決する見込みはあまりなさそう
0808ご冗談でしょう?名無しさん垢版2020/05/23(土) 18:06:59.10ID:zHDd7oPF
>>645です
レスくださった方ありがとうございます
まとめてのお返事ですみません
アホすぎる質問でレスいただけないかと思ったのでうれしいしありがたいです
しかも引火の可能性があるとのこと、質問して本当によかったです
火花がとぶのはどこかおかしいからなのですね
勉強になりました
安全第一なので使わないことにします
スイッチの導入もやってみます
本当にありがとうございました
0810ご冗談でしょう?名無しさん垢版2020/05/23(土) 19:35:57.16ID:???
空間を拡張して云々、ってのはカルツァ・クライン理論(1921年、1926年)の昔からある
で、この理論は重力と電磁場だけだけど、その後素粒子が一杯発見されたので、もっと一般的なことを考えなくちゃならなくなった
単なる時空の変数の拡張だとうまくいかない、っていう定理があって、それがグラスマン数(反交換する特殊な数)だとうまくいく、という定理もある
だからそのグラスマン数をつかった超対称理論ってのが考えられた
でも超対称性は自然界で発見されたわけではない

で、弦理論もその制約をうけて、超弦理論になった
0811ご冗談でしょう?名無しさん垢版2020/05/23(土) 19:52:04.54ID:???
>>801
>等価原理とはいかのものである。このアインシュタインの思いつきだけを根拠に・・・
アホ
ニュートン力学の最初に慣性の法則が必要な理由を考えろ
馬鹿でも分るように言えば、外力が作用していない粒子は等速直線運動をする
一般相対性理論でも”等速直線運動”をする座標系を規定するということだ。
0812ご冗談でしょう?名無しさん垢版2020/05/24(日) 05:12:08.11ID:???
エネルギーは作れる(正確にいうと移動できる)が、
時間や空間はどうしてだめなのだろう

時間生成みたいなもので周囲1kmの時間を速くすすむようにできるマシン
0815ご冗談でしょう?名無しさん垢版2020/05/24(日) 09:29:13.16ID:???
別次元は時間/空間の内部に折り畳まれ,その中に含まれている。
意識宇宙、ホログラフィック宇宙、とか言われるが次元は多様性がある。
光速とは違う向こう側の次元を経由して線形時間/空間を迂回出来る可能性はある。

おそらく次の科学は空間性質と意識を知る科学になるだろうな。キーマンはスカラーポテンシャル(重力ポテンシャルを含む)だろうな。
0818ご冗談でしょう?名無しさん垢版2020/05/24(日) 09:46:12.89ID:???
マクスウェルは、エーテルがあるから電気が生まれるとした。

それは無視したのがアインシュタイン

マクスウェルとアインシュタインは相成れない存在です。

その事はマクスウェル自身、残した初期のマクスウェル方程式が掲載されている本を見れば分かる。
0819ご冗談でしょう?名無しさん垢版2020/05/24(日) 09:51:06.45ID:???
馬鹿でもできる相対運動の実験をすれば面白いことが判る。
地上に静止した物体に視線を合わせる。
次に首を振るか体を動かして見ると、物体は静止してるように見える。

誰でも当たり前の体感事実だが、物理学的に考えると不自然だ。
物理では相対運動なのだから対等なはずである。視覚の中央から物体位置は外れている。
この体感現象をどう理解すればいいかだが、人間の脳(他の動物でも)が地上を静止
させた座標系に変換したほうが脳の情報処理が簡単で生存競争に有利に働くと推測できる。
その証拠の一つは、車列の渋滞が長く続き他の車列が動き出す時、自分の車が逆走した
ように錯覚する事実から判る。
同様に地上から惑星運動を計算するより、太陽系から計算した方が遥かに簡単になる。
0824ご冗談でしょう?名無しさん垢版2020/05/24(日) 10:40:31.79ID:/Iut6k9O
物理学の話をしているとヒトの知覚の問題に行き着かざるを得ないのが面白いところではある
0825ご冗談でしょう?名無しさん垢版2020/05/24(日) 10:59:41.60ID:???
>>822
キミには >>819 の内容が理解できないということ

物理学では相対運動が基本的に対等であるはずが、視覚の認識では脳が特定の座標系に
座標変換してしまうということだ。
地上と人間の運動では地上が静止し人間が運動する座標系に変換され認識される。

よく知られた実験では上下逆に見えるゴーグルを長時間付け続けると地上に正立して
見えるようになる。目玉が180度回転したのではない。
宇宙ステーションの長期滞在ではどうなるかだが、宇宙船内の特定方向が上になるように
訓練すると推測できる。
0826ご冗談でしょう?名無しさん垢版2020/05/24(日) 11:12:02.20ID:???
>>825
涙拭けや知障。お前が VOR 知らなかったのは明らかなんだからよ↓

>誰でも当たり前の体感事実だが、物理学的に考えると不自然だ。
>物理では相対運動なのだから対等なはずである。視覚の中央から物体位置は外れている。

それとお前、「反射」抑えることが出来る↓ってんなら病気だぞ。病院逝け。

>目玉を動かさないでやっても、周りが静止して体感するということだ。
0828ご冗談でしょう?名無しさん垢版2020/05/24(日) 11:39:40.76ID:???
>>826
キミは目玉を動かさないで首や体を動かすとができないらしい。
誰でも完全には無理だがどうしても目玉の視線が動いてしまうなら、
指で瞼から目玉を押さえて実験すればよい。
0829ご冗談でしょう?名無しさん垢版2020/05/24(日) 12:05:50.53ID:???
地上を静止座標として認識するのは人間共通の脳の機能だから
近代科学以前で天動説を誰でも認めていたのが当然で宗教の教義からとかではないのが分かる。
体感認識に支配されない物理学による運動の相対性を初めて主張したのがガリレオ・ガリレイになる。
0832ご冗談でしょう?名無しさん垢版2020/05/24(日) 12:59:12.13ID:???
>>807
αを具体化できなきゃただの空論
既知の場を具体化しようとしてるのが超弦理論なのに
既知ですらないダークエネルギーは手のつけようがない
0833ご冗談でしょう?名無しさん垢版2020/05/24(日) 13:33:13.63ID:???
ここで質問するには低レベルで恐縮ですが、最近スマホやwebカメラで体温を測る話が散見されます。
私の知識では可視光ないし近赤外線で温度は測れないと思うのですか、一体どういう原理なのでしょうか。
0837ご冗談でしょう?名無しさん垢版2020/05/24(日) 14:37:21.98ID:Fzm/aF6M
物理学掲示板群 ttp://x0000.net/forum.aspx?id=2

学術巨大掲示板群: アルファ・ラボ ttp://x0000.net
物理学 化学 数学 生物学 天文学 地理地学
IT 電子 工学 国語 方言 言語学 など

PS ペンローズの量子脳理論
ttp://x0000.net/topic.aspx?id=3702-0
連続と離散を統一した!
ttp://x0000.net/topic.aspx?id=3709-0
0838ご冗談でしょう?名無しさん垢版2020/05/24(日) 15:35:46.50ID:osGJ6Mf5
鵜呑みにするな!
0840ご冗談でしょう?名無しさん垢版2020/05/24(日) 17:48:28.90ID:82SuzJss
>>830
VORじゃなくて移動不変性の話だろ
0842ご冗談でしょう?名無しさん垢版2020/05/24(日) 18:38:22.34ID:???
VORはもっと微細な外乱振動を打ち消す反射の話だろ。
でもいずれにせよ、相対性理論と全く関係ない話。
0848ご冗談でしょう?名無しさん垢版2020/05/24(日) 19:55:51.81ID:/Iut6k9O
衝動性眼球運動(視点移動)は実はめちゃくちゃ早くて
光速に近い速度を出しているので
相対論的な時空間が出現する
0849ご冗談でしょう?名無しさん垢版2020/05/24(日) 20:48:28.46ID:???
ある範囲の振幅値の間を均一(一様)にとるデータを量子化する.量子化は 一様(線形)量子化することにしたので幅dは一定である.量子化をnビットで 表現する状況から3nビットで表現する状況に変更するとき量子化雑音を何 dB低減できるか(nを用いて表現せよ).

この問題解説していただきたいです
0850ご冗談でしょう?名無しさん垢版2020/05/24(日) 21:25:53.59ID:???
工学の量子化と、物理の量子化はちょっと違うが

これはAD変換の話で
ttps://www.analog.com/media/jp/training-seminars/tutorials/MT-001_jp.pdf
一様量子化だと1ビット増えると6.02dB向上する
0853ご冗談でしょう?名無しさん垢版2020/05/24(日) 21:47:52.96ID:???
>>852
レポート問題とかだとどこまで求められてるかわからんけど、ちゃんと上のドキュメントみたいにしなけりゃならんかも
そこはこっちには目的がわからんのでなんとも
0855ご冗談でしょう?名無しさん垢版2020/05/25(月) 00:19:45.50ID:???
鏡は大きさが違ってもうつるものの大きさは変わらないのに
レンズ(例えば瞳)にうつるものはレンズの大きさによってサイズが変わるのはなぜですか?
0857ご冗談でしょう?名無しさん垢版2020/05/25(月) 00:34:22.80ID:???
先ほどから議論されてる相対性の質問です
光が宇宙の「空間」に乗って伝わるなら星が超光速で遠ざかっても超光速で空間の電気状態を歪めるだけだから空間に乗ってる光速の速度は光速で変わらず超光速で遠ざかる星も見えるとかってありえますか?
空間の電気状態が光子説、光速度不変が空間との相対速度だから説、時間空間歪まない説って可能性皆無でしょうか?
初心者なのでただの思いつきです
0858ご冗談でしょう?名無しさん垢版2020/05/25(月) 07:52:00.24ID:+YiHry2v
相対論的な時空間を再現した宇宙トラベルもののゲームとか体験アプリでいいのないかな

MIT開発の「特殊相対性理論3Dゲーム」で、時空間の歪みを勉強してみよう!
https://jp.techcrunch.com/2012/11/13/20121112experience-time-warp-with-mits-new-special-relativity-3d-educational-game/

相対論的ゲーム
https://twitter.com/hashimotostring/status/806837339917955073

こういうのはあるみたい
ミレニアム・ファルコンで宇宙飛べればいいんだけどな
https://twitter.com/5chan_nel (5ch newer account)
0859ご冗談でしょう?名無しさん垢版2020/05/25(月) 11:19:43.80ID:KafVWYxD
ポエマーだが
レスの勢い落ちてるんであえて質問する
物理の素養のある人と物理の素養のない人間どちらが面白いと思う?
0860ご冗談でしょう?名無しさん垢版2020/05/25(月) 11:30:19.36ID:???
>>859
そんな物理の素養のあるなしで人間の面白さは変わりません
ただ自分が面白いと思えてることをやれている人は人間として面白いと思います
0861ご冗談でしょう?名無しさん垢版2020/05/25(月) 11:32:49.73ID:+YiHry2v
頭が悪いことをポエマーだと誤魔化すのはあまり感心しないな
0862ご冗談でしょう?名無しさん垢版2020/05/25(月) 14:12:03.41ID:KafVWYxD
物理の素養のある人ってのはわるく言えば「学術に迎合してきた人」で「凡百」

「学術に囲われた人」で「学術に合わせる人」で「最適化された人」で「他に換えが利く人」で「凡百」

「学術通り」で「高度な知恵」で「その中で成長できる」から物理の素養のない人間より「頭がよくなる」
素養のない自分たちより「天才的」に「頭がいい」

もう一度言う「凡百な頭の良さ」

物理の素養のない人間は「凡百から弾かれた人間」
「非凡」に「頭がわるい」
「代わりになりたい奴はいない」から「換えが利かない」
「学術に恵まれないから正道な学術の代わりを求める」

神とごみってどっちが面白い?
0865ご冗談でしょう?名無しさん垢版2020/05/25(月) 14:33:55.67ID:???
>>857
光速度は光がある局所空間で光速度不変だから
地球から超光速で遠ざかってる場所からの光は
地球に向かう光でも遠ざかるから届かない

3つの説のうち「光速度不変が空間との相対速度」は
上の意味で正しいが、他は意味不明
0866ご冗談でしょう?名無しさん垢版2020/05/25(月) 15:08:10.84ID:???
>>862
君は先生の言う通りにしてテストで満点取るのが学問だと思ってる学歴コンプの高卒くんかな?
物理的常識を物理学的手法で変革できるのが非凡だよ
この意味わかる?
0867ご冗談でしょう?名無しさん垢版2020/05/25(月) 15:38:12.51ID:???
>>857
>光が宇宙の「空間」に乗って伝わるなら星が超光速で遠ざかって ...
相対性理論のような自説を書いても
・一般相対性理論の基本を理解してから分かり易い言葉に直そうとしてるのか
・一般相対性理論でない説を基に自分の説を書いてるのか
ではまったく異なる。
肝である重力場に何も言及してなから後者か、ならばマトモな他者は認めないだろう。
0868ご冗談でしょう?名無しさん垢版2020/05/25(月) 16:03:36.54ID:???
2chの頃にハンドルがタミによれば、絶対的に地上が静止して自分が運動してる自説を
展開し、他人が幾ら力学理論で説明しても全く自説を曲げなかった。
 おそらく自分で動こうが、電車に乗ろうが地上が静止して視覚認識されるという体感事実を
基に「支配の原理」なる自説の原理にしたと推測できる。
自身の”視覚認識に支配されてる”と言うべきで、物理学の原理には成りえない。
0869ご冗談でしょう?名無しさん垢版2020/05/25(月) 16:33:49.57ID:KafVWYxD
>>866
ほう。じゃあ君に期待してもいいんだな
今からこのスレに、物理学手法で非凡な、新しい、学術的意味のある、万人が評価する理論を書き込んで貰おう。俺と同じポエムでもいい。

でこのスレの住人にお知らせです
今から866がすごい理論を書き込むようです。期待しましょう

面白い理論だったら俺も面白いから割と期待するよ
0870ご冗談でしょう?名無しさん垢版2020/05/25(月) 16:39:41.03ID:KafVWYxD
ちなみに 俺自分をポエマーと呼んでるけど、住人が識別しやすいように自称してるんだが、俺がどの書き込みの者か識別してる?
してないような気がするんだが思い違いか?

学歴コンプとかいってるからしてないんだろうな
NAS6とかくっくっくとかと同じ種族なんだけど
0871ご冗談でしょう?名無しさん垢版2020/05/25(月) 16:40:18.40ID:+YiHry2v
小学生かw
0872ご冗談でしょう?名無しさん垢版2020/05/25(月) 17:09:57.36ID:+YiHry2v
自分を認識して欲しかったらコテつけてね
0876ご冗談でしょう?名無しさん垢版2020/05/26(火) 02:23:05.75ID:???
物理で使う積分は区分求積法で十分ですか?数学的にこれでまずいのはわかるのですが
物理学で使うのに限りεδ論法でないとダメな場合とかあるのでしょうか?
0880ご冗談でしょう?名無しさん垢版2020/05/26(火) 10:47:13.85ID:Wwh3rqiV
元アイドルの夫な小谷川拳次先生って誰よ?って人向け
https://fakelielife.fc2.net/
0881ご冗談でしょう?名無しさん垢版2020/05/26(火) 12:32:34.31ID:zSqunhOE
物理学掲示板群 ttp://x0000.net/forum.aspx?id=2

学術巨大掲示板群: アルファ・ラボ ttp://x0000.net
物理学 化学 数学 生物学 天文学 地理地学
IT 電子 工学 国語 方言 言語学 など

PS ペンローズの量子脳理論
ttp://x0000.net/topic.aspx?id=3702-0
連続と離散を統一した!
ttp://x0000.net/topic.aspx?id=3709-0
0883ご冗談でしょう?名無しさん垢版2020/05/26(火) 13:59:55.04ID:???
>>875
とりあえず直線上に座標とって慣性中心(重心)
を考えれば
そのあと普通に慣性モーメントもとめて適当に式変形すれば
0886脳リハビリ翁垢版2020/05/26(火) 18:18:14.93ID:???
>>875
ランダウリフシッツの力学かの?
あの本、重心周りでないモーメントは慣性モーメントと呼ばないという
最近の本ではマイナーな流儀だったはず
だから >>885 のXは0としてええんでないかのう
0887ご冗談でしょう?名無しさん垢版2020/05/27(水) 01:28:20.84ID:???
テレビのニュース番組で、コロナの感染状況や予測に使える
K値というのを紹介していました。
提唱者は中野貴志大阪大学教授と池田陽一九州大学准教授
https://note.com/yagena/n/n22215ecd9175

物理の人がコロナ感染に有用な指標を発掘したところが面白いです
0888ご冗談でしょう?名無しさん垢版2020/05/27(水) 07:40:44.10ID:???
>>857
星が超光速で遠さがると言う意味がわからない。それなら物質も超光速で行けることになる。

地球も動く、銀河系も動く、恒星系も動く
おそらく地球を基準にして赤方偏移とかで測定してんだろうけど、それって意味無いとは思う

ドップラー効果=赤方偏移があるのだからといって、空間が膨張しているとは限らない。
0890ご冗談でしょう?名無しさん垢版2020/05/27(水) 08:27:22.55ID:???
仮にそれ以前に星が超光速で移動するなら、光が光速なので、その星は見えない...

というか媒体がなければ光は生まれんでしょ
0892ご冗談でしょう?名無しさん垢版2020/05/27(水) 08:37:58.00ID:6sPZ9q6C
超高速と書きたかったんだろうな
0894ご冗談でしょう?名無しさん垢版2020/05/27(水) 09:47:06.76ID:???
>>888 >>890
>星が超光速で遠さがると言う意味がわからない。
>星が超光速で移動するなら...その星は見えない

ニュートン力学(or特殊相対論)の平行移動で推論すればそうなる。その意味なら間違いではない。
一般相対性理論ではそうはならない。宇宙論では現実の宇宙全体は一般相対性理論が成り立つ
と仮定している。

>ドップラー効果=赤方偏移があるのだからといって、空間が膨張しているとは限らない。
実際の宇宙観測によれば、銀河の赤方偏移、宇宙背景放射などが観測方向によらない。
観測では他の銀河の恒星系も太陽系と同様な原子で構成され対等と推定できる(コペルニクス原理)

ニュートン力学などで推論すれば太陽系が宇宙の中心になってしまう。(天動説)
現実の宇宙全体では一般相対性理論が成り立つと推論すれば矛盾がなくなる。
論理的に納得したければ一般相対性理論を学ぶしかない。
0895ご冗談でしょう?名無しさん垢版2020/05/27(水) 10:46:02.24ID:???
>>894
宇宙に限らず光速に関係する質問では、出題者がどの基本理論が前提なのか明確にする必要がある。
例えば           ニュートン力学  特殊相対性理論  一般相対性理論
超光速の相対運動が可能    ○         ✕        ○
超光速で離れても見える    ✕         ✕        ○
0897ご冗談でしょう?名無しさん垢版2020/05/27(水) 12:10:43.00ID:???
>>896
ニュートン力学の理論で静止座標からみて

 ・P点から出た光速c---->  ・qの速度v---------->

c < v でも qからP点から出た光が見えると言うのか?
0901ご冗談でしょう?名無しさん垢版2020/05/27(水) 12:41:17.47ID:???
膨張している証明がほしいなら、少なくとも片方が絶対静止した座標が必要になる。

方向に依らないなんて、詐欺だろ。

長さも変わる、時間も変わる、では何処を基準にしたらよいの?
0906ご冗談でしょう?名無しさん垢版2020/05/27(水) 13:41:34.58ID:???
>>901
>膨張している証明がほしいなら、少なくとも片方が絶対静止した座標が必要になる。

その様な論理がニュートン力学的な相対運動理論の最大の弱点といえる。
 アインシュタインの一般相対性理論はその弱点を解決するのが最初の目的であることが
自身の論文、解説書には書いてある。
0908ご冗談でしょう?名無しさん垢版2020/05/27(水) 13:55:41.02ID:???
>>897
光源がどんな速度であろうと電磁場の振動があることに変わりはない
電磁場の振動があれば全方向に電磁波が放射される
したがってどこからも見える

君の誤解は「P点から出た光速」の概念(相対論)
ニュートン力学と電磁気学を合わせれば
絶対静止系(エーテル)は不可避だから光速は絶対静止系でしかあり得ない
0909ご冗談でしょう?名無しさん垢版2020/05/27(水) 14:28:12.16ID:???
>>908
くっくっく の理屈みたいだな意味不明な展開するより
 >>897 Pから光がcの速度で出た想定に答えてくれないか?
ニュートン力学の差表だからcは3*10^8m/sとは限らない、小学生でも答えられと思うが
0912ご冗談でしょう?名無しさん垢版2020/05/27(水) 15:16:49.82ID:DshWLRHK
光速が3.0を超えることを観測すれば相対性理論をぶっ壊せる
そして俺が物理学に名を残すのだ
0913ご冗談でしょう?名無しさん垢版2020/05/27(水) 16:10:03.59ID:???
>>897
ニュートン力学の理論上では真空中の光速は電磁波の媒質(エーテル)との相対速度で変わる。
物理学史でも19世紀のマックスウェル、ヘルツ、ローレンツなどの天才達もそう信じていた。
マックスウェル方程式の電磁波の位相速度c=1/√μ0ε0は電磁波の媒質に静止した座標系での
普遍光速であり、その(絶対)静止座標系では進行方向と無関係に光は一定速度cで進む。

19世紀の光行差観測や地上の水流中の光速変化等の実験から電磁波の媒質はほぼ
太陽系に静止しているとニュートン力学から結論された。
地球の公転vと反対方向に進む光c'を地球上で測定すれば、c'= c+v ( c+3*10^4m/s )となる。
(超光速!)
実際に地球上で光線の往復による測定では精度的に困難で、マイケルソンとモーレー
の実験装置で初めて測定可能になった。
MM実験の否定的な結果は当時の物理学者を混乱に陥れた。
0914ご冗談でしょう?名無しさん垢版2020/05/27(水) 21:22:37.83ID:???
ニュートリノ検出に関して教えてください。
カミオカンデなんかはかなり苦労して純水を貯めて観測してるのに、
アイスキューブ(南極の氷)とか、アンタレス(地中海の深海 ※計画段階)
これらは純水には程遠く不純物だらけなのにOKなのは何故なんでしょうか?
ノイズ除去技術が格段に向上したみたいな事なのでしょうか?
0917ご冗談でしょう?名無しさん垢版2020/05/27(水) 23:54:31.25ID:???
>>916
なるほどサイズが全然違いますね。
・スーパーカミオカンデ
〜検出器は、約5万トンの純水で満たされた円筒型タンク(直径39.3m、高さ41.4m)

・アイスキューブ・ニュートリノ観測所
〜これらのセンサーは深さ方向に1km、上から見て1km^2の正六角形の領域に分布しており、全体として1km^3もの体積を持つ巨大な検出器を構成している。

※ アンタレス(ANTARES)はまだ計画段階と日経サイエンスで見た記憶があったんですが、すでに稼働してるようです。
0918ご冗談でしょう?名無しさん垢版2020/05/28(木) 00:01:20.35ID:a+LPXS5B
45度は2つの角度を持つ
0923ご冗談でしょう?名無しさん垢版2020/05/28(木) 07:42:28.03ID:???
一般相対性理論によれば音速不変の原理は時空が曲がっているからどの位置でも同時に聞こえるのだそうだ。
0925ご冗談でしょう?名無しさん垢版2020/05/28(木) 09:53:08.62ID:j+j90Lgu
>>923
平気で捏造を吹聴するのはチョンの特徴
自分を有利にするためには真実などどうでもいい
ノーベル賞をひとつも取れない民族だけのことはある
0929ご冗談でしょう?名無しさん垢版2020/05/28(木) 11:15:59.41ID:argLxcUx
チョンはノーベル賞の話を出すと脊髄反射でファビョるからおもろい
0931ご冗談でしょう?名無しさん垢版2020/05/28(木) 11:27:49.97ID:yZiJD3l2
>>927
ただそれだけが、チョンの心の支えw
0932ご冗談でしょう?名無しさん垢版2020/05/28(木) 11:43:43.44ID:ZART0DMt
ノーベル賞なんかただの賞に過ぎないよ
ありがたがりすぎ
人が与える賞だから間違いもあるし政治的思惑もある
足の引っ張り合いや取った人が権威持ちすぎる弊害もある
憲法やコロナで頓珍漢なことを言うノーベル賞受賞者もいる
0933ご冗談でしょう?名無しさん垢版2020/05/28(木) 11:46:52.94ID:???
今の馬鹿どもが何も知らずに「空間」なる言葉が使えるのは、デカルト様が直交座標を
創造してくれたお陰なのだよ。
それを学校教育で馬鹿どもに刷り込ませた成果にすぎない。
それと同時に
デカルトは同時に空虚な何もない(物理)空間など存在しない。
離れた物体同士に直接力が作用するなど有りえない。
と確信していた。現代の学校教育では何故か教えない。
0935ご冗談でしょう?名無しさん垢版2020/05/28(木) 13:20:11.65ID:???
>>933
から (任意に設定可能)空間座標と(物理)空間はイコールではないのが分かる。
「空虚な何もない(物理)空間など存在しない」ならば物理的な実在がなければならない。
アインシュタインはデカルトと同様な哲学概念を持っており、一般相対性理論の後の著作
で新たなマクロ的なエーテルの概念を述べている。
これを従来の媒質エーテル説と区別する為に仮に、ネオ・エーテルと呼ぶことにすれば
物質、電磁場、重力場がゼロの真空空間に現れる最低エネルギーのスカラー(ポテンシャル)
場とみなせる。
宇宙論の観測によれば、物質、電磁場の重力場理論では説明できない70%未知の
ダークエネルギーが、ネオ・エーテルのマクロ的な物理作用として説明可能だろう。
0936ご冗談でしょう?名無しさん垢版2020/05/28(木) 13:54:46.00ID:???
ビッグバン直後に起こったとされる素粒子論の自発的対称性の破れも
マクロのネオ・エーテルの状態変化・相転移現象として理解可能だろう。
0938ご冗談でしょう?名無しさん垢版2020/05/28(木) 16:02:22.29ID:???
>>973
勘違いしてるようだな
マクロからミクロを理解できるという意味ではない。
マクロはマクロの現象、ミクロはミクロの現象として理解可能という意味だ。

現実の宇宙の中に物質、電磁場、重力場がゼロの場所を局所的に作り出すことは可能であり
そこには「何も無い」のではなくネオ・エーテルだけが有る空間になる。
宇宙が有限ならばその外側は何なのか? 馬鹿でもいう質問だが答えは「何も無い」
宇宙の外側にはネオ・エーテルが無い。
「真空」という言葉と混同しやすいが、マクロでは物質が無い空間の意味でしかない。
0939ご冗談でしょう?名無しさん垢版2020/05/28(木) 21:29:58.53ID:???
軌道電子の配置=(基底エネルギー)位置保持エネルギー

絶対温度0Kもこの位置保持エネルギーは存在する。

エーテル=位置保持エネルギーとすると軌道電子の位置保持を変えることによって、その差分により巨大なエネルギーを引き出せることは可能であるだろう。

ポール・ディラックは電子がマイナスの無限な海に沈むという奇妙な答えを出したが、ディラックの海自体がエーテルなのだろう。
0940ご冗談でしょう?名無しさん垢版2020/05/28(木) 23:45:18.40ID:???
ディラックの海自体は物性論でフェルミエネルギーとして生きてるんだよな

まあディラックは後年「流石に当時陽電子を予言するのは不可能だった」と振り返ってるけど
0943ご冗談でしょう?名無しさん垢版2020/05/29(金) 04:41:33.46ID:PD7Lnwux
>>913
光を含めた電磁波に実体はなく、すべて電磁現象の遠隔作用である。
空間を伝わる波ではない。遠隔作用である。
空間には何もなく、光も電磁波も存在しない。

昔の人間は「光あるいは電磁波は波である」と思い込みすぎてたんだよ。
そんな波は空間には存在せんのだ。

遠隔作用の伝わる時間は距離に正比例する。
だからマイケルソンモーレーの実験で方向による違いが出なかったのは当たり前だ。
距離が同じなら方向に関係なく同じ時間で到達するからだ。

遠隔作用の間に
水などの大きな誘電率あるいは透磁率をもつ物質が存在すれば
反作用を受けるから遠隔作用の伝わる速さも変わる。

それらの物質が移動していれば実効誘電率・透磁率が変わるから
やはり遠隔作用の伝わる速さは移動速度に依存することになる。

たったこれだけのことに
いまだに気づけない大半の人類はまことにアホザルばっかだわ。

いつまでも光や電磁波は空間に実在する波であると
バカなことを思い込んでいればよい。

くっくっく
0944ご冗談でしょう?名無しさん垢版2020/05/29(金) 04:54:55.59ID:PD7Lnwux
>ニュートン力学の理論上では真空中の光速は電磁波の媒質(エーテル)との相対速度で変わる。

光は電磁現象の遠隔作用であり、遠隔作用の伝わる速さが光速である。
遠隔作用を及ぼすものと及ぼされるものの間に相対速度差があれば
c’=c−vのようになって当たり前だ。遠隔作用の実効距離が変わるからである。

接近(v<0)していれば遠隔作用は早く伝わり、
遠ざかって(v>0)いれば遅く伝わる。v>cなら当たり前だが伝わらない。

この考え方は
あたかも空間に波が存在すると考えるのとまったく同じだが
それは便宜上であり、実際にはそんな波は存在しないというのが重要なのである。

哀れ、大方のアホなサルどもは
いつまで経ってもこれに気づくことができないのだな。

くっくっく
0945ご冗談でしょう?名無しさん垢版2020/05/29(金) 05:08:55.68ID:PD7Lnwux
>デカルトは同時に空虚な何もない(物理)空間など存在しない。
>離れた物体同士に直接力が作用するなど有りえない。
>と確信していた。

名前をオカルトに変えとけよ。
何もない空間に何があるんだボケが。
磁力も電気力も重力もすべては直接力が作用する遠隔作用だってーの。

くっくっく
0947ご冗談でしょう?名無しさん垢版2020/05/29(金) 07:22:51.30ID:???
ところで光速度一定のリーマン計量を取る時空が他のリーマン多様体に対して有利である理由ってなんかあるの?
ミンコフスキー時空で考えるならまあ数学はそうなんだろうと思えなくもないけど、他のリーマン多様体の存在を意識すると何で光速度一定の宇宙に我々は張り付いてるんだと感じてしまう
0950ご冗談でしょう?名無しさん垢版2020/05/29(金) 09:39:44.71ID:???
>遠隔作用の伝わる速さは移動速度に依存する

相間くっくっくの屁理屈は笑えるよな
移動速度や間の物質などに依存しないで直接離れた距離の物体同士に力が作用すること
が「遠隔作用」の意味なんだけど。
電荷が互いに静止状態で成り立つクーロン力の公式は運動状態では厳密には成り立たない。
万有引力の法則も運動している惑星に適用した場合は「遠隔作用」と見なしてることになる。
0951ご冗談でしょう?名無しさん垢版2020/05/29(金) 09:54:41.69ID:???
日常用語で使ってる「遠隔操作」は、遠隔作用で離れたものを操作する意味ではない。
「遠隔操作」には電磁波伝搬などの「近接作用」により情報通信することで操作(制御)する。
オンラインゲームのキャラクターを動かす「遠隔操作」も物理的には「近接作用」である。
0952ご冗談でしょう?名無しさん垢版2020/05/29(金) 11:36:35.91ID:???
距離が離れた物体同士に働く力が遠隔作用か近接作用か対立の科学史は非常に興味深い。
17世紀の太陽系重力の理論では
ニュートンの遠隔作用による万有引力 VS デカルト、ライプニッツの(エーテル)渦動説
第一ラウンドはニュートンの遠隔作用説の勝利、デカルト派は概念だけ。
19世紀の電磁気力の理論では
ファラデー・マックスウェルの電磁場 VS アンペール、ガウスの遠隔作用による力
第二ラウンドはヘルツが電磁波を発見し、ファラデー・マックスウェルの近接作用の勝利。
20世紀の重力のリターンマッチ
アインシュタインによる近接作用の重力場 VS 万有引力の遠隔作用による重力
第三ラウンドは2015年の重力波の観測成功で最終的に近接作用による重力場の勝利に終わった。
0955ご冗談でしょう?名無しさん垢版2020/05/29(金) 12:38:28.91ID:???
>>952
アインシュタインの一般相対性理論では、観測場所の重力場が外部からの重力波で
変動すれば観測場所の電磁波の速度がどのように変化するか計算することが出来る。
その計算結果が正しいことは、2017年の中性子性合体による重力波観測で後から来る
ガンマ線観測と方位を予言したことで証明された。

注意:遠隔作用の万有引力理論では重力波は理論的に存在しない。また
特殊相対性理論だけでも重力波は存在せず、近距離で質量物体を動かしても光不変となる。
0959ご冗談でしょう?名無しさん垢版2020/05/29(金) 13:20:53.57ID:???
>>952
第二ラウンド戦で結果的に
小学校も卒業していないファラデーが数学の絶対王者ガウスに勝利した理由は何か?
物理学と数学の違いといえる、ファラデーが実験物理学の天才であり非常に多くの
実験結果から帰納的に推論して(真空)空間中に独立した電磁力の源があると洞察した。
対して数学者は公理的に演繹した理論全体が無矛盾であるかが重要であり、現実の
物理現象に従う必要はない。
0961ご冗談でしょう?名無しさん垢版2020/05/29(金) 14:31:25.39ID:???
>>959
ガウスやラプラスはガウスの法則のベクトル、ラプラスのポテンシャルなどが
逆二乗力の法則の数学変形としてしか見ず、物理的な実在とは考えていない。
0962956垢版2020/05/29(金) 14:52:03.09ID:m/S+4qc3
>>960

エネルギー等分配則か、なるほど

個々の粒子のシュレーディンガー方程式ではエネルギーは温度に依存しないけど、
粒子の集合として見れば、温度はあるエネルギーを持つ状態にある粒子数の期待値を与える量なのか。
それがマクロな性質から決まる熱力学的温度と結びつけられてるってことでいいのかな。
0963ご冗談でしょう?名無しさん垢版2020/05/29(金) 15:24:53.53ID:qmjKfPeH
物理学掲示板群 ttp://x0000.net/forum.aspx?id=2

学術巨大掲示板群: アルファ・ラボ ttp://x0000.net
物理学 化学 数学 生物学 天文学 地理地学
IT 電子 工学 国語 方言 言語学 など

PS ペンローズの量子脳理論
ttp://x0000.net/topic.aspx?id=3702-0
連続と離散を統一した!
ttp://x0000.net/topic.aspx?id=3709-0
0966ご冗談でしょう?名無しさん垢版2020/05/29(金) 17:34:28.35ID:ynllsiri
データの改竄が多いのは医療系
ここは未だに科学が成り立っていない象牙の塔
金と権威と権力が大事
0967ご冗談でしょう?名無しさん垢版2020/05/29(金) 17:34:48.30ID:ynllsiri
日本の話ね
0970ご冗談でしょう?名無しさん垢版2020/05/29(金) 17:44:53.47ID:???
>小保方もそうだった
高分子と生物の中間的なコロナウィルスでさえ全世界の生物学者が研究しても解明が困難。
まして生物細胞なら現象が複雑すぎて意図的に改竄・捏造された研究論文を暴くのは難しい。

その点では現代物理学の実験装置は巨大で複雑だが、物理実験の目的は極めてシンプルで
改竄・捏造などすれば直ぐバレる。
0972ご冗談でしょう?名無しさん垢版2020/05/29(金) 17:56:31.16ID:???
>>969
中性子星合体と思われる重力波が観測されたので、重力波源と思われる領域を
電磁波でも観測してくれと世界中の天文台に速報依頼したら、実際に世界中の
あちこちの天文台で電磁波を観測。
さらにガンマ線天文衛星でも重力波観測と同時刻にガンマ線バーストを観測していた。

これらの観測事実に、解釈次第で変わるどんな結果があるというのか
0973ご冗談でしょう?名無しさん垢版2020/05/29(金) 17:59:17.39ID:???
>>972
LIGOはマイケルソンモーリーのでかい実験装置で、波の揺らぎがあったと...
それで重力波を観測していますね。それは間違いがないでしょう。
0974ご冗談でしょう?名無しさん垢版2020/05/29(金) 18:06:22.52ID:???
>>973
何が言いたいかわからん。マイケルソンモーリー型干渉計そのものが嘘ごまかしと言いたいのか?
だとしたら、その嘘ごまかしでどうやって、電磁波による観測の前に中性子星合体による
超新星が発生していることを予言できたんだ?
0975ご冗談でしょう?名無しさん垢版2020/05/29(金) 18:08:43.00ID:???
簡単に言えば
素粒子の質量を高精度で何億回も測定したり、レーザー光を数百数千回反射させて
超高精度の光速を比較測定する単純目的以外になく。
その結果の物理解釈もすでに承認された素粒子理論や相対性理論の上での解釈しか行わない。
つまりその理論と一致しているか、誤差以上にズレてるかのシンプル解釈であって
改竄・捏造しようとする意味すらない。
0976ご冗談でしょう?名無しさん垢版2020/05/29(金) 18:09:07.55ID:???
>>974
マイケルソン干渉計は正しく動作していますよ?

何を言いたいのかわかりませんな。
重力波を観測したのだからいいじゃあありませんか?
0977ご冗談でしょう?名無しさん垢版2020/05/29(金) 19:30:06.33ID:???
>>975
現代の実験物理学者と同じく、マイケルソン・モーリーの実験目的は当時の物理学者
に認められた恒星・太陽系にほぼ静止したエーテル(光媒質)のニュートン力学理論
により地球は静止エーテル中を貫通して運動する。(光行差観測、水流中の光速観測より)
マイケルソン・モーリーの干渉計で静止エーテルと地球の相対速度を精密観測する為である。
実験では観測できず、当時認められたニュートン力学の静止エーテル説と一致しない。

当時の物理学者達がエーテルに拘った理由は、光は電磁波の波動であると確認しており
波動(波動方程式)一定速度には物質的な媒質が不可欠と信じていてからと推測できる。
0979ご冗談でしょう?名無しさん垢版2020/05/29(金) 20:43:23.35ID:???
>>977
>当時の物理学者達がエーテルに拘った理由
中高教育レベルでは仮想的な波の速度や、静止した均一な媒質中の音速が一定と天下り的に教育される
空気中の音速は実際の測定値で与えられると思ってる大人が大多数だろう。
理工系の学生ならば物質の定数から波動方程式を導出する過程で一定の伝搬速度がでてくることが解る。
伝搬速度 = √(弾性率/密度) のようになる。光(電磁波)ではどうなるか?
エーテルの弾性率 1/ε0  エーテルの密度 μ0 とすれば
光速 = 電磁波の伝搬速度 = √(1/ε0 / μ0) = 1/√(ε0*μ0)
電磁気学的にも1/εは電気力の強さ、μは電流の慣性に比例するからエーテルの定数と見なせる。
0981ご冗談でしょう?名無しさん垢版2020/05/29(金) 21:42:13.32ID:???
氷で冷やした水を飲むことを考えます
冷えた後も氷が残るくらいの氷の量です
水のおかわりをする時って水全部飲んでから新しい水入れた方がいいですか?
それとも半分くらい飲んだら新しい水を入れる方が効率いいんですかね
0982ご冗談でしょう?名無しさん垢版2020/05/29(金) 21:48:49.33ID:???
>>977
繰り返しだが
恒星・太陽系にほぼ静止したエーテル(光媒質)と地球は静止エーテル中を貫通して運動する。
という19世紀当時の物理理論を検証するのがマイケルソン・モーリー実験の目的である。
それ以外の説うんぬんは問題外!
相対速度の予想値は換算してほぼ地球の公転速度30km/sにならなければならない。
MM実験の結果の相対速度(エーテル風速) < 8km/s
エーテル理論の予想値の26%以下の実験値から、エーテル理論と一致しないと結論した。
マトモな人なら以上の結論は正しいと追認するだろう。検証実験とは何なのかも解る。

実際のMM実験では干渉縞の移動は起こっている。それ自体は問題でないということ。
0985ご冗談でしょう?名無しさん垢版2020/05/29(金) 22:39:02.90ID:???
>>984
我々がたまたま特殊相対性理論の成り立つリーマン多様体に張り付いているだけって答えはただの人間原理じゃね?
ミンコフスキー時空や一般相対論の成り立つリーマン時空しか見えてなかった頃は「相対論は量子論と違って決定論的で綺麗だな」程度にしか思ってなかったけど
多様体を真面目に勉強したら「何故この多様体に我々は張り付いているのか」が疑問に思えてきた

マルチバースとかはあまり信じていません
0987ご冗談でしょう?名無しさん垢版2020/05/30(土) 00:31:12.98ID:???
>>986
ぬるい水飲んで氷を齧るのと、氷を完全に融かした冷たい水を飲むのが同じ快を得られるかどうか
個人的には後者の方が快を得られると思うが、こういう話をどう定式化するか
0988ご冗談でしょう?名無しさん垢版2020/05/30(土) 02:46:14.87ID:???
虫眼鏡はどんな波長でも光を集める作用は同じですか

ドアノブのコロナを虫眼鏡で太陽光を集めて殺菌(紫外線が増える)、
スマホの電波が弱い時に虫眼鏡で通信が復活できるか
0990ご冗談でしょう?名無しさん垢版2020/05/30(土) 03:31:03.39ID:???
https://i.imgur.com/0iwhkVj.jpg
最小作用の原理について見たのですが
最速降下曲線でオイラーラグランジュ方程式に似た式が出てきて
その式に出てくる関数Tが任意だからこの式が運動方程式と同じになるようにラグランジアンを決めてもいい
それならそうしよう
という流れになりました

この話の流れの中で座標qはいつ一般化されているのですか?
この式ができた時点で一般化されていますか
https://i.imgur.com/dNUt7g2.jpg
それともこの後に運動方程式に合わせてラグランジアンを決めていく中で一般化されるのですか
0991ご冗談でしょう?名無しさん垢版2020/05/30(土) 03:46:41.24ID:???
「何らかの量Iを最小にする軌道q(t)」とした時点で(x,y)で書いても(r,θ)で書いてもいい形になってるから
この時既に座標は一般化されてると考えていいですか?
0993ご冗談でしょう?名無しさん垢版2020/05/30(土) 08:45:21.58ID:QmJZN6QX
原資はスカスカなのに、
それが集まってる、我々の体や周囲の物など、なぜ視認できるのでしょうか?

可視光を反射してるのは、原子核や電子ではなく、原子全体が反射してるということでしょうか?
0994ご冗談でしょう?名無しさん垢版2020/05/30(土) 10:08:19.48ID:???
>>993
原子がスカスカ という様な意味は物質中の電荷や電磁場の作用を無視した表現。

光は電磁波であり電磁場の波が伝搬する、外部からの電磁波は原子核や電子の電荷と
相互作用して反射または吸収される。
0996ご冗談でしょう?名無しさん垢版2020/05/30(土) 10:18:31.72ID:???
>>993
光が入ると、基底状態から励起してしばらくすると基底状態に戻る。その際に、蛍光を発する。

つまり原子や分子には反応しやすい波長と言うものがあって、それに反応して励起する。しばらくして元の状態に戻る際に、その物質の色を発光する。

原子は光を反射するのではなく光を吸収して発光するといった方が正しい。
0997ご冗談でしょう?名無しさん垢版2020/05/30(土) 10:27:35.25ID:???
>>994
電荷を持つ原子核と電子で構成する原子が密に詰まった物質は外部からの電磁波反射
だけでなく外部から移動する電子(電流)も反射する。
したがって物質はほとんど不導体になるが、一部の金属物質は自由に電子が移動する
導体が存在する。
ガラスなど透明物質と同様に原子理論の謎だったが、量子力学によって解決された。
0998ご冗談でしょう?名無しさん垢版2020/05/30(土) 10:37:26.55ID:???
>>993
原子核と電荷の間は真空だしその空間はエーテルだとすればエーテル相互作用して物質対応する光を出している。

で解決できる。
10011001垢版Over 1000Thread
このスレッドは1000を超えました。
新しいスレッドを立ててください。
life time: 23日 11時間 47分 25秒
10021002垢版Over 1000Thread
5ちゃんねるの運営はプレミアム会員の皆さまに支えられています。
運営にご協力お願いいたします。


───────────────────
《プレミアム会員の主な特典》
★ 5ちゃんねる専用ブラウザからの広告除去
★ 5ちゃんねるの過去ログを取得
★ 書き込み規制の緩和
───────────────────

会員登録には個人情報は一切必要ありません。
月300円から匿名でご購入いただけます。

▼ プレミアム会員登録はこちら ▼
https://premium.5ch.net/

▼ 浪人ログインはこちら ▼
https://login.5ch.net/login.php
レス数が1000を超えています。これ以上書き込みはできません。

ニューススポーツなんでも実況